MedSurg Final Exam

¡Supera tus tareas y exámenes ahora con Quizwiz!

The nurse assesses a client after a total hip arthroplasty. The client's surgical leg is visibly shorter than the other one and the client reports extreme pain. While a co-worker calls the surgeon, what action by the nurse is appropriate? a. Assess neurovascular status in both legs. b. Elevate the surgical leg and apply ice. c. Prepare to administer pain medication. d. Try to place the surgical leg in abduction.

ANS: A This client has signs and symptoms of hip dislocation, a potential complication of this surgery. Hip dislocation can cause neurovascular compromise. The nurse would assess neurovascular status while comparing both legs. The nurse would not try to move the extremity to elevate or abduct it. Pain medication may be administered if possible, but first the nurse would thoroughly assess the client.

A nurse is caring for a client with diabetes mellitus who has fractured her arm. Which action would the nurse take first? a. Remove the medical alert bracelet from the fractured arm. b. Immobilize the arm by splinting the fractured site. c. Place the client in a supine position with a warm blanket. d. Cover any open areas with a sterile dressing.

ANS: A A client's medical alert bracelet or any other jewelry would be removed from the fractured arm before the affected extremity swells. Immobilization, positioning, and dressing should occur after the bracelet is removed.

A client with HIV-III is hospitalized and has weeping Kaposi sarcoma lesions. The nurse dresses them with sterile gauze. When changing these dressings, which action is most important for the nurse's safety? a. Adhering to Standard Precautions b. Assessing tolerance to dressing changes c. Performing hand hygiene before and after care d. Disposing of soiled dressings properly

ANS: A All of the actions are important, but due to the infectious nature of this illness, the nurse would ensure he or she is following Standard Precautions (and Transmission-Based Precautions when necessary) to avoid a potential exposure.

The nurse is teaching a client who is prescribed acetaminophen for control of osteoarthritic joint pain. What statement by the client indicates a need for further teaching? a. "I won't take more than 5000 mg of this drug each day." b. "I'll follow up to get my lab tests done to check my liver c. "I'll check drugs that I take for acetaminophen in them." d. "I can use topical patches and creams to help relieve pain."

ANS: A All of the choices are correct about acetaminophen except that the maximum daily dosage is 4000 mg. For older adults, 3000 mg are recommended due to slower drug metabolism by the liver.

A nurse is teaching a client newly diagnosed with osteoarthritis (OA) about drugs used to treat the disease. For which drug does the nurse plan health teaching? a. Acetaminophen b. Cyclobenzaprine hydrochloride c. Hyaluronate d. Ibuprofen

ANS: A All of these drugs may be appropriate to treat OA. However, the first-line drug is acetaminophen. Cyclobenzaprine is a muscle relaxant given to treat muscle spasms. Hyaluronate is a synthetic joint fluid implant. Ibuprofen is a nonsteroidal anti-inflammatory drug.

The nurse is presenting information to a community group on safer sex practices. The nurse would teach that which sexual practice is the riskiest? a. Anal intercourse b. Masturbation c. Oral sex d. Vaginal intercourse

ANS: A Anal intercourse is the riskiest sexual practice because the fragile anal tissue can tear, creating a portal of entry for human immune deficiency virus in addition to providing mucus membrane contact with the virus.

After teaching a client who is recovering from a vertebroplasty, the nurse assesses the patient's understanding. Which statement by the client indicates a need for additional teaching? a. "I can drive myself home after the procedure." b. "I will monitor the puncture site for signs of infection." c. "I can start walking tomorrow and increase my activity slowly." d. "I will remove the dressing the day after discharge."

ANS: A Before discharge, a client who has a vertebroplasty would be taught to avoid driving or operating machinery for the first 24 hours. The client should monitor the puncture site for signs of infection. Usual activities can resume slowly, including walking and slowly increasing activity over the next few days. The client should keep the dressing dry and remove it the next day.

A client who has rheumatoid arthritis is prescribed etanercept. What health teaching by the nurse about this drug is appropriate? a. Giving subcutaneous injections b. Having a chest x-ray once a year c. Taking the medication with food d. Using heat on the injection site

ANS: A Etanercept is given as a subcutaneous injection twice a week. The nurse would teach the client how to self-administer the medication. The other options are not appropriate for etanercept.

A client has been advised to perform weight-bearing exercises to help slow bone loss, but has not followed this advice. What response by the nurse is appropriate at this time? a. Ask the client about fear of falling. b. Instruct the client to increase calcium. c. Suggest other exercises the client can do. d. Tell the client to try weight lifting.

ANS: A Fear of falling can limit participation in activity. The nurse would first assess if the client has this fear and then offer suggestions for dealing with it. The client may or may not need extra calcium, other exercises, or weight lifting.

A nurse evaluates a client with acute glomerulonephritis (GN). Which assessment finding would the nurse recognize as a positive response to the prescribed treatment? a. The client lost 11 lb (5 kg) in the past 10 days. b. The client's urine specific gravity is 1.048. c. No blood is observed in the client's urine. d. The client's blood pressure is 152/88 mm Hg.

ANS: A Fluid retention is a major feature of acute GN. This weight loss represents fluid loss, indicating that the glomeruli are performing the function of filtration. A urine specific gravity of 1.048 is high. Blood is not usually seen in GN, so this finding would be expected. A blood pressure of 152/88 mm Hg is too high; this may indicate kidney damage or fluid overload.

A client is taking furosemide 40 mg/day for management of early chronic kidney disease (CKD). To assess the therapeutic effect of the medication, what action of the nurse is best? a. Obtain daily weights of the client. b. Auscultate heart and breath sounds. c. Palpate the client's abdomen. d. Assess the client's diet history.

ANS: A Furosemide is a loop diuretic that helps reduce fluid overload and hypertension in patients with early stages of CKD. One kilogram of weight equals about 1 L of fluid retained in the client, so daily weights are necessary to monitor the response of the client to the medication. Heart and breath sounds would be assessed if there is fluid retention, as in heart failure. Palpation of the client's abdomen is not necessary, but the nurse would check for edema. The diet history of the client would be helpful to assess electrolyte replacement since potassium is lost with this diuretic, but this does not assess the effectiveness of the medication.

After teaching a client who has stress incontinence, the nurse assesses the client's understanding. Which statement made by the client indicates a need for further teaching? a. "I will limit my total intake of fluids." b. "I must avoid drinking alcoholic beverages." c. "I must avoid drinking caffeinated beverages." d. "I shall try to lose about 10% of my body weight."

ANS: A Limiting fluids concentrates urine and can irritate tissues, leading to increased incontinence or cystitis. Many people try to manage incontinence by limiting fluids. Alcoholic and caffeinated beverages are bladder stimulants. Obesity increases intra-abdominal pressure, causing incontinence.

A client has a metastatic bone tumor in the left leg. What action by the nurse is appropriate? a. Administer pain medication as prescribed. b. Elevate the extremity and apply moist heat. c. Teach the client about amputation care. d. Place the client on protective precautions.

ANS: A Pain medication should be given to control metastatic bone pain. Elevation and heat may or may not be helpful. Protective precautions are not needed for this client.

A 70-kg adult client with chronic kidney disease (CKD) is on a 40-g protein diet. The patient has a reduced glomerular filtration rate and is not undergoing dialysis. Which result would be of most concern to the nurse? a. Albumin level of 2.5 g/dL (3.63 mcmol/L) b. Phosphorus level of 5 mg/dL (1.62 mmol/L) c. Sodium level of 135 mEq/L (135 mmol/L) d. Potassium level of 5.5 mEq/L (5.5 mmol/L)

ANS: A Protein restriction is necessary with CKD due to the buildup of waste products from protein breakdown. The nurse would be concerned with the low albumin level since this indicates that the protein in the diet is not enough for the client's metabolic needs. The electrolyte values are not related to the protein-restricted diet.

A nurse cares for a client placed in skeletal traction. The client asks, "What is the primary purpose of this type of traction?" How would the nurse respond? a. "Skeletal traction will assist in realigning your fractured bone." b. "This treatment will prevent future complications and back pain." c. "Traction decreases muscle spasms that occur with a fracture." d. "This type of traction minimizes damage as a result of fracture treatment."

ANS: A Skeletal traction pins or screws are surgically inserted into the bone to aid in bone alignment. As a last resort, traction can be used to relieve pain, decrease muscle spasm, and prevent or correct deformity and tissue damage. These are not primary purposes of skeletal traction.

A nurse obtains the health history of a client with a suspected diagnosis of bladder cancer. Which question would the nurse ask when determining this client's risk factors? a. "Do you smoke cigarettes?" b. "Do you use any alcohol?" c. "Do you use recreational drugs?" d. "Do you take any prescription drugs?"

ANS: A Smoking is known to be a factor that greatly increases the risk of bladder cancer. Alcohol use, recreational drug use, and prescription drug use (except medications that contain phenacetin) are not known to markedly increase the risk of developing bladder cancer.

An HIV-negative client who has an HIV-positive partner asks the nurse about receiving tenofovir/emtricitabine. What information is most important to teach the client about this drug? a. Does not reduce the need for safe sex practices. b. Has been taken off the market due to increases in cancer. c. Reduces the number of HIV tests you will need. d. Is only used for postexposure prophylaxis.

ANS: A Tenofovir/emtricitabine is a newer drug used for preexposure prophylaxis and appears to reduce transmission of human immune deficiency virus (HIV) from known HIV-positive people to HIV-negative people. The drug does not reduce the need for practicing safe sex. Since the drug can lead to drug resistance if used, clients will still need HIV testing every 3 months. This drug has not been taken off the market and is not used for postexposure prophylaxis.

A nurse is talking with a client about a negative enzyme-linked immunosorbent assay (ELISA) test for human immune deficiency virus (HIV). The test is negative and the client states "Whew! I was really worried about that result." What action by the nurse is most important? a. Assess the client's sexual activity and patterns. b. Express happiness over the test result. c. Remind the client about safer sex practices. d. Tell the client to be retested in 3 months.

ANS: A The ELISA test can be falsely negative if testing occurs after the client has become infected but prior to making antibodies to HIV. This period of time is known as the window period and can last up to 21 days. The confirmatory Western Blot test takes an additional 7 days, so using that testing algorithm, the client's status may not truly be known for up to 28 days. The client may have had exposure that has not yet been confirmed. The nurse needs to assess the client's sexual behavior further to determine the proper response. The other actions are not the most important, but discussing safer sex practices is always appropriate. Testing would be recommended every 3 months for someone engaging in high risk behaviors.

A client has a serum potassium level of 6.5 mEq/L (6.5 mmol/L), a serum creatinine level of 2 mg/dL (176 mcmol/L), and a urine output of 350 mL/day. What is the best action by the nurse? a. Place the client on a cardiac monitor immediately. b. Teach the client to limit high-potassium foods. c. Continue to monitor the client's intake and output. d. Ask to have the laboratory redraw the blood specimen.

ANS: A The best action by the nurse would be to check the cardiac status with a monitor. High-potassium levels can lead to dysrhythmias. The other choices are logical nursing interventions for acute kidney injury but not the best immediate action.

The nurse is teaching a client how to use a cane after a right surgical fractured fibula repair. What health teaching would the nurse include? a. "Place the cane on your left side." b. "Move the cane and your left leg at the same time." c. "Be sure the cane is parallel to your waist." d. "Use the cane only when your right leg is painful."

ANS: A The cane should be placed on the unaffected side (left for this client) and moved forward with the injured leg (right for this client) to provide support. The cane should be parallel to the stylus of the wrist and used at all times when ambulating.

A nurse assesses an older adult who was admitted 2 days ago with a fractured hip. The nurse notes that the client is confused and restless with an oxygen saturation of 88%. Which action would the nurse take first? a. Administer oxygen via nasal cannula. b. Re-position to a semi-Fowler position. c. Increase the intravenous flow rate. d. Assess response to pain medication.

ANS: A The client is at high risk for a fat embolism syndrome and pulmonary embolus. Although these complications are life-threatening emergencies, the nurse would administer oxygen first and then notify the primary health care provider. Oxygen administration can reduce the risk for cerebral damage from hypoxia. Pain medication most likely would not cause the client to be restless.

A client who had a surgical fractured femur repair reports new-onset shortness of breath and increased respirations. What is the nurse's first action? a. Place the client in a high-Fowler position. b. Document the client's oxygen saturation level. c. Start oxygen therapy at 2 L/min via nasal cannula. d. Contact the primary health care provider.

ANS: A The client is experiencing respiratory distress which could be due to pulmonary embolus, fat embolism syndrome, or anxiety. Regardless of the cause, the nurse would place the client in a sitting position first and then perform additional assessment. Oxygen would likely be needed, especially if the client's oxygen saturation was under 95%.

A client has been hospitalized with an opportunistic infection secondary to HIV-III. The client's partner is listed as the emergency contact, but the client's mother insists that she should be listed instead. What action by the nurse is best? a. Contact the social worker to assist the client with advance directives. b. Ignore the mother; the client does not want her to be involved. c. Let the client know, gently, that nurses cannot be involved in these disputes. d. Tell the client that, legally, the mother is the emergency contact.

ANS: A The client should make his or her wishes known and formalize them through advance directives. The nurse would help the client by contacting someone to help with this process. Ignoring the mother or telling the client that nurses cannot be involved does not help the situation. Legal statutes vary by state, but the nurse would be the client's advocate and help ensure his or her wishes are met.

A nurse teaches a young female client who is prescribed cephalexin for a urinary tract infection. Which statement would the nurse include in this client's teaching? a. "Use a second form of birth control while on this medication." b. "You will experience increased menstrual bleeding while on this drug." c. "You may experience an irregular heartbeat while on this drug." d. "Watch for blood in your urine while taking this medication."

ANS: A The client should use a second form of birth control because antibiotic therapy reduces the effectiveness of estrogen-containing contraceptives. She should not experience increased menstrual bleeding, an irregular heartbeat, or blood in her urine while taking the drug.

A client with chronic kidney disease (CKD) has an elevated serum phosphorus level. What drug would the nurse anticipate to be prescribed for this client? a. Calcium acetate b. Doxycyline c. Magnesium sulfate d. Lisinopril

ANS: A The client with CKD often has a high phosphorus level which tends to lower the calcium level in an inverse relationship, and causes osteodystrophy. To prevent this bone disease, the client needs to take a drug that can bind with phosphorus for elimination via the GI tract. When phosphorus is lowered to within normal limits, normal calcium levels may be restored.

A client is getting out of bed into the chair for the first time after an uncemented total hip arthroplasty. What action by the nurse is appropriate? a. Have adequate help to transfer the patient. b. Provide socks so the patient can slide easier. c. Tell the patient full weight bearing is allowed. d. Use a footstool to elevate the patient's leg.

ANS: A The client with an uncemented hip will be on toe-touch only after surgery. The nurse would ensure there is adequate help to transfer the patient while preventing falls. Slippery socks may cause a fall. Elevating the leg is not going to assist with the client's transfer.

A client with HIV-II is hospitalized for an unrelated condition, and several medications are prescribed in addition to the regimen already being used. What action by the nurse is most important? a. Consult with the pharmacy about drug interactions. b. Ensure that the client understands the new medications. c. Give the new drugs without considering the old ones. d. Schedule all medications at standard times.

ANS: A The drug regimen for someone with HIV/AIDS is complex and consists of many medications that must be given at specific times of the day, and that have many interactions with other drugs and food. The nurse would consult with a pharmacist about possible interactions. Client teaching is important but does not take precedence over ensuring the medications do not interfere with each other, which could lead to drug resistance or a resurgence of symptoms.

A client with bone cancer is hospitalized for a limb salvage procedure. How can the nurse best address the client's psychosocial needs? a. Assess the client's coping skills and support systems. b. Explain that the surgery leads to a longer life expectancy. c. Refer the client to the social worker or hospital chaplain. d. Reinforce physical therapy to aid with ambulating normally.

ANS: A The first step in the nursing process is assessment. The nurse would assess coping skills and possible support systems that will be helpful in this client's treatment. Explaining that a limb salvage procedure will extend life does not address the client's psychosocial needs. Referrals may be necessary, but the nurse should assess first. Reinforcing physical therapy is also helpful but again does not address the psychosocial needs of the client.

A client with chronic kidney disease (CKD) is refusing to take his medication and has missed two hemodialysis appointments. What is the best initial action for the nurse? a. Discuss what the treatment regimen means to the client. b. Refer the client to a mental health nurse practitioner. c. Reschedule the appointments to another date and time. d. Discuss the option of peritoneal dialysis.

ANS: A The initial action for the nurse is to assess anxiety, coping styles, and the client's acceptance of the required treatment for CKD. The client may be in denial of the diagnosis. While rescheduling hemodialysis appointments may help, and referral to a mental health practitioner and the possibility of peritoneal dialysis are all viable options, assessment of the client's acceptance of the treatment would come first.

A client comes into the emergency department with a serum creatinine of 2.2 mg/dL (1944 mcmol/L) and a blood urea nitrogen (BUN) of 24 mL/dL (8.57 mmol/L). What question would the nurse ask first when taking this client's history? a. "Have you been taking any aspirin, ibuprofen, or naproxen recently?" b. "Do you have anyone in your family with renal failure?" c. "Have you had a diet that is low in protein recently?" d. "Has a relative had a kidney transplant lately?"

ANS: A There are some medications that are nephrotoxic, such as the nonsteroidal anti-inflammatory drugs ibuprofen, aspirin, and naproxen. This would be a good question to initially ask the patient since both the serum creatinine and BUN are elevated, indicating some renal problems. A diet high in protein could be a factor in an increased BUN.

A nurse assesses a client who presents with renal calculi. Which question would the nurse ask? a. "Do any of your family members have this problem?" b. "Do you drink any cranberry juice?" c. "Do you urinate after sexual intercourse?" d. "Do you experience burning with urination?"

ANS: A There is a strong association between family history and stone formation and recurrence. Nephrolithiasis is associated with many genetic variations; therefore, the nurse should ask whether other family members have also had renal stones. The other questions do not refer to renal calculi but instead are questions that should be asked of a patient with a urinary tract infection.

A marathon runner comes into the clinic and states "I have not urinated very much in the last few days." The nurse notes a heart rate of 110 beats/min and a blood pressure of 86/58 mm Hg. Which action by the nurse is most appropriate? a. Give the client a bottle of water immediately. b. Start an intravenous line for fluids. c. Teach the patient to drink 2 to 3 L of water daily. d. Perform an electrocardiogram.

ANS: A This athlete is mildly dehydrated as evidenced by the higher heart rate and lower blood pressure. The nurse can start hydrating the client with a bottle of water first, followed by teaching the patient to drink 2 to 3 L of water each day. An intravenous line may be needed later, after the patient's degree of dehydration is assessed. An electrocardiogram is not necessary at this time.

The nurse is caring for a client diagnosed with HIV-II. The client's CD4+ cell count is 399/mm3 (0.399 × 109 /L). What action by the nurse is best? a. Counsel the client on safer sex practices/abstinence. b. Encourage the client to abstain from alcohol. c. Facilitate genetic testing for CD4+ CCR5/CXCR4 co-receptors. d. Help the client plan high-protein/iron meals

ANS: A This client is in the Centers for Disease Control and Prevention HIV-II case definition group. He or she remains highly infectious and would be counseled on either safer sex practices or abstinence. Abstaining from alcohol is healthy but not required, although some medications may need to be taken while abstaining. Genetic testing is not commonly done, but an alteration on the CCR5/CXCR4 co-receptors is seen in long-term nonprogressors. High-protein/iron meals are important for people who are immunosuppressed, but helping to plan them does not take precedence over stopping the spread of the disease.

A client has just been informed of a positive HIV test. The client is distraught and does not know what to do. What intervention by the nurse is best? a. Assess the client for support systems. b. Determine if a clergy member would help. c. Explain legal requirements to tell sex partners. d. Offer to tell the family for the client.

ANS: A This client needs the assistance of support systems. The nurse would help the client identify them and what role they can play in supporting him or her. A clergy member may or may not be welcome. Positive HIV test results are reportable in all 50 states, Washington, D.C., and Canada but the nurse works with the client to support his or her choices in disclosure. The nurse would not tell the family for the client.

What information does the nurse teach a women's group about osteoporosis? a. "Primary osteoporosis occurs in postmenopausal women due to lack of estrogen." b. "Men actually have higher rates of the disease but are underdiagnosed." c. "There is no way to prevent or slow osteoporosis after menopause." d. "Women and men have an equal chance of getting osteoporosis."

ANS: A Women are more at risk of developing primary osteoporosis after menopause due to the lack of estrogen. Men have a slower loss of bone after the age of 75. Many treatments are now available for women to slow osteoporosis after menopause.

A nurse teaches a client about self-management after experiencing a urinary calculus treated by lithotripsy. Which statements would the nurse include in this client's discharge teaching? (Select all that apply.) a. "Finish the prescribed antibiotic even if you are feeling better." b. "Drink at least 3 L of fluid each day." c. "The bruising on your back may take several weeks to resolve." d. "Report any blood present in your urine." e. "It is normal to experience pain and difficulty urinating."

ANS: A, B, C The client should be taught to finish the prescribed antibiotic to ensure that he or she does not get a urinary tract infection. The client should drink at least 3 L of fluid daily to dilute potential stone-forming crystals, prevent dehydration, and promote urine flow. After lithotripsy, the client should expect bruising that may take several weeks to resolve. The client should also experience blood in the urine for several days. The client should report any pain, fever, chills, or difficulty with urination to the primary health care provider as these may signal the beginning of an infection or the formation of another stone.

The nurse is caring for five clients on the medical-surgical unit. Which clients would the nurse consider to be at risk for postrenal acute kidney injury (AKI)? (Select all that apply.) a. Client with prostate cancer b. Client with blood clots in the urinary tract c. Client with ureterolithiasis d. Client with severe burns e. Client with lupus

ANS: A, B, C Urine flow obstruction, such as prostate cancer, blood clots in the urinary tract, and kidney stones (ureterolithiasis), causes postrenal AKI. Severe burns would be a prerenal cause. Lupus would be an intrarenal cause for AKI.

A nurse is learning about human immune deficiency virus (HIV) infection. Which statements about HIV infection are correct? (Select all that apply.) a. CD4+ cells begin to create new HIV virus particles. b. Antibodies produced are incomplete and do not function well. c. Macrophages stop functioning properly. d. Opportunistic infections and cancer are leading causes of death. e. People with HIV-I disease are not infectious to others. f. The CD4+ T-cell is only affected when the disease has progressed to HIV-III

ANS: A, B, C, D In HIV, CD4+ cells begin to create new HIV particles. Antibodies the client produce are incomplete and do not function well. Macrophages also stop functioning properly. Opportunistic infections and cancer are the two leading causes of death in client's with HIV infection. People infected with HIV are infectious in all stages of the disease. The CD4+ T-cell is the immune system cell most affected by infection with the HIV virus.

A client who had a recent total knee arthroplasty will be using a continuous passive motion (CPM) machine after discharge at home. What health teaching about the CPM machine will the nurse include? (Select all that apply.) a. "Keep the machine padded well to prevent skin breakdown." b. "Ensure that your leg is placed properly on the machine." c. "Use the machine as prescribed but not at mealtime." d. "When the machine is not being used, do not store it on the floor." e. "Check that the cycle and range of motion is kept at the level prescribed."

ANS: A, B, C, D, E Although not used as often today, some clients are prescribed to use the CPM machine to increase range of motion in the surgical knee. All of these teaching points are important for any client who uses a CPM machine.

A nurse begins a job at a Veterans Administration Hospital and asks why so much emphasis is on HIV testing for the veterans. What reasons is this nurse given? (Select all that apply.) a. Veterans have a high prevalence of substance abuse. b. Many veterans may engage in high risk behaviors. c. Many older veterans may not know their risks. d. Everyone should know their HIV status. e. Belief that the VA has tested them and would notify them if positive.

ANS: A, B, C, D, E All options are correct for the veteran population. The nurse interacting with veteran would ensure they know about the HIV testing offered by the VA.

The nurse is teaching assistive personnel about postoperative care for an older adult who had a posterolateral total hip arthroplasty. What teaching will the nurse include? (Select all that apply.) a. "Move the client slowly to prevent dizziness and a possible fall." b. "Encourage the client to deep breathe and cough at least every 2 hours." c. "Help the client use the incentive spirometer at least every 2 hours." d. "Keep the abduction pillow in place at all times while the client is in bed." e. "Let me know if the client has an elevated temperature or pulse." f. "Keep in mind that the client may be a little confused after surgery." g. "Please let me know if you see any reddened or open skin areas during bathing."

ANS: A, B, C, D, E, F Older adults are at risk for complications of decreased mobility after surgery, including atelectasis, pneumonia, pressure injuries, and orthostatic hypotension. Therefore these precautions are to help keep the client safe and avoid complications that could be life threatening.

The nurse is assessing a client with acute pyelonephritis. What assessment findings would the nurse expect? (Select all that apply.) a. Fever b. Chills c. Tachycardia d. Tachypnea e. Flank or back pain f. Fatigue

ANS: A, B, C, D, E, F All of these assessment findings commonly occur in clients who have acute pyelonephritis because this health problem is a kidney infection.

The nurse is assessing a client with long-term rheumatoid arthritis (RA) who has been taking prednisone for 10 years. For which complications of chronic drug therapy would the nurse assess? (Select all that apply.) a. Osteoporosis b. Diabetes mellitus c. Glaucoma d. Hypertension e. Hypokalemia f. Decreased immunity

ANS: A, B, C, D, E, F Prednisone is a corticosteroid that is sometimes used for autoimmune disorders like RA when other drugs are not effective or cannot be tolerated. However, it can cause many complications when used long-term, including all of the health problems listed in the choices.

The nurse is caring for a client who is diagnosed with urinary tract infection (UTI). What common urinary signs and symptoms does the nurse expect? (Select all that apply.) a. Dysuria b. Frequency c. Burning d. Fever e. Chills f. Hematuria

ANS: A, B, C, F Fever and chills may occur in clients who have a UTI if the infection has expanded beyond the bladder into the kidneys. However, these symptoms are not urinary signs and symptoms.

A client has rheumatoid arthritis (RA) and the nurse is conducting a home assessment. What options can the nurse suggest for the client to maintain independence in activities of daily living (ADLs)? (Select all that apply.) a. Grab bars to reach high items b. Long-handled bath scrub brush c. Soft rocker-recliner chair d. Toothbrush with built-up handle e. Wheelchair cushion for comfort

ANS: A, B, D Grab bars, long-handled bath brushes, and toothbrushes with built-up handles all provide modifications for daily activities, making it easier for the client with RA to complete ADLs independently. The rocker-recliner and wheelchair cushion are comfort measures but do not help increase independence. Most clients who have RA are not wheelchair-bound.

A nurse assesses a client who has had two episodes of bacterial cystitis in the last 6 months. Which question(s) would the nurse ask? (Select all that apply.) a. "How much water do you drink every day?" b. "Do you take estrogen replacement therapy?" c. "Does anyone in your family have a history of cystitis?" d. "Are you on steroids or other immune-suppressing drugs?" e. "Do you drink grapefruit juice or orange juice daily?"

ANS: A, B, D Fluid intake, estrogen levels, and immune suppression all can increase the chance of recurrent cystitis. Family history is usually insignificant, and cranberry juice, not grapefruit or orange juice, has been found to increase the acidic pH and reduce the risk for bacterial cystitis.

A client is undergoing hemodialysis. The client's blood pressure at the beginning of the procedure was 136/88 mm Hg, and now it is 110/54 mm Hg. What actions would the nurse perform to maintain blood pressure? (Select all that apply.) a. Adjust the rate of extracorporeal blood flow. b. Place the patient in the Trendelenburg position. c. Stop the hemodialysis treatment. d. Administer a 250-mL bolus of normal saline. e. Contact the primary health care provider.

ANS: A, B, D Hypotension occurs often during hemodialysis treatments as a result of vasodilation from the warmed dialysate. Modest decreases in blood pressure, as is the case with this client, can be maintained with rate adjustment, Trendelenburg positioning, and a fluid bolus. If the blood pressure drops considerably after two boluses and cooling dialysate, the hemodialysis can be stopped and the primary health care provider contacted.

A nurse assesses a client with nephrotic syndrome. Which assessment findings would the nurse expect? (Select all that apply.) a. Proteinuria b. Hypoalbuminemia c. Dehydration d. Lipiduria e. Dysuria f. Costovertebral angle (CVA) tenderness

ANS: A, B, D Nephrotic syndrome is caused by glomerular damage and is characterized by proteinuria (protein level higher than 3.5 g/24 hr), hypoalbuminemia, edema, and lipiduria. Fluid overload leading to edema and hypertension is common with nephrotic syndrome; dehydration does not occur. Dysuria is present with cystitis. CVA tenderness is present with inflammatory changes in the kidney.

A nurse plans care for a client who is recovering from open reduction and internal fixation (ORIF) surgery for a right hip fracture. Which interventions would the nurse include in this client's plan of care? (Select all that apply.) a. Elevate heels off the bed with a pillow. b. Ambulate the client on the first postoperative day. c. Push the client's patient-controlled analgesia button. d. Re-position the client every 2 hours. e. Use pillows to encourage subluxation of the hip.

ANS: A, B, D Postoperative care for a client who has ORIF of the hip includes elevating the client's heels off the bed and re-positioning every 2 hours to prevent pressure and skin breakdown. It also includes ambulating the client on the first postoperative day, and using pillows or an abduction pillow to prevent subluxation of the hip. The nurse would teach the client to use the patient-controlled analgesia pump, but the nurse would never push the button for the client.

A nurse cares for clients with urinary incontinence. Which types of incontinence are correctly paired with their description? (Select all that apply.) a. Stress incontinence—urine loss with physical exertion b. Urge incontinence—loss of urine upon feeling the need to void c. Functional incontinence—urine loss results from abnormal detrusor contractions d. Overflow incontinence—constant dribbling of urine e. Reflex incontinence—leakage of urine without lower urinary tract disorder

ANS: A, B, D Stress incontinence is a loss of urine with physical exertion, coughing, sneezing, or exercising. Urge incontinence presents with an abrupt and strong urge to void and usually has a large amount of urine released with each occurrence. Overflow incontinence occurs with bladder distention and results in a constant dribbling of urine. Functional incontinence is the leakage of urine caused by factors other than a disorder of the lower urinary tract. Reflex incontinence results from abnormal detrusor contractions from a neurologic abnormality.

A nurse is visiting a client discharged home after a total hip arthroplasty. What safety precautions would the nurse recommend to the client and family? (Select all that apply.) a. Buy and install an elevated toilet seat. b. Install grab bars in the shower and by the toilet. c. Step into the bathtub with the affected leg first. d. Remove all throw rugs throughout the house. e. Use a shower chair while taking a shower.

ANS: A, B, D, E Buying and installing an elevated toilet seat, installing grab bars, removing throw rugs, and using a shower chair will all promote safety for this client. The client is still on partial weight bearing, so he or she cannot step into the bathtub leading with the operative side.

A nurse is assessing a community group for dietary factors that contribute to osteoporosis. In addition to inquiring about calcium, the nurse also assesses for which other dietary components? (Select all that apply.) a. Alcohol b. Caffeine c. Fat d. Carbonated beverages e. Vitamin D

ANS: A, B, D, E Dietary components that affect the development of osteoporosis include alcohol, caffeine, high phosphorus intake, carbonated beverages, and vitamin D. Tobacco is also a contributing lifestyle factor. Fat intake does not contribute to osteoporosis.

The nurse is educating a client with HIV-II and the partner on self-care measures to prevent infection when blood counts are low. What information does the nurse provide? (Select all that apply.) a. Do not work in the garden or with houseplants. b. Do not empty the kitty litter boxes. c. Clean your toothbrush in the dishwasher daily. d. Bathe daily using antimicrobial soap. e. Avoid people who are sick and large crowds. f. Make sure meat, fish, and eggs are cooked well.

ANS: A, B, D, E, F Ways to avoid infection when immunocompromised include not working in the garden or with houseplants; not emptying litter boxes; running the toothbrush through the dishwasher at least weekly; bathing daily using antimicrobial soap; avoiding sick people and large crowds; and making sure meat, fish, and eggs are cooked well prior to eating them.

A client asks the nurse about what medications may be included for nonopioid multimodal analgesia following a total knee arthroplasty. What medications may be given to the client? (Select all that apply.) a. Gabapentin b. Ketorolac c. Hydrocodone d. Ketamine e. Morphine f. Bupivacaine

ANS: A, B, D, F All of the choices are appropriate to use for nonopioid multimodal analgesia except for the two opioid drugs—hydrocodone and morphine. The nonopioid medications are used to decrease inflammation and pain.

Which findings are AIDS-defining characteristics? (Select all that apply.) a. CD4+ cell count less than 200/mm3 (0.2 × 109 /L) or less than 14% b. Infection with P. jiroveci c. Positive enzyme-linked immunosorbent assay (ELISA) test for human immune deficiency virus (HIV) d. Presence of HIV wasting syndrome e. Taking antiretroviral medications f. Confusion, dementia, or memory loss

ANS: A, B, D, F A diagnosis of AIDS requires that the person be HIV positive and have either a CD4+ T-cell count of less than 200 cells/mm3 (0.2 × 109 /L) or less than 14% (even if the total CD4+ count is above 200 cells/mm3 ) or an opportunistic infection such as P. jiroveci and HIV wasting syndrome. Confusion, dementia, and memory loss are central nervous system indications. Having a positive ELISA test and taking antiretroviral medications are not AIDS-defining characteristics.

A nurse teaches a client who is at risk for carpal tunnel syndrome. Which health promotion activities would the nurse include in the health teaching? (Select all that apply.) a. "Frequently assesses the ergonomics of the equipment being used." b. "Take breaks to stretch fingers and wrists during working hours." c. "Do not participate in activities that require repetitive actions." d. "Take ibuprofen to decrease pain and swelling in wrists." e. "Adjust chair height to allow for good posture."

ANS: A, B, E Health promotion activities to prevent carpal tunnel syndrome include assessing the ergonomics of the equipment being used, taking breaks to stretch fingers and wrists during working hours, and adjusting chair height to allow for good posture. The client should be allowed to participate in activities that require repetitive actions as long as precautions are taken to promote health. Pain medications are not part of health promotion activities.

A nurse teaches a client with a fractured tibia about external fixation. Which advantages of external fixation for the immobilization of fractures would the nurse share with the client? (Select all that apply.) a. It leads to minimal blood loss. b. It allows for early ambulation. c. It decreases the risk of infection. d. It increases blood supply to tissues. e. It promotes healing.

ANS: A, B, E External fixation is a system in which pins or wires are inserted through the skin and bone and then connected to a ridged external frame. With external fixation, blood loss is less than with internal fixation, but the risk for infection is much higher. The device allows early ambulation and exercise, maintains alignment, stabilizes the fracture site, and promotes healing. The device does not increase blood supply to the tissues. The nurse would assess for distal circulation, movement, and sensation, which can be disturbed by fracture injuries and treatments.

The nurse is teaching a client about medications for HIV-II treatment. What drugs are paired with the correct information? (Select all that apply.) a. Abacavir: avoid fatty and fried foods. b. Efavirenz: take 1 hour before or 2 hours after antacids. c. Atazanavir: check pulse daily and report pulse greater than 100 beats/min. d. Dolutegravir: do not take this medication if you become pregnant. e. Enfuvirtide: teach client how to operate syringe infusion pump for administration. f. All drugs: you must adhere to the drug schedule at least 90% of the time for effectiveness.

ANS: A, B, F Abacavir is a nucleoside reverse transcriptase inhibitor and clients are taught to avoid fried and fatty foods because they can lead to digestive upsets and even pancreatitis. Efavirenz is a nonnucleoside reverse transcriptase inhibitor and clients are taught to take them (doraverene) all except spaced 1 hour before or 2 hours after antacids to avoid inhibiting drug absorption. Atazanavir is a protease inhibitor and can cause bradycardia which should be reported. Dolutegravir is an integrase inhibitor and can cause birth defects. Enfuvirtide is a fusion inhibitor and is given subcutaneously. All drugs must be taken as scheduled 90% of the time in order to remain effective.

The nurse is caring for an older client who had a total knee arthroplasty. Prior to surgery, the client lived alone independently. With which interprofessional health care team members will the nurse collaborate to ensure positive client outcomes? (Select all that apply.) a. Case manager b. Mental health counselor c. Physical therapist d. Occupational therapist e. Speech-language pathologist f. Clergy/Spiritual leader

ANS: A, C The client was independent and living alone prior to surgery but will likely need help for a short time at home. However, if the client was ADL independent, he or she will not need referral to an occupational therapist. Therefore, a case manager can assess the living situation and identify any special needs to be addressed. The physical therapist will help the client learn to ambulate independently with a walker. There is no indication that the client needs referral for mental, spiritual, or speech-language services.

The nurse is assessing a client for chronic osteomyelitis. Which features distinguish this from the acute form of the disease? (Select all that apply.) a. Draining sinus tracts b. High fevers c. Presence of foot ulcers d. Swelling and redness e. Tenderness or pain

ANS: A, C Draining sinus tracts and foot ulcers are seen in chronic osteomyelitis. High fever, swelling, and redness are more often seen in acute osteomyelitis. Pain or tenderness can be in either case.

The nurse is reviewing the results of a client's urinalysis. The client has a diagnosis of acute glomerulonephritis. Which urine findings would the nurse expect? (Select all that apply.) a. Presence of protein b. Presence of red blood cells c. Presence of white blood cells d. Acidic urine e. Dilute urine

ANS: A, C, D The nurse would expect all of these findings except that the urine is usually concentrated with a high specific gravity.

A nurse is planning postoperative care for a client following a total hip arthroplasty. What nursing interventions would help prevent venous thromboembolism for this client? (Select all that apply.) a. Early ambulation b. Fluid restriction c. Quadriceps-setting exercises d. Compression stockings/devices e. Anticoagulant drug therapy

ANS: A, C, D, E Early ambulation, leg exercises, and compression stockings/devices promote venous return and peripheral circulation which helps prevent deep vein thrombi. Anticoagulants such as subcutaneous low-molecular-weight heparin (LMWH) or factor Xa inhibitors are used for all clients who have a total lower extremity joint arthroplasty. The nurse would encourage fluids to expand blood volume and promote circulation; fluids would not be restricted.

The nurse is caring for a client who recently sustained a sports injury to his right leg. What nursing interventions are appropriate for this client? (Select all that apply.) a. Immobilize the right leg. b. Apply heat immediately after the injury. c. Use compression to support the leg. d. Obtain an x-ray to detect possible fracture. e. Elevate the right leg to decrease swelling. f. Administer an opioid every 4 to 6 hours.

ANS: A, C, D, E The client who experiences a sports injury should be managed using the RICE treatment plan. Rest, ice, compression, and elevation are all appropriate. Heat would increase swelling and probably pain. An x-ray would be obtained to determine if one or more fractures are present. Opioids may not be needed depending on the nature of the injury.

A client asks the nurse why she has urinary incontinence. What risk factors would the nurse recall in preparing to respond to the client's question? (Select all that apply.) a. Diuretic therapy b. Anorexia nervosa c. Stroke d. Dementia e. Arthritis f. Parkinson disease

ANS: A, C, D, E, F Drugs, such as diuretics, cause frequent voiding, often in large amounts. Diseases or disorders that limit mobility, such as stroke, arthritis, and Parkinson disease, can prevent an individual from getting to the bathroom in a timely manner. Mental/behavioral problems, such as dementia, impair cognition and the ability to recognize when he or she needs to void.

A client is hospitalized in the oliguric phase of acute kidney injury (AKI) and is receiving tube feedings. The nurse is teaching the client's spouse about the renal-specific formulation for the enteral solution compared to standard formulas. What components would be discussed in the teaching plan? (Select all that apply.) a. Lower sodium b. Higher calcium c. Lower potassium d. Higher phosphorus e. Higher calories

ANS: A, C, E Many clients with AKI are too ill to meet caloric goals and require tube feedings with renal-specific formulas that are lower in sodium, potassium, and phosphorus, and higher in calories than are standard formulas.

A client with chronic osteomyelitis is being discharged from the hospital. What information is important for the nurse to teach this client and family? (Select all that apply.) a. Adherence to the antibiotic regimen b. Correct intramuscular injection technique c. Eating high-protein and high-carbohydrate foods d. Keeping daily follow-up appointments e. Proper use of the intravenous equipment

ANS: A, C, E The client going home with chronic osteomyelitis will need long-term antibiotic therapy—first intravenous, and then oral. The client needs education on how to properly administer IV antibiotics, care for the IV line, adhere to the regimen, and eat a healthy diet to encourage wound healing. The antibiotics are not given by IM injection. The client does not need daily follow-up.

A nurse is caring for a postoperative 70-kg client who had major blood loss during surgery. Which findings by the nurse would prompt immediate action to prevent acute kidney injury? (Select all that apply.) a. Urine output of 100 mL in 4 hours b. Urine output of 500 mL in 12 hours c. Large amount of sediment in the urine d. Amber, odorless urine e. Blood pressure of 90/60 mm Hg

ANS: A, C, E The low urine output, sediment, and blood pressure would be reported to the primary health care provider. Postoperatively, the nurse would measure intake and output, check the characteristics of the urine, and report sediment, hematuria, and urine output of less than 0.5 mL/kg/hr for 3 to 4 hours. A urine output of 100 mL is low, but a urine output of 500 mL in 12 hours would be within normal limits. Perfusion to the kidneys is compromised with low blood pressure. The amber odorless urine is normal.

After treating several young women for urinary tract infections (UTIs), the college nurse plans an educational offering on reducing the risk of getting a UTI. What information does the nurse include? (Select all that apply.) a. Void before and after each act of intercourse. b. Consider changing to spermicide from birth control pills. c. Do not douche or use scented feminine products. d. Wear loose-fitting nylon panties. e. Wipe or clean the perineum from front to back.

ANS: A, C, E Woman can reduce their risk of contracting UTIs by voiding before and after intercourse, not douching or using scented feminine products, and wiping from front to back. If spermicides are currently used, the woman should consider another form of birth control. Loose-fitting cotton underwear is best.

The nurse is planning health teaching for a client starting mirabegron for urinary incontinence. What health teaching would the nurse include? (Select all that apply.) a. "Monitor blood tests carefully if you are prescribed warfarin." b. "Avoid crowds and individuals with infection." c. "Report any fever to your primary health care provider." d. "Take your blood pressure frequently at home." e. "Report palpitations or chest soreness that may occur."

ANS: A, D This drug can cause increase blood pressure and, therefore, the client's blood pressure should be monitored. Mirabegron can increase the effect of warfarin causing bleeding or bruising. The client will need additional coagulation studies to ensure that the INR is within a therapeutic range.

A nurse teaches a client about prosthesis care after amputation. Which statements would the nurse include in the health teaching? (Select all that apply.) a. "The device has been custom made specifically for you." b. "Your prosthetic is good for work but not for exercising." c. "A prosthetist will clean your inserts for you each month." d. "Make sure that you wear the correct liners with your prosthetic." e. "I have scheduled a follow-up appointment for you."

ANS: A, D, E A client with a new prosthetic should be taught that the prosthetic device is custom made for the client, taking into account the level of amputation, lifestyle (including exercise preferences), and occupation. In collaboration with a prosthetist, the client should be taught proper techniques for cleansing the sockets and inserts, wearing the correct liners, and assessing shoe wear. Follow-up care and appointments are important for ongoing assessment.

A client with HIV-III is hospitalized with P. jiroveci pneumonia and is started on the drug of choice for this infection. What laboratory values would be most important for the nurse report to the primary health care provider? (Select all that apply.) a. Aspartate transaminase, alanine transaminase: elevated b. CD4+ cell count: 180/mm3 c. Creatinine: 1.0 mg/dL (88 mcmol/L) d. Platelet count: 80,000/mm3 (80 × 109 /L) e. Serum sodium: 120 mEq/L (120 mmol/L) f. Serum potassium: 3.4 mEq/L (3.4 mmol/L)

ANS: A, D, E The drug of choice to treat P. jiroveci pneumonia is trimethoprim with sulfamethoxazole. Side effects of this drug include hepatitis, hyponatremia, and thrombocytopenia. The elevated liver enzymes, low platelet count, and low sodium would all be reported. The CD4+ cell count is within the expected range for a client with an AIDS-defining infection. The creatinine level is normal and the potassium is just below normal.

A nurse teaches a client with polycystic kidney disease (PKD). Which statements would the nurse include in this client's discharge teaching? (Select all that apply.) a. "Take your blood pressure every morning." b. "Weigh yourself at the same time each day." c. "Adjust your diet to prevent diarrhea." d. "Contact your provider if you have visual disturbances." e. "Assess your urine for renal stones."

ANS: A,B,D A client who has PKD would measure and record his or her blood pressure and weight daily, limit salt intake, and adjust dietary selections to prevent constipation. The client should notify the primary health care provider if urine smells foul or has blood in it, as these are signs of a urinary tract infection or glomerular injury. The client should also notify the provider if visual disturbances are experienced, as this is a sign of a possible berry aneurysm, which is a complication of PKD. Diarrhea and renal stones are not manifestations or complications of PKD; therefore, teaching related to these concepts would be inappropriate.

A client is unsure of the decision to undergo peritoneal dialysis (PD) and wishes to discuss the advantages of this treatment with the nurse. Which statements by the nurse are correct regarding PD? (Select all that apply.) a. "You will not need vascular access to perform PD." b. "There is less restriction of protein and fluids." c. "You will have no risk for infection with PD." d. "You have flexible scheduling for the exchanges." e. "It takes less time than hemodialysis treatments."

ANS: A,B,D PD is based on exchanges of waste, fluid, and electrolytes in the peritoneal cavity. There is no need for vascular access. Protein is lost in the exchange, which allows for more protein and fluid in the diet. There is flexibility in the time for exchanges, but the treatment takes a longer period of time compared to hemodialysis. There still is risk for infection with PD, especially peritonitis.

A nurse assesses a group of clients who have rheumatoid arthritis (RA). Which client would the nurse see first? a. Client who reports jaw pain when eating b. Client with a red, hot, swollen right wrist c. Client who has a puffy-looking area behind the knee d. Client with a worse joint deformity since the last visit

ANS: B All of the options are possible manifestations of RA. However, the presence of one joint that is much redder, hotter, or more swollen that the other joints may indicate infection or an exacerbation of the RA disease process. The nurse needs to see this client first.

A client has a bone density score of -2.8. What intervention would the nurse anticipate based on this assessment? a. Asking the client to complete a food diary b. Planning to teach about bisphosphonates c. Scheduling another scan in 2 years d. Scheduling another scan in 6 months

ANS: B A T-score from a bone density scan at or lower than -2.5 indicates osteoporosis. The nurse would plan to teach about medications used to treat this disease, such as the bisphosphonates. A food diary is helpful to determine if the client gets adequate calcium and vitamin D, but at this point, dietary changes will not prevent the disease. Simply scheduling another scan will not help treat the disease either.

A client with HIV-II has had a sudden decline in status with a large increase in viral load. What action would the nurse take first? a. Ask the client about travel to any foreign countries. b. Assess the client for adherence to the drug regimen. c. Determine if the client has any new sexual partners. d. Request information about new living quarters or pets.

ANS: B Adherence to the complex drug regimen needed for HIV treatment can be daunting. Clients must take their medications on time and correctly at a minimum of 90% of the time to be effective. Since this client's viral load has increased dramatically, the nurse would first assess this factor. After this, the other assessments may or may not be needed.

The nurse assesses a client with diabetes and osteoarthritis (OA) during a checkup. The nurse notes the client's blood glucose readings have been elevated. What question by the nurse is most appropriate? a. "Are you following the prescribed diabetic diet?" b. "Have you been taking glucosamine supplements?" c. "How much exercise do you really get each week?" d. "You're still taking your diabetic medication, right?"

ANS: B All of the topics are appropriate for a client whose blood glucose readings have been higher than usual. However, since this client also has OA, and glucosamine can increase blood glucose levels, the nurse would ask about its use. The other questions all have an element of nontherapeutic communication in them. Asking how much exercise the client "really" gets is or if the diet is being followed is accusatory. Asking if the client takes his or her medications "right?" is patronizing.

A nurse reviews the laboratory findings of a client with a urinary tract infection (bacterial cystitis). The laboratory report notes a "shift to the left" in the client's white blood cell count. What action would the nurse take? a. Request that the laboratory perform a differential analysis on the white blood cells. b. Notify the primary health care provider and start an intravenous line for parenteral antibiotics. c. Ask assistive personnel (AP) to strain the client's urine for renal calculi. d. Assess the client for a potential allergic reaction and anaphylactic shock.

ANS: B An increase in band cells creates a "shift to the left." A left shift most commonly occurs with urosepsis and is seen rarely with uncomplicated urinary tract infections. The nurse will be administering antibiotics, most likely via IV, so he or she would notify the primary health care provider and prepare to give the antibiotics. The shift to the left is part of a differential white blood cell count. The nurse would not need to strain urine for stones. Allergic reactions are associated with elevated eosinophil cells, not band cells.

A client is having a peritoneal dialysis treatment. The nurse notes an opaque color to the effluent. What is the priority action by the nurse? a. Warm the dialysate solution in a microwave before instillation. b. Obtain a sample of the effluent and send to the laboratory. c. Flush the tubing with normal saline to maintain patency of the catheter. d. Check the peritoneal catheter for kinking and curling.

ANS: B An opaque or cloudy effluent is the first sign of peritonitis. A sample of the effluent would need to be sent to the laboratory for culture and sensitivity in order to administer the correct antibiotic. Warming the dialysate in a microwave and flushing the tubing are not safe actions by the nurse. Checking the catheter for obstruction is a viable option but will not treat the peritonitis.

A client is admitted to the emergency department with a fractured femur resulting from a motor vehicle crash. What the nurse's priority action? a. Keep the client warm and comfortable. b. Assess airway, breathing, and circulation. c. Maintain the client in a supine position. d. Immobilize the injured extremity with a splint.

ANS: B As part of the primary survey, the nurse would ensure that the client does not have any life-threatening problem by assessing the ABCs first. If there are not major problems, then the nurse could attend to the injured extremity.

A client who had a fractured ankle open reduction internal fixation (ORIF) 4 weeks ago reports burning pain and tingling in the affected foot. For which potential complication would the nurse anticipate? a. Delayed bone healing b. Complex regional pain syndrome c. Peripheral neuropathy d. Compartment syndrome

ANS: B Burning pain and tingling that occurs weeks or months after a fracture or other trauma may indicate complex regional pain syndrome. Compartment syndrome tends to occur within days of the initial injury.

After teaching a client with early polycystic kidney disease (PKD) about nutritional therapy, the nurse assesses the client's understanding. Which statement made by the client indicates a correct understanding of the teaching? a. "I will take a laxative every night before going to bed." b. "I must increase my intake of dietary fiber and fluids." c. "I shall only use salt when I am cooking my own food." d. "I'll eat white bread to minimize gastrointestinal gas."

ANS: B Clients with PKD often have constipation, which can be managed with increased fiber, exercise, and drinking plenty of water. Laxatives would be used cautiously. Clients with PKD would be on a restricted salt diet, which includes not cooking with salt. White bread has a low-fiber count and would not be included in a high-fiber diet.

A nurse plans care for a client who is recovering from a below-the-knee amputation of the left leg. Which intervention would the nurse include in this client's plan of care? a. Place pillows between the client's knees. b. Encourage range-of-motion exercises. c. Administer prophylactic antibiotics. d. Implement strict bedrest in a supine position.

ANS: B Clients with a below-the-knee amputation should complete range-of-motion exercises to prevent flexion contractions and prepare for a prosthesis. A pillow may be used under the limb as support. Clients recovering from this type of amputation are at low risk for infection and should not be prescribed prophylactic antibiotics. The client should be encouraged to re-position, move, and exercise frequently, and therefore should not be restricted to bedrest.

A client with HIV-III has been hospitalized with suspected cryptosporidiosis. What physical assessment would be most important with this condition? a. Auscultating the lungs b. Assessing mucous membranes c. Listening to bowel sounds d. Performing a neurologic examination

ANS: B Cryptosporidiosis can cause diarrhea and wasting with extreme loss of fluids and electrolytes. The nurse would assess signs of hydration/dehydration as the priority, including checking the client's mucous membranes for dryness. The nurse will perform the other assessments as part of a comprehensive assessment.

A client has been prescribed denosumab. What health teaching about this drug is most appropriate for the nurse to include? a. "Drink at least 8 ounces (240mL) of water with it." b. "Make appointments to come get your injection." c. "Sit upright for 30 to 60 minutes after taking it." d. "Take the drug on an empty stomach."

ANS: B Denosumab is given by subcutaneous injection twice a year. The client does not need to drink 8 ounces (240 mL) of water with this medication as it is not taken orally. The client does not need to remain upright for 30 to 60 minutes after taking this medication, nor does the client need to take the drug on an empty stomach.

A nurse is caring for a client with HIV-III who was admitted with HAND. What sign or symptom would be most important for the nurse to report to the primary health care provider? a. Nausea b. Change in pupil size c. Weeping open lesions d. Cough

ANS: B HIV-associated neurocognitive disorder (HAND) is a sign of neurologic involvement. The nurse would report any sign of increasing intracranial pressure immediately, including change in pupil size, level of consciousness, vital signs, or limb strength. The other signs and symptoms are not life threatening and would be documented and reported appropriately.

After delegating care to assistive personnel (AP) for a client who is prescribed habit training to manage incontinence, a nurse evaluates the AP's understanding. Which action indicates that the AP needs additional teaching? a. Toileting the client after breakfast b. Changing the client's incontinence brief when wet c. Encouraging the client to drink fluids d. Recording the client's incontinence episodes.

ANS: B Habit training is undermined by the use of absorbent incontinence briefs or pads. The nurse should reeducate the AP on the technique of habit training. The AP should continue to toilet the client after meals, encourage the client to drink fluids, and record incontinent episodes.

A client with rheumatoid arthritis (RA) has an acutely swollen, red, and painful joint. What nonpharmacologic intervention does the nurse recommend? a. Heating pad b. Ice packs c. Splint d. Paraffin dip

ANS: B Ice is best for acute inflammation. Heat often helps with joint stiffness. Splinting helps preserve joint function. A paraffin dip is used to provide warmth to the joint which is more appropriate for chronic pain and stiffness.

After teaching a client with nephrotic syndrome and a normal glomerular filtration, the nurse assesses the client's understanding. Which statement made by the client indicates a correct understanding of the diet therapy for this condition? a. "I must decrease my intake of fat." b. "I will increase my intake of protein." c. "A decreased intake of carbohydrates will be required." d. "An increased intake of vitamin C is necessary."

ANS: B In nephrotic syndrome, the renal loss of protein is significant, leading to hypoalbuminemia and edema formation. If glomerular filtration is normal or near normal, increased protein loss would be matched by increased intake of protein. The client would not need to adjust fat, carbohydrates, or vitamins based on this disorder.

The nurse is caring for four clients with chronic kidney disease (CKD). Which client would the nurse assess first upon initial rounding? a. Client with a blood pressure of 158/90 mm Hg b. Client with Kussmaul respirations c. Client with skin itching from head to toe d. Client with halitosis and stomatitis

ANS: B Kussmaul respirations indicate that the client has metabolic acidosis which is a complication of CKD. The client is increasing the rate and depth of breathing to excrete carbon dioxide through the lungs to lower serum pH. Hypertension is common in most patients with CKD, and skin itching increases with calcium-phosphate imbalances and elevations of nitrogenous wastes, another common finding in CKD. Uremia from CKD causes ammonia to be formed, resulting in the common findings of halitosis and stomatitis.

A nurse cares for a postmenopausal client who has had two episodes of bacterial urethritis in the last 6 months. The client asks, "I never have urinary tract infections. Why is this happening now?" How would the nurse respond? a. "Your immune system becomes less effective as you age." b. "Low estrogen levels can make the tissue more susceptible to infection." c. "You should be more careful with your personal hygiene in this area." d. "It is likely that you have an untreated sexually transmitted disease."

ANS: B Low estrogen levels decrease moisture and secretions in the perineal area and cause other tissue changes, predisposing it to the development of infection. Urethritis is most common in postmenopausal women for this reason. Although immune function does decrease with aging and sexually transmitted diseases are a known cause of urethritis, the most likely reason in this patient is low estrogen levels. Personal hygiene usually does not contribute to this disease process.

A client asks the nurse about having a total knee arthroplasty to relieve joint pain. Which factor would place the client at the highest risk for impaired postoperative healing? a. Controlled hypertension b. Obesity c. Osteoarthritis d. Mild osteopenia

ANS: B Obesity places a client at high risk for many postoperative complications including slower wound and bone healing. The other factors usually do not affect healing after surgery.

A nurse assesses a client with polycystic kidney disease (PKD). Which assessment finding would alert the nurse to immediately contact the primary health care provider? a. Flank pain b. Periorbital edema c. Bloody and cloudy urine d. Enlarged abdomen

ANS: B Periorbital edema would not be a finding related to PKD and would be investigated further. Flank pain and a distended or enlarged abdomen occur in PKD because the kidneys enlarge and displace other organs. Urine can be bloody or cloudy as a result of cyst rupture or infection.

The nurse is assessing a client with a diagnosis of prerenal acute kidney injury (AKI). Which condition would the nurse expect to find in the patient's recent history? a. Pyelonephritis b. Dehydration c. Bladder cancer d. Kidney stones

ANS: B Prerenal causes of AKI are related to a decrease in perfusion, such as in clients who have prolonged dehydration. Pyelonephritis is an intrinsic or intrarenal cause of AKI related to kidney damage. Bladder cancer and kidney stones are postrenal causes of AKI related to urine flow obstruction.

The nurse assesses a client with a history of urinary incontinence who presents with extreme dry mouth, constipation, and an inability to void. Which question would the nurse ask first? a. "Are you drinking plenty of water?" b. "What medications are you taking?" c. "Have you tried laxatives or enemas?" d. "Has this type of thing ever happened before?"

ANS: B Some types of incontinence or other health problems are treated with anticholinergic medications. Anticholinergic side effects include dry mouth, constipation, and urinary retention. The nurse needs to assess the client's medication list to determine whether the he or she is taking an anticholinergic medication. The other questions are not as helpful to understanding the current situation.

After teaching a client with hypertension secondary to renal disease, the nurse assesses the client's understanding. Which statement made by the client indicates a need for additional teaching? a. "I can prevent more damage to my kidneys by managing my blood pressure." b. "If I have increased urination at night, I need to drink less fluid during the day." c. "I need to see the registered dietitian to discuss limiting my protein intake." d. "It is important that I take my antihypertensive medications as directed."

ANS: B The client should not restrict fluids during the day due to increased urination at night. Clients with renal disease may be prescribed fluid restrictions, and would be assessed thoroughly for potential dehydration. Increased nocturnal voiding can be decreased by consuming fluids earlier in the day. Blood pressure control is needed to slow the progression of renal dysfunction. When dietary protein is restricted, refer the client to the registered dietitian nutritionist as needed.

A nurse cares for a client with a recently fractured tibia. Which assessment would alert the nurse to take immediate action? a. Pain of 4 on a scale of 0-10 b. Numbness in the extremity c. Swollen extremity at the injury site d. Feeling cold while lying in bed

ANS: B The client with numbness and/or tingling of the extremity may be displaying the first signs of acute compartment syndrome. This is an acute problem that requires immediate intervention because of possible decreased circulation. Moderate pain and swelling is an expected assessment after a fracture. These findings can be treated with comfort measures. Being cold can be treated with additional blankets or by increasing the temperature of the room.

A client who had a traumatic above-the-knee amputation states that he fears he will never have an intimate relationship again. What is the nurse's best response? a. "You'll be able to get a leg prosthesis soon." b. "You think you won't be able to have sex again?" c. "I will ask the social worker to talk with you." d. :Are you married now or have a girlfriend?"

ANS: B The nurse's response needs to allow further exploration of the client's feelings. Referring the client to another health professional might be appropriate at a later time but discounts the client's current feelings. Asking about marriage or a girlfriend assumes that the client is heterosexual.

A nurse assesses a client who is recovering from extracorporeal shock-wave lithotripsy for renal calculi. The nurse notes an ecchymotic area on the client's right lower back. What action would the nurse take? a. Administer fresh-frozen plasma. b. Apply an ice pack to the site. c. Place the client in the prone position. d. Obtain serum coagulation test results.

ANS: B The shock waves from lithotripsy can cause bleeding into the tissues through which the waves pass. Application of ice can reduce the extent and discomfort of the bruising. Although coagulation test results and fresh-frozen plasma are used to assess and treat bleeding disorders, ecchymosis after this procedure is not unusual and does not warrant a higher level of intervention. Changing the client's position will not decrease bleeding.

A client with osteoporosis is going home where the client lives alone. What action by the nurse is best? a. Refer the client to Meals on Wheels. b. Arrange a home safety evaluation. c. Ensure that the client has a walker at home. d. Help the client look into assisted living.

ANS: B This client has several risk factors that place him or her at a high risk for falling. The nurse should consult social work or home health care to conduct a home safety evaluation. The other options may or may not be needed based upon the client's condition at discharge.

A client with HIV-III is admitted to the hospital with Toxoplasma gondii infection. Which action by the nurse is most appropriate? a. Initiate Contact Precautions. b. Conduct frequent neurologic assessments. c. Conduct frequent respiratory assessments. d. Initiate Protective Precautions.

ANS: B Toxoplasma gondii infection is an opportunistic infection that causes an encephalitis but poses only a rare threat to immunocompetent individuals The nurse would perform ongoing neurologic assessments. Contact and Protective Precautions are not needed. Good respiratory assessments are important to the client, but toxoplasmosis will demonstrate neurologic signs and symptoms.

A nurse is giving discharge instructions to a client recently diagnosed with chronic kidney disease (CKD). Which statements made by the client indicate a correct understanding of the teaching? (Select all that apply.) a. "I can continue to take antacids to relieve heartburn." b. "I need to ask for an antibiotic when scheduling a dental appointment." c. "I'll need to check my blood sugar often to prevent hypoglycemia." d. "The dose of my pain medication may have to be adjusted." e. "I should watch for bleeding when taking my anticoagulants."

ANS: B, C, D, E In discharge teaching, the nurse must emphasize that the client needs to have an antibiotic prophylactically before dental procedures to prevent infection. There may be a need for dose reduction in medications if the kidney is not excreting them properly (antacids with magnesium, antibiotics, antidiabetic drugs, insulin, opioids, and anticoagulants).

A client with HIV-III is in the hospital with severe diarrhea. What actions does the nurse delegate to assistive personnel (AP)? (Select all that apply.) a. Assessing the client's fluid and electrolyte status b. Assisting the client to get out of bed to prevent falls c. Obtaining a bedside commode if the client is weak d. Providing gentle perianal cleansing after stools e. Reporting any perianal abnormalities

ANS: B, C, D, E The AP can assist the client with getting out of bed, obtain a bedside commode for the client's use, cleanse the client's perianal area after bowel movements, and report any abnormal observations such as redness or open areas. The nurse assesses fluid and electrolyte status.

A nurse is providing education about HIV risks at a health fair. What groups would the nurse include as needing to be tested for HIV on an annual basis? (Select all that apply.) a. Anyone who received a blood product in 1989 b. Couples planning on getting married c. Those who are sexually active with multiple partners d. Injection drugs users e. Sex workers and their customers f. Adults over the age of 65 years

ANS: B, C, D, E The CDC recommends that HIV testing would be performed on those who received a transfusion between 1978 and 1985 only. People planning on getting married should be tested and all sexually active people should know their HIV status. Those engaged in sex work and their customers should also be tested, as well as injection drug users. Those over the age of 65 years need a one-time screen.

The nurse is caring for a client with a new diagnosis of chronic kidney disease. Which priority complications would the nurse anticipate? (Select all that apply.) a. Dehydration b. Anemia c. Hypertension d. Dysrhythmias e. Heart failure

ANS: B, C, D, E The client who has CKD has fluid overload and electrolyte imbalances, especially hyperkalemia, that can cause hypertension, heart failure, and dysrhythmias. Anemia results because erythropoietin production by the kidneys is decreased.

The nurse is caring for a client who just had a kyphoplasty. What nursing care is needed for the client at this time? (Select all that apply.) a. Place the client in a prone position to prevent pressure on the surgical area. b. Apply an ice pack to the surgical area to help relieve pain. c. Assess the client's pain level to compare it with pain before the procedure. d. Take vital signs, including oxygen saturation, frequently. e. Monitor for complications such as bleeding or shortness of breath. f. Perform frequent neurologic assessments and report major changes.

ANS: B, C, D, E, F All of the choices are correct except that the client should stay in a flat supine position immediately after the procedure.

The nurse is teaching a client with mild rheumatoid arthritis (RA) about how to protect synovial joints. Which health teaching will the nurse include? (Select all that apply.) a. "Use small joints rather than larger ones during tasks." b. "Use both hands instead of one with holding objects." c. "When getting out of bed or a chair, use the palms of your hands." d. "Bend your knees instead of your waist and keep your back straight." e. "Do not use multiple pillows under your head to prevent neck flexion." f. "Use a device or rubber grip to open jars or bottle tops." g. "Use long-handled devices such as a hairbrush with an extended handle."

ANS: B, C, D, E, F, G All of these options are part of health teaching for joint protection except that large joints should be used instead of smaller ones.

A nurse assesses a client who has a family history of polycystic kidney disease (PKD). Which assessment findings would the nurse expect? (Select all that apply.) a. Nocturia b. Flank pain c. Increased abdominal girth d. Dysuria e. Hematuria f. Diarrhea

ANS: B, C, E Clients with PKD experience abdominal distention that manifests as flank pain and increased abdominal girth. Bloody urine is also present with tissue damage secondary to PKD. Clients with PKD often experience constipation, but would not report nocturia or dysuria.

The nurse assesses a client with long-term rheumatoid arthritis (RA) for late signs and symptoms. Which assessment findings will the nurse document as late signs and symptoms of RA? (Select all that apply.) a. Anorexia b. Felty syndrome c. Joint deformity d. Low-grade fever e. Weight loss

ANS: B, C, E Late signs and symptoms of RA include Felty syndrome, joint deformity, weight loss, organ involvement, osteoporosis, extreme fatigue, and anemia, among others. Anorexia and low-grade fever are both seen early in the course of the disease.

The nurse assesses a client who is admitted with a pelvic fracture. Which assessments would the nurse monitor to prevent or detect a complication of this injury? (Select all that apply.) a. Temperature b. Urinary output c. Blood pressure d. Pupil reaction e. Skin color

ANS: B, C, E With a pelvic fracture, internal organ damage may result in bleeding and hypovolemic shock. The nurse monitors the client's heart rate, blood pressure, urine output, skin color, and level of consciousness frequently to detect assess for shock. It is important to monitor the urine for blood to assess whether the urinary system has been damaged with the pelvic fracture. Changes in temperature and pupil reactions are not directly associated with hypovolemic shock. Temperature changes are usually associated with hypo- or hyperthermia or infectious processes. Pupillary changes occur with brain injuries, bleeds, or strokes.

5. A client with HIV-III has oral thrush and difficulty eating. What actions does the nurse delegate to the assistive personnel (AP)? (Select all that apply.) a. Apply oral anesthetic gels before meals. b. Assist the client with oral care every 2 hours. c. Offer the client frequent sips of cool drinks. d. Provide the client with alcohol-based mouthwash. e. Remind the client to use only a soft toothbrush. f. Offer the client soft foods like gelatin or pudding.

ANS: B, C, E, F The AP can help the client with oral care, offer fluids, and remind the client of things the nurse (or other professional) has already taught. Soft foods and liquids are tolerated better than harder foods. Applying medications is performed by the nurse. Alcohol-based mouthwashes are harsh and drying and would not be used.

A nurse assesses a client who is recovering from a nephrostomy. Which assessment findings would alert the nurse to urgently contact the primary health care provider? (Select all that apply.) a. Clear drainage b. Bloody drainage at site c. Patient reports headache d. Foul-smelling drainage e. Urine draining from site

ANS: B, D, E After a nephrostomy, the nurse would assess the client for complications and urgently notify the primary health care provider if drainage decreases or stops, drainage is cloudy or foul smelling, the nephrostomy site leaks blood or urine, or the client has back pain. Clear drainage is normal. A headache would be an unrelated finding.

The primary health care provider prescribes methotrexate (MTX) for a client with a new diagnosis of rheumatoid arthritis. The nurse provides health teaching about the drug. What statement by the nurse is appropriate to include about methotrexate? a. "It will take at least 1 to 2 weeks for the drug to help relieve your symptoms." b. "The drug is very expensive but there are pharmacy plans to help pay for it." c. "The drug can increase your risk for infection, so you should avoid crowds." d. "It's OK for you to drink about 2 to 3 glasses of wine each week while taking the drug."

ANS: C MTX takes up to 4 to 6 weeks to begin to help relieve RA symptoms and is very inexpensive. Clients should avoid alcohol due to the potential for liver toxicity. MTX suppresses the immune system which makes clients susceptible to infection. The nurse teaches clients to avoid crowds and anyone with a known infection.

A client has long-term rheumatoid arthritis that especially affects the hands. The client wants to finish quilting a baby blanket before the birth of her grandchild. What response by the nurse is appropriate? a. "Let's ask your provider about increasing your pain pills." b. "Hold ice bags against your hands before quilting." c. "Try a paraffin wax dip 20 minutes before you quilt." d. "You need to stop quilting before it destroys your fingers."

ANS: C Paraffin wax dips are beneficial for decreasing pain in arthritic hands and lead to increased mobility. Increasing pain pills may not help with movement. Ice has limited use unless the client has a "hot" or exacerbated joint. The client wants to finish the project, so the nurse would not negate its importance by telling the client it is destroying her joints.

The nurse is teaching a client who is planning to have a total hip arthroplasty. What statement by the client indicates a need for further teaching? a. "I will get an IV antibiotic right before surgery to prevent infection." b. "I may request a regional nerve block as part of the surgical anesthesia." c. "I will receive IV heparin before surgery to decrease the risk of clots." d. "I will receive tranexamic acid to help reduce blood loss during surgery."

ANS: C All of the choices are correct except that IV heparin is not given before or after surgery. A different anticoagulant is given after surgery to prevent postoperative venous thromboembolism, such as deep vein thrombosis and pulmonary embolus.

A client is prescribed celecoxib for joint pain. What statement by the client indicates a need for further teaching? a. "I'll report any signs of bleeding or bruising to my primary health care provider." b. "I'll take this drug only as prescribed by my primary health care provider." c. "I'll be sure to take this drug three times a day only on an empty stomach." d. "I'll monitor the amount of urine that I excrete every day and report any changes."

ANS: C All of the choices are correct for this NSAID except that celecoxib can cause GI distress unless taken with meals or food. The drug should not be taken on an empty stomach and is rarely taken more than twice a day.

A client has a left knee arthrocentesis to remove excess joint fluid. What postprocedure health teaching will the nurse include? a. "Take your opioid medication as prescribed by the primary health care provider." b. "Do not bear weight on your left leg for at least a week after you get home." c. "Monitor the site for bleeding or clear fluid leakage when you are home." d. "Tell your employer that you can't come back to work for 2 to 3 weeks."

ANS: C An arthrocentesis is performed as an ambulatory procedure and may require a mild analgesic such as acetaminophen for discomfort. Opioids are not used. The client may bear weight and return to work, but needs to monitor for bleeding or leakage of synovial fluid at the injection site.

A client with pneumonia and dementia is admitted with an indwelling urinary catheter in place. During interprofessional rounds the following day, which question would the nurse ask the primary health care provider? a. "Do you want daily weights on this client?" b. "Will the client be able to return home?" c. "May we discontinue the indwelling catheter?" d. "Should we get another chest x-ray today?"

ANS: C An indwelling urinary catheter dramatically increases the risks of urinary tract infection and urosepsis. Nursing staff should ensure that catheters are left in place only as long as they are medically needed. The nurse would inquire about removing the catheter. All other questions might be appropriate, but because of client safety, this question takes priority.

The nurse teaches assistive personnel (AP) about care of an older adult diagnosed with osteoporosis. What teaching would the nurse include? a. "Teach the client to eat high-calcium foods in the diet." b. "Assist the client with activities of daily living." c. "Osteoporosis places the client is at risk for fractures." d. "The client should stay in bed to prevent falling."

ANS: C Anyone who has osteoporosis is at risk for fragility fractures even if he or she does not experience trauma like a fall. The client needs to keep active rather than stay in bed where more bone could be lost. High-calcium foods may not be helpful because bone loss is already severe. There is no indication that the client needs assistance with ADLs

A nurse cares for a client who has pyelonephritis. The client states, "I am embarrassed to talk about my symptoms." How would the nurse respond? a. "I am a professional. Your symptoms will be kept in confidence." b. "I understand. Elimination is a private topic and shouldn't be discussed." c. "Take your time. It is okay to use words that are familiar to you." d. "You seem anxious. Would you like a nurse of the same gender to care for you?"

ANS: C Clients may be uncomfortable discussing issues related to elimination and the genitourinary area. The nurse would encourage the client to use language that is familiar to the client. The nurse must assess the client and cannot take the time to stop the discussion or find another nurse to complete the assessment.

A nurse cares for a middle-age female client with diabetes mellitus who is being treated for the third episode of acute pyelonephritis in the past year. The client asks, "What can I do to help prevent these infections?" How would the nurse respond? a. "Test your urine daily for the presence of ketone bodies and proteins." b. "Use tampons rather than sanitary napkins during your menstrual period." c. "Drink more water and empty your bladder more frequently during the day." d. "Keep your hemoglobin A1C under 9% by keeping your blood sugar controlled."

ANS: C Clients with long-standing diabetes mellitus are at risk for pyelonephritis for many reasons. Chronically elevated blood glucose levels spill glucose into the urine, changing the pH, and providing a favorable climate for bacterial growth. The neuropathy associated with diabetes reduces bladder tone and reduces the client's sensation of bladder fullness. Thus, even with large amounts of urine, the client voids less frequently, allowing stasis and overgrowth of microorganisms. Increasing fluid intake (specifically water) and voiding frequently prevent stasis and bacterial overgrowth. Testing urine and using tampons will not help prevent pyelonephritis. A hemoglobin A1C of 9% is too high.

A client with diabetes mellitus type 2 has been well controlled with metformin. The client is scheduled for magnetic resonance imaging (MRI) scan with contrast. What priority would the nurse take at this time? a. Teach the client about the purpose of the MRI. b. Assess the client's blood urea nitrogen and creatinine. c. Tell the client to withhold metformin for 24 hours before the MRI. d. Ask the client if he or she is taking antibiotics.

ANS: C Contrast media can be nephrotoxic (damaging to the kidneys). Metformin can also be nephrotoxic and the client should not be exposed to two agents. Clients who have diabetes are already at risk for renal damage.

A client is started on continuous venovenous hemofiltration (CVVH). Which finding would require immediate action by the nurse? a. Potassium level of 5.5 mEq/L (5.5 mmol/L) b. Sodium level of 138 mEq/L (138 mmol/L) c. Blood pressure of 76/58 mm Hg d. Pulse rate of 88 beats/min

ANS: C Hypotension can be a problem with CVVH if replacement fluid does not provide enough volume to maintain blood pressure. The nurse needs to monitor for ongoing fluid and electrolyte replacement. The sodium level is normal and the potassium level is slightly elevated, which could be normal findings for someone with acute kidney injury. A pulse rate of 88 beats/min is within usual limits.

An older client with diabetes is admitted with a heavily draining leg wound. The client's white blood cell count is 38,000/mm3 (38 × 109/L) but the client is afebrile. Which nursing action is most appropriate at this time? a. Administer acetaminophen as needed. b. Educate the client on amputation. c. Place the client on Contact Precautions. d. Refer the client to the wound care nurse.

ANS: C In the presence of a heavily draining wound, the nurse would place the client on Contact Precautions. If the client has discomfort, acetaminophen can be used, but this client has not reported pain and is afebrile. The client may or may not need an amputation in the future. The wound care nurse may be consulted but not as the most appropriate action.

The nurse is teaching a client how to increase the flow of dialysate into the peritoneal cavity during dialysis. Which statement by the client demonstrates a correct understanding of the teaching? a. "I should leave the drainage bag above the level of my abdomen." b. "I could flush the tubing with normal saline if the flow stops." c. "I should take a stool softener every morning to avoid constipation." d. "My diet should have low fiber in it to prevent any irritation."

ANS: C Inflow and outflow problems of the dialysate are best controlled by preventing constipation. A daily stool softener is the best option for the client. The drainage bag should be below the level of the abdomen. Flushing the tubing will not help with the flow. A diet high in fiber will also help with a constipation problem.

A client with HIV-III asks the nurse why gabapentin is part of the drug regimen when the client does not have a history of seizures. What response by the nurse is best? a. "Gabapentin can be used as an antidepressant too." b. "I have no idea why you would be taking this drug." c. "This drug helps treat the pain from nerve irritation." d. "You are at risk for seizures due to fungal infections."

ANS: C Many classes of medications are used for neuropathic pain, including tricyclic antidepressants and anticonvulsants such as gabapentin. It is not being used to prevent seizures from fungal infections. If the nurse does not know the answer, he or she would find out for the client.

After teaching a client with bacterial cystitis who is prescribed phenazopyridine, the nurse assesses the client's understanding. Which statement made by the client indicates a correct understanding of the teaching? a. "I will not take this drug with food or milk." b. "I will have my partners tested for STIs." c. "An orange color in my urine should not alarm me." d. "I will drink two glasses of cranberry juice daily."

ANS: C Phenazopyridine discolors urine, most commonly to a deep reddish orange. Many clients think that they have blood in their urine when they see this. In addition, the urine can permanently stain clothing. There are no dietary restrictions or needs while taking this medication.

A client with known HIV-II is admitted to the hospital with fever, night sweats, and severe cough. Laboratory results include a CD4+ cell count of 180/mm3 and a negative tuberculosis (TB) skin test 4 days ago. What action would the nurse take first? a. Initiate Droplet Precautions for the client. b. Notify the primary health care provider about the CD4+ results. c. Place the client under Airborne Precautions. d. Use Standard Precautions to provide care.

ANS: C Since this client's CD4+ cell count is so low, he or she may have energy, or the inability to mount an immune response to the TB test. The client also appears to have progressed to HIV-III. The nurse would first place the client on Airborne Precautions to prevent the spread of TB if it is present. Next the nurse notifies the primary health care provider about the low CD4+ count and requests alterative testing for TB. Droplet Precautions are not used for TB. Standard Precautions are not adequate in this case.

A client is placed on fluid restriction because of chronic kidney disease (CKD). Which assessment finding would alert the nurse that the client's fluid balance is stable at this time? a. Decreased calcium levels b. Increased phosphorus levels c. No adventitious sounds in the lungs d. Increased edema in the legs

ANS: C The absence of adventitious sounds upon auscultation of the lungs indicates a lack of fluid overload and fluid balance in the client's body. Decreased calcium levels and increased phosphorus levels are common findings with CKD. Edema would indicate a fluid imbalance.

The nurse assesses a client who has possible bladder cancer. What common assessment finding associated with this type of cancer would the nurse expect? a. Urinary retention b. Urinary incontinence c. Painless hematuria d. Difficulty urinating

ANS: C The classic and most common finding in clients who have bladder cancer is painless and intermittent hematuria that can be with gross or microscopic. Dysuria, frequency, and urgency occur in clients who have bladder infection or obstruction.

A nurse is caring for an older client who is recovering from a leg amputation surgery. The client states, "I don't want to live with only one leg. I should have died during the surgery." What is the nurse's best response? a. "Your vital signs are good, and you are doing just fine right now." b. "Your children are waiting outside. Do you want them to grow up without a father?" c. "This is a big change for you. What support system do you have to help you cope?" d. "You will be able to do some of the same things as before you became disabled."

ANS: C The client feels like less of a person following the amputation. The nurse would help the client to identify coping mechanisms that have worked in the past and current support systems to assist with coping. The nurse would not ignore the client's feelings by focusing on vital signs. The nurse would not try to make the client feel guilty by alluding to family members. The nurse would not refer to the patient as being "disabled" as this labels the client and may fuel poor body image.

A nurse is caring for a client recovering from an above-the-knee amputation of the right leg. The client reports pain in the right foot. Which prescribed medication would the nurse most likely administer? a. Intravenous morphine b. Oral acetaminophen c. Intravenous calcitonin d. Oral ibuprofen

ANS: C The client is experiencing phantom limb pain, which usually manifests as intense burning, crushing, or cramping. IV infusions of calcitonin during the week after amputation can reduce phantom limb pain. Opioid analgesics such as morphine are not as effective for phantom limb pain as they are for residual limb pain. Oral acetaminophen and ibuprofen are not used in treating phantom limb pain.

A nurse reviews the laboratory values of a client who returned from kidney transplantation 12 hours ago: Sodium 136 mEq/L (135 mmol/L) Potassium 5 mEq/L (5 mmol/L) Blood urea nitrogen (BUN) 44 mg/dL (15.7 mmol/L) Serum creatinine 2.5 mg/dL (221 mcmol/L) What initial intervention would the nurse anticipate? a. Start hemodialysis immediately. b. Discuss the need for peritoneal dialysis. c. Increase the dose of immunosuppression. d. Return the client to surgery for exploration.

ANS: C The client may need a higher dose of immunosuppressive medication as evidenced by the elevated BUN and serum creatinine levels. This increased dose may reverse the possible acute rejection of the transplanted kidney. The client does not need hemodialysis, peritoneal dialysis, or further surgery at this point.

A nurse assesses clients in an osteoporosis clinic. Which client would the nurse assess first? a. Client taking calcium with vitamin D who reports flank pain 2 weeks ago. b. Client taking ibandronate who cannot remember when the last dose was. c. Client taking raloxifene who reports unilateral calf swelling. d. Client taking risedronate who reports occasional dyspepsia.

ANS: C The client on raloxifene needs to be assessed first because of the potential for deep vein thrombosis, which is an adverse effect of raloxifene. The client with flank pain may have had a kidney stone but is not acutely ill now. The client who cannot remember taking the last dose of ibandronate can be seen last. The client on risedronate may need to change medications.

A nurse teaches a client who is starting urinary bladder training. Which statement would the nurse include in this client's teaching? a. "Use the toilet when you first feel the urge, rather than at specific intervals." b. "Initially try to use the toilet at least every half hour for the first 24 hours." c. "Try to consciously hold your urine until the scheduled toileting time." d. "The toileting interval can be increased once you have been continent for a week."

ANS: C The client should try to hold urine consciously until the next scheduled toileting time. Toileting should occur at specific intervals during the training. The interval can be increased once the client becomes comfortable with the interval.

The nurse is caring for a client who has chronic pyelonephritis. What assessment finding would the nurse expect? a. Fever b. Flank pain c. Hypertension d. Nausea and vomiting

ANS: C The client who has chronic pyelonephritis has renal damage and therefore has hypertension. The other assessment findings commonly occur in clients with acute pyelonephritis.

A client had a bunionectomy with osteotomy. The client asks why healing may take up to 3 months. What explanation by the nurse is best? a. "The bones in your feet are hard to operate on." b. "The surrounding bones and tissue are damaged." c. "Your feet have less blood flow, so healing is slower." d. "Your feet bear weight so they never really heal."

ANS: C The feet are the most distal to the heart and receive less blood flow than other organs and tissues, prolonging the healing time after surgery. The other explanations are not correct.

The nurse is teaching assistive personnel (AP) about fluid restriction for a client who has acute kidney injury (AKI). The client's 24-hour urinary output is 120 mL. How much fluid would the client be allowed to have over the next 24 hours? a. 380 mL b. 500 mL c. 620 mL d. 750 mL

ANS: C The general principle for fluid restriction for clients is that they may have a daily fluid intake of 500 mL plus the amount of their urinary output. In this case, 120 mL urinary output plus 500 mL equals 620 mL fluid allowance.

The nurse is performing an assessment of a client with possible plantar fasciitis in the right foot. What assessment finding would the nurse expect in the right foot? a. Multiple toe deformities b. Numbness and paresthesia c. Severe pain in the arch of the foot d. Redness and severe swelling

ANS: C The most common assessment finding is the client's report of severe pain in the arch of the foot, especially when walking. The other findings are not typical in clients with this health problem.

The nurse is caring for a client with urinary incontinence. The client states, "I am so embarrassed. My bladder leaks like a young child's bladder." How would the nurse respond? a. "I understand how you feel. I would be mortified." b. "Incontinence pads will minimize leaks in public." c. "I can teach you strategies to help control your incontinence." d. "More people experience incontinence than you might think."

ANS: C The nurse would accept and acknowledge the client's concerns, and assist the client to learn techniques that will allow control of urinary incontinence. The nurse would not diminish the client's concerns with the use of pads or stating statistics about the occurrence of incontinence.

The charge nurse is orienting a new nurse about care for an assigned client with an arteriovenous (AV) fistula for hemodialysis in her left arm. Which action by the float nurse would be considered unsafe? a. Palpating the access site for a bruit or thrill b. Using the right arm for a blood pressure reading c. Administering intravenous fluids through the AV fistula d. Checking distal pulses in the left arm

ANS: C The nurse would not use the arm with the AV fistula for intravenous infusion, blood pressure readings, or venipuncture. Compression and infection can result in the loss of the AV fistula. The AV fistula would be monitored by auscultating or palpating the access site. Checking the distal pulse would be an appropriate assessment.

The nurse is caring for a client who has severe osteoarthritis. What primary joint problems will the nurse expect the client to report? a. Crepitus b. Effusions c. Pain d. Deformities

ANS: C The primary assessment finding typically reported by clients who have osteoarthritis is joint pain, although crepitus, effusions (fluid), and mild deformities may occur.

The nurse is caring for a client who had a closed reduction of the left arm and notes a large wet area of drainage on the cast. What action is the most important? a. Cut off the old cast. b. Document the assessment. c. Notify the primary health care provider. d. Wrap the cast with gauze.

ANS: C The primary health care provider should be notified to examine the client and determine the source of the drainage. The nurse's assessment should be documented, but that is not the most important action.

A nurse is caring for four clients. After the hand-off report, which client would the nurse see first? a. Client with osteoporosis and a white blood cell count of 27,000/mm3 (27 × 109/L) b. Client with osteoporosis and a bone fracture who requests pain medication c. Post-microvascular bone transfer client whose distal leg is cool and pale d. Client with suspected bone tumor who just returned from having a spinal CT

ANS: C This client is the priority because the assessment findings indicate a critical lack of perfusion. A high white blood cell count is an expected finding for the client with osteoporosis. The client requesting pain medication should be seen second. The client who just returned from a CT scan is stable and needs no specific postprocedure care.

A client is admitted with acute kidney injury (AKI) and a urine output of 2000 mL/day. What is the major concern of the nurse regarding this patient's care? a. Edema and pain b. Cardiac and respiratory status c. Electrolyte and fluid imbalance d. Mental health status

ANS: C This client may have an inflammatory cause of AKI with proteins entering the glomerulus and holding the fluid in the filtrate, causing polyuria. Electrolyte loss and fluid balance are essential. Edema and pain are not usually a problem with fluid loss. There could be changes in the client's cardiac, respiratory, and mental health status if the electrolyte imbalance is not treated.

After a total knee arthroplasty, a client is on the postoperative nursing unit with a continuous femoral nerve blockade. On assessment, the nurse notes the skin of both legs is pale pink, warm, and dry, but the client is unable to dorsiflex or plantarflex the surgical foot. What action would the nurse take next? a. Document the findings and monitor as prescribed. b. Increase the frequency of monitoring the client. c. Notify the surgeon or anesthesia provider immediately. d. Palpate the client's bladder or perform a bladder scan.

ANS: C With the femoral nerve block, the client would still be able to dorsiflex and plantarflex the affected surgical foot. Since this client has an abnormal finding, the nurse would notify either the surgeon or the anesthesia provider immediately. Documentation is the last priority. Increasing the frequency of assessment may be appropriate, but first the nurse must notify the appropriate provider. Palpating the bladder is not related.

A nurse is providing education to a community women's group about lifestyle changes helpful in preventing osteoporosis. What topics does the nurse cover? (Select all that apply.) a. Cut down on tobacco product use. b. Limit alcohol to two drinks a day. c. Strengthening exercises are important. d. Take recommended calcium and vitamin D. e. Walk for 30 minutes at least three times a week.

ANS: C, D, E Lifestyle changes can be made to decrease the occurrence of osteoporosis and include strengthening and weight-bearing exercises and getting the recommended amounts of both calcium and vitamin D. Tobacco should be totally avoided. Women should not have more than one drink per day.

The nurse reviews a list of drugs that can cause secondary osteoporosis. Which drugs are most commonly associated with this health problem? (Select all that apply.) a. Antianxiety agents b. Antibiotics c. Barbiturates d. Corticosteroids e. Loop diuretics

ANS: C, D, E Several classes of drugs can cause secondary osteoporosis, including barbiturates, corticosteroids, and loop diuretics. Antianxiety agents and antibiotics are not associated with the formation of osteoporosis.

The nurse assesses a client with rheumatoid arthritis (RA) and Sjögren syndrome. What assessment would be most important for this client? a. Abdominal assessment b. Oxygen saturation c. Breath sounds d. Visual acuity

ANS: D Sjögren syndrome may be seen in clients with RA and manifests with dryness of the eyes, mouth, and vagina in females. Visual disturbances can occur. The other assessments are not related to Sjögren syndrome.

The nurse is caring for a postoperative client who have a regional nerve blockade for a surgical tibial fracture repair this morning. What assessment finding would the nurse expect? a. Client reports nausea and vomiting. b. Client reports tingling in the surgical leg. c. Client responds well to imagery. d. Client reports little to no pain.

ANS: D A regional nerve blockade can last for about 24 hours so the client has little to no pain until it wears off. The blockade is localized and therefore does not cause nausea or vomiting.

The nurse interviews an older client with moderate osteoarthritis and her husband. What psychosocial assessment question would the nurse include? a. "Do you feel like hurting yourself or others?" b. "Are you planning to retire due to your disease?" c. "Do you ask your husband for assistance?" d. "Do you experience discomfort during sex?"

ANS: D Although some clients can become depressed and anxious as a result of having OA, suicidal ideation is not common. The nurse should not assume that an older adult will want to retire or that the client will need help from her husband. Many clients avoid sexual intercourse because of joint pain and stiffness.

The nurse is admitting a client who has acute glomerulonephritis caused by beta streptococcus. What drug therapy would the nurse expect to be prescribed for this client? a. Antihypertensives b. Antilipidemics c. Antidepressants d. Antibiotics

ANS: D Beta streptococcus is a bacterium that can cause acute glomerulonephritis, so antibiotic therapy is indicated.

The nurse is caring for several clients with osteoporosis. For which client would bisphosphonates not be a good option? a. Client with diabetes who has a serum creatinine of 0.8 mg/dL (61 mcmol/L). b. Client who recently fell and has vertebral compression fractures. c. Hypertensive client who takes calcium channel blockers. d. Client with a spinal cord injury who cannot tolerate sitting up.

ANS: D Clients on bisphosphonates must be able to sit upright for 30 to 60 minutes after taking them. The client who cannot tolerate sitting up is not a good candidate for this class of drug. Poor renal function also makes clients poor candidates for this drug, but the client with a creatinine of 0.8 mg/dL (61 mcmol/L) is within normal range. Diabetes and hypertension are not related unless the client also has renal disease. The client who recently fell and sustained fractures is a good candidate for this drug if the fractures are related to osteoporosis.

A nurse is caring for several clients with fractures. Which client would the nurse identify as being at the highest risk for developing deep vein thrombosis? a. An 18-year-old male athlete with a fractured clavicle b. A 36-year-old female with type 2 diabetes and fractured ribs c. A 55-year-old female prescribed ibuprofen for osteoarthritis d. A 74-year-old male who smokes and has a fractured pelvis

ANS: D Deep vein thrombosis (DVT) as a complication with bone fractures occurs more often when fractures are sustained in the lower extremities and the client has additional risk factors for thrombus formation. Other risk factors include obesity, smoking, oral contraceptives, previous thrombus events, advanced age, venous stasis, and heart disease. The other clients do not have additional risk factors for DVT.

The nurse is teaching a client with chronic kidney disease (CKD) about the sodium restriction needed in the diet to prevent edema and hypertension. Which statement by the client indicates that more teaching is needed? a. "I will probably lose weight by cutting out potato chips." b. "I will cut out bacon with my eggs every morning." c. "My cooking style will change by not adding salt." d. "I am thrilled that I can continue to eat fast food.

ANS: D Fast-food restaurants usually serve food that is high in sodium. This statement indicates that more teaching needs to occur. The other statements show a correct understanding of the teaching.

A nurse teaches a client with functional urinary incontinence. Which statement would the nurse include in this client's teaching? a. "You must clean around your catheter daily with soap and water." b. "You will need to be on your drug therapy for life." c. "Operations to repair your bladder are available, and you can consider these." d. "You might want to get pants with elastic waistbands."

ANS: D Functional urinary incontinence occurs as the result of problems not related to the client's bladder, such as trouble ambulating or difficulty accessing the toilet. One desired outcome is that the client will be able to manage his or her clothing independently. Elastic waistband slacks that are easy to pull down and back up can help the client get on the toilet in time to void. The other instructions do not relate to functional urinary incontinence.

A nurse plans care for a client with overflow incontinence. Which intervention does the nurse include in this client's plan of care to assist with elimination? a. Stroke the medial aspect of the thigh. b. Use intermittent catheterization. c. Provide digital anal stimulation. d. Use the Valsalva maneuver.

ANS: D In patients with overflow incontinence, the voiding reflex arc is not intact. Mechanical pressure, such as that achieved through the Valsalva maneuver (holding the breath and bearing down as if to defecate), can initiate voiding. Stroking the medial aspect of the thigh or providing digital anal stimulation requires the reflex arc to be intact to initiate elimination. Due to the high risk for infection, intermittent catheterization should only be implemented when other interventions are not successful.

A nurse cares for a client who had a wrist cast applied 3 days ago. The client states, "The cast is loose enough to slide off." How would the nurse respond? a. "Keep your arm above the level of your heart." b. "As your muscles atrophy, the cast is expected to loosen." c. "I will wrap a bandage around the cast to prevent it from slipping." d. "You need a new cast now that the swelling is decreased."

ANS: D Often the surrounding soft tissues may be swollen considerably when the cast is initially applied. After the swelling has resolved, if the cast is loose enough to permit two or more fingers between the cast and the client's skin, the cast needs to be replaced. Elevating the arm will not solve the problem, and the client's muscles should not atrophy while in a cast for 6 weeks or less. An elastic bandage will not prevent slippage of the cast.

The nurse is caring for a young client who has been diagnosed with osteopenia. Which risk factor in the client's history most likely contributed to the bone loss? a. Osteoarthritis b. Hypothyroidism c. Addison disease d. Rheumatoid arthritis

ANS: D Rheumatoid arthritis often occurs in young female adults and can lead to osteoporosis as a common complication. Cushing disease (rather than Addison disease) and hyperthyroidism (rather than hypothyroidism) are also risk factors. Osteoarthritis is a joint disease.

A nurse cares for a client who has kidney stones from gout ricemia. Which medication does the nurse anticipate administering? a. Phenazopyridine b. Doxycyline c. Tolterodine d. Allopurinol

ANS: D Stones caused by hyperuricmia caused by gout or other reason respond to allopurinol. Phenazopyridine is given to clients with urinary tract infections. Doxycycline is an antibiotic. Tolterodine is an anticholinergic with smooth muscle-relaxant properties.

The nurse teaches a client who has stress incontinence methods to regain more urinary continence. Which health teaching is the most important for the nurse to include for this client? a. What type of incontinence pads to use? b. What types of liquids to drink and when? c. Need to perform intermittent catheterizations. d. How to do Kegel exercises to strengthen muscles?

ANS: D The client who has stress incontinence needs to strengthen the muscles of the pelvic floor using Kegel exercises. Catheterizations would not help with incontinence. Incontinence pads may need to be used by this client but that is not the most important thing to teach, and it does not help the client regain more control over his or her bladder.

After teaching a client with a fractured humerus, the nurse assesses the client's understanding. Which dietary choice demonstrates that the client correctly understands the nutrition needed to assist in healing the fracture? a. Baked fish with orange juice and a vitamin D supplement b. Bacon, lettuce, and tomato sandwich with a vitamin B supplement c. Vegetable lasagna with a green salad and a vitamin A supplement d. Roast beef with low-fat milk and a vitamin C supplement

ANS: D The client with a healing fracture needs supplements of vitamins B and C and a high-protein, high-calorie diet. Milk for calcium supplementation and vitamin C supplementation is appropriate. Meat would increase protein in the diet that is necessary for bone healing. Fish, a sandwich, and vegetable lasagna would provide less protein.

An older client who fell at home is admitted to the emergency department and reports pain in her left groin and behind her left knee. What action would the nurse anticipate? a. Administer IV push morphine. b. Prepare for application of a leg cast. c. Begin oxygen at 6 L/min via mask. d. Obtain a left hip x-ray.

ANS: D The location of the client's pain indicates a possible fractured hip and therefore an x-ray of the hip is needed. A leg cast is not appropriate and oxygen may not be needed. Medication to make the client more comfortable would likely be needed after a diagnosis is determined.

The nurse is performing a neurovascular assessment for an older client who has an extremity fracture. How many seconds would the nurse expect for a capillary refill in it is within normal range? a. 20 seconds b. 15 seconds c. 10 seconds d. 5 seconds

ANS: D The normal capillary refill is usually 3 seconds, but for older adults, the refill usually takes up to 5 seconds due to vascular changes associated with aging.

A nurse is caring for a client who is recovering from an above-the-knee amputation and reports pain in the limb that was removed. How would the nurse respond? a. "The pain you are feeling does not actually exist." b. "This type of pain is common and will eventually go away." c. "Would you like to learn how to use imagery to minimize your pain?" d. "How would you describe the pain that you are feeling?"

ANS: D The nurse would ask the client to rate the pain on a scale of 0-10 and describe how the pain feels. Although phantom limb pain is common, the nurse would not minimize the pain that the client is experiencing by stating that it does not exist or will eventually go away. Although imagery may help, the nurse must assess the client's pain before determining the best action.

A client with acute kidney injury (AKI) has a blood pressure of 76/55 mm Hg. The primary health care provider prescribed 1000 mL of normal saline to be infused over 1 hour to maintain perfusion. The client starts to develop shortness of breath. What is the nurse's priority action? a. Calculate the mean arterial pressure (MAP). b. Ask for insertion of a pulmonary artery catheter. c. Take the client's pulse. d. Decrease the rate of the IV infusion.

ANS: D The nurse would assess that the client could be developing fluid overload and respiratory distress and slow down the normal saline infusion. The calculation of the MAP also reflects perfusion. The insertion of a pulmonary artery catheter would evaluate the client's hemodynamic status, but this would not be the initial or priority action by the nurse. Vital signs are also important after adjusting the intravenous infusion.

A nurse assesses a client who is recovering from a radical nephrectomy for renal cell carcinoma. The nurse notes that the client's blood pressure has decreased from 134/90 to 100/56 mm Hg and urine output is 20 mL for this past hour. What action would the nurse take? a. Position the client to lay on the surgical incision. b. Measure the specific gravity of the client's urine. c. Administer intravenous pain medications. d. Assess the rate and quality of the client's pulse.

ANS: D The nurse would first fully assess the client for signs of volume depletion and shock, and then notify the primary health care provider. The extensive nature of the surgery and the proximity of the surgery to the adrenal gland put the client at risk for hemorrhage and adrenal insufficiency. Hypotension is a clinical manifestation associated with both hemorrhage and adrenal insufficiency. Hypotension is particularly dangerous for the remaining kidney, which must receive adequate perfusion to function effectively. Repositioning the patient, measuring specific gravity, and administering pain medication would not provide data necessary to make an appropriate clinical decision, nor are they appropriate interventions at this time.

A client is scheduled to have a total hip arthroplasty. What preoperative teaching by the nurse is most important? a. Teach the need to discontinue all medications for 5 days before surgery. b. Teach the patient about foods high in protein, Vitamin C, and iron. c. Explain to the client the possible need for blood transfusions postoperatively. d. Remind the client to have all dental procedures completed at least 2 weeks prior to surgery.

ANS: D The nurse would include teaching about dental procedures to avoid infection after new joint has been inserted. Planned procedures would be completed at least 2 weeks before surgery and the client will need to tell any future primary health care providers about having a total joint arthroplasty. Only home medications prescribed that increase the risk for bleeding or clotting need to be discontinued 5 to 10 days before surgery. Clients need to be aware that any postoperative anemia may need to be treated with a blood transfusion, but it is not the most important. Diets high in protein, Vitamin C, and iron help with tissue repair, but are not the most important.

A nurse assesses a client with a pelvic fracture. Which assessment finding would the nurse identify as a complication of this injury? a. Hypertension b. Diarrhea c. Infection d. Hematuria

ANS: D The pelvis is very vascular and close to major organs. Injury to the pelvis can cause integral damage that may manifest as blood in the urine (hematuria) or stool. The nurse would also assess for signs of hemorrhage and hypovolemic shock, which include hypotension and tachycardia. Diarrhea and infection are not common complications of a pelvic fracture.

The nurse administers epoetin alfa to a client who has chronic kidney disease (CKD). Which laboratory test value would the nurse monitor to determine this drug's effectiveness? a. Potassium b. Sodium c. Renin d. Hemoglobin

ANS: D The purpose of giving epoetin alfa to a client with CKD is to manage anemia by stimulating the bone marrow to produce more red blood cells. Therefore, monitoring the client's hemoglobin, hematocrit, and red blood cell count would indicate if the drug was effective.

A client with HIV-III and wasting syndrome has inadequate nutrition. What assessment finding by the nurse best indicates that goals have been met for this client problem? a. Chooses high-protein food. b. Has decreased oral discomfort. c. Eats 90% of meals and snacks. d. Has a weight gain of 2 lb (1 kg)/1 mo.

ANS: D The weight gain is the best indicator that goals for this client problem have been met because it demonstrates that the client not only is eating well but also is able to absorb the nutrients. Choosing high-protein food is important, but only if the client eats and absorbs the nutrients.

A client is hospitalized with Pneumocystis jiroveci pneumonia. The client reports shortness of breath with activity and extreme fatigue. What intervention is best to promote comfort? a. Administer sleeping medication. b. Perform most activities for the client. c. Increase the client's oxygen during activity. d. Pace activities, allowing for adequate rest.

ANS: D This client has two major reasons for fatigue: decreased oxygenation and systemic illness. The nurse would not do everything for the client but rather let the client do as much as possible within limits and allow for adequate rest in between. Sleeping medications may be needed but not as the first step, and only with caution. Increasing oxygen during activities may or may not be warranted, but first the nurse must try pacing the client's activity.

A nurse plans care for a client who has an external fixator on the lower leg. Which intervention would the nurse include in the plan of care to decrease the client's risk for infection? a. Washing the frame of the fixator once a day b. Releasing fixator tension for 30 minutes twice a day c. Avoiding moving the extremity by holding the fixator d. Scheduling for pin care to be provided every shift

ANS: D To decrease the risk for infection in a client with skeletal traction or external fixation, the nurse would provide routine pin care and assess for signs and symptoms of infection at the pin sites every shift.

A nurse teaches assistive personnel (AP) about providing hygiene for a client in traction. Which statement would the nurse include as part of the teaching about this client's care? a. "Remove the traction when re-positioning the client." b. "Assess the client's skin when performing a bed bath." c. "Provide pin care by using alcohol wipes to clean the sites." d. "Ensure that the weights remain freely hanging at all times."

ANS: D Traction weights should be freely hanging at all times. They should not be lifted manually or allowed to rest on the floor. The client should remain in traction during hygiene activities. The nurse would assess the client's skin and provide pin and wound care for a patient who is in traction; this would not be delegated to the AP.

The nurse providing direct client care uses specific practices to reduce the chance of acquiring infection with human immune deficiency virus (HIV) from clients. Which practice is most effective? a. Consistent use of Standard Precautions b. Double-gloving before body fluid exposure c. Labeling charts and armbands "HIV+" d. Wearing a mask within 3 feet (1 m) of the client.

ANS:A According to The Joint Commission, the most effective preventative measure to avoid HIV exposure is consistent use of Standard Precautions. Standard Precautions are required by the CDC. Double-gloving is not necessary. Labeling charts and armbands in this fashion is a violation of the Health Information Portability and Accountability Act (HIPAA). Wearing a mask within 3 feet (1 m) of the client is not necessary with every client contact.

After teaching a client with a history of renal calculi, the nurse assesses the client's understanding. Which statement made by the client indicates a correct understanding of the teaching? a. "I should drink at least 3 L of fluid every day." b. "I will eliminate all dairy or sources of calcium from my diet." c. "Aspirin and aspirin-containing products can lead to stones." d. "The doctor can give me antibiotics at the first sign of a stone."

After teaching a client with a history of renal calculi, the nurse assesses the client's understanding. Which statement made by the client indicates a correct understanding of the teaching? a. "I should drink at least 3 L of fluid every day." b. "I will eliminate all dairy or sources of calcium from my diet." c. "Aspirin and aspirin-containing products can lead to stones." d. "The doctor can give me antibiotics at the first sign of a stone."

The nurse is teaching a client who has been treated for acute gastritis. What statement by the client indicates a need for further teaching? a. "I need to cut down on drinking martinis every might." b. "I should decrease my intake of caffeinated drinks, especially coffee." c. "I will only take ibuprofen once in a while when I really need it." d. "I can continue smoking cigarettes which is better than chewing tobacco."

D To prevent another episode of acute gastritis, alcohol, caffeinated drinks, and NSAIDs should be avoided or kept at a minimum. Smoking and all forms of tobacco should also be avoided.

2. A client is admitted with a large oral tumor. What assessment by the nurse takes priority? a. Airway b. Breathing c. Circulation d. Nutrition

a Airway always takes priority. Airway must be assessed first and any problems managed if present.

A client presents to the emergency department in sickle cell disease crisis. What intervention by the nurse takes priority? a. Administer oxygen. b. Initiate pulse oximetry. c. Give pain medication. d. Start an IV line.

a All actions are appropriate, but remembering the ABCs, oxygen would come first. The main problem in a sickle cell crisis is tissue and organ hypoxia, so providing oxygen helps halt the process.

A nurse working with clients who experience alopecia knows that which is the best method of helping clients manage the psychosocial impact of this problem? a. Assisting the client to pre-plan for this event b. Reassuring the client that alopecia is temporary c. Teaching the client ways to protect the scalp d. Telling the client that there are worse side effects

a Alopecia does not occur for all clients who have cancer, but when it does, it can be devastating. The best action by the nurse is to teach the client about the possibility and to give the client multiple choices for preparing for this event. Not all clients will have the same reaction, but some possible actions the client can take are buying a wig ahead of time, buying attractive hats and scarves, and having a hairdresser modify a wig to look like the client's own hair. Teaching about scalp protection is important but does not address the psychosocial impact. Reassuring the client that hair loss is temporary and telling him or her that there are worse side effects are both patronizing and do not give the client tools to manage this condition.

The nurse is caring for a client with hepatitis C. The client's brother states, "I do not want to get this infection, so I'm not going into his hospital room." How would the nurse respond? a. "Hepatitis C is not spread through casual contact." b. "If you wear a gown and gloves, you will not get this virus." c. "This virus is only transmitted through a fecal specimen." d. "I can give you an update on your brother's status from here."

a Although family members may be afraid that they will contract hepatitis C, the nurse would educate them about how the virus is spread. Hepatitis C is spread via blood-to-blood transmission and is associated with illicit IV drug needle sharing, blood and organ transplantation, accidental needlesticks, unsanitary tattoo equipment, and sharing of intranasal drug paraphernalia. Wearing a gown and gloves will not decrease the transmission of this virus. Hepatitis C is not spread through casual contact or a fecal specimen. The nurse would be violating privacy laws by sharing the client's status with the brother.

A nurse caring for a client with sickle cell disease (SCD) reviews the client's laboratory test results. Which finding would the nurse report to the primary health care provider? a. Creatinine: 2.9 mg/dL (256 mcmol/L) b. Hematocrit: 30% c. Sodium: 146 mEq/L (146 mmol/L) d. White blood cell count: 12,000/mm3 (12 × 109 /L)

a An elevated creatinine indicates kidney damage, which occurs in SCD. A hematocrit level of 30% is an expected finding, as is a slightly elevated white blood cell count due to chronic inflammation. A sodium level of 146 mEq/L (146 mmol/L), although slightly high, is not concerning.

A male client was admitted with a left-sided stroke this morning. The assistive personnel asks about meeting the client's nutritional needs. Which response by the nurse is appropriate? a. "He is NPO until the speech-language pathologist performs a swallowing evaluation." b. "You may give him a full-liquid diet, but please avoid solid foods until he gets stronger." c. "Just be sure to add some thickener in his liquids to prevent choking and aspiration." d. "Be sure to sit him up when you are feeding him to make him feel more natural."

a Any client who has or is suspected of having a stroke should have nothing by mouth until he or she is evaluated for any swallowing problem by the speech-language pathologist (SLP). If dysphagia is present, the SLP makes specific recommendations for the client's plan of care which all staff members must follow to prevent choking and aspiration/aspiration pneumonia.

The nurse working with oncology clients understands that which age-related change increases the older client's susceptibility to infection during chemotherapy? a. Decreased immune function b. Diminished nutritional stores c. Existing cognitive deficits d. Poor physical reserves

a As people age, there is an age-related decrease in immune function, causing the older adult to be more susceptible to infection than other clients. Not all older adults have diminished nutritional stores, cognitive dysfunction, or poor physical reserves.

A nurse obtains a focused health history for a client who is suspected of having bacterial meningitis. Which question would the nurse ask? a. "Do you live in a crowded residence?" b. "When was your last tetanus vaccination?" c. "Have you had any viral infections recently?" d. "Have you traveled out of the country in the last month?"

a Bacterial meningitis tends to occur in multiple outbreaks. It is most likely to occur in areas of high-density population, such as college dormitories, prisons, and military barracks. A tetanus vaccination would not place the client at increased risk for meningitis or protect the client from meningitis. A viral infection would not lead to bacterial meningitis but could lead to viral meningitis. Simply knowing if the client traveled out of the country does not provide enough information.

A client in sickle cell crisis is dehydrated and in the emergency department. The nurse plans to start an IV. Which fluid choice is best? a. 0.45% normal saline b. 0.9% normal saline c. Dextrose 50% (D50) d. Lactated Ringer's solution

a Because clients in sickle cell crisis are often dehydrated, the fluid of choice is a hypotonic solution such as 0.45% normal saline. 0.9% normal saline and lactated Ringer's solution are isotonic. D50 is hypertonic and not used for hydration.

The nurse caring for oncology clients knows that which form of metastasis is the most common? a. Bloodborne b. Direct invasion c. Lymphatic spread d. Via bone marrow

a Bloodborne metastasis is the most common way for cancer to metastasize. Direct invasion and lymphatic spread are other methods. Bone marrow is not a medium in which cancer spreads, although cancer can occur in the bone marrow

A client has a brain tumor and is receiving phenytoin (Dilantin). The spouse questions the use of the drug, saying that the client does not have a seizure disorder. What response by the nurse is correct? a. "Increased pressure from the tumor can cause seizures." b. "Preventing febrile seizures with a tumor is important." c. "Seizures always occur in clients with brain tumors." d. "This drug is used to sedate with a brain tumor."

a Brain tumors can lead to seizures as a complication. The nurse would explain this to the spouse. Preventing febrile seizures is not related to a tumor. Seizures are possible but do not always occur in clients with brain tumors. This drug is not used for sedation.

A nurse in the oncology clinic is providing preoperative education to a client just diagnosed with cancer. The client has been scheduled for surgery in 3 days. What action by the nurse is best? a. Call the client at home the next day to review teaching. b. Give the client information about a cancer support group. c. Provide all the preoperative instructions in writing. d. Reassure the client that surgery will be over soon.

a Clients are often overwhelmed at a sudden diagnosis of cancer and may be more overwhelmed at the idea of a major operation so soon. This stress significantly impacts the client's ability to understand, retain, and recall information. The nurse would call the client at home the next day to review the teaching and to answer questions. The client may or may not be ready to investigate a support group, but this does not help with teaching. Giving information in writing is important (if the client can read it), but in itself will not be enough. Telling the client that surgery will be over soon is giving false reassurance and does nothing for teaching.

A client who has multiple sclerosis reports increased severe muscle spasticity and tremors. What nursing action is most appropriate to manage this client's concern? a. Request a prescription for an antispasmodic drug such as baclofen. b. Prepare the client for deep brain stimulation surgery. c. Refer the client to a massage therapist to relax the muscles. d. Consult with the occupational therapist for self-care assistance.

a Clients who have multiple sclerosis often have muscle spasticity which may be reduced by drug therapy, such as baclofen. While massage and assistance with self-care may be helpful, these interventions are not the most effective and therefore not the most appropriate in managing muscle spasticity. If drug therapy and other interventions do not help reduce muscle spasms, some client are candidates for deep brain stimulation as a last resort.

The nurse is teaching a client who has pernicious anemia about necessary dietary changes. Which statement by the client indicates understanding about those changes? a. "I'll increase animal proteins like fish and meat." b. "I'll work on increasing my fats and carbohydrates." c. "I'll avoid eating green leafy vegetables. d. "I'll limit my intake of citrus fruits."

a Clients who have pernicious anemia have a Vitamin B12 deficiency and need to consume foods high in Vitamin B12, such as animal and plant proteins, citrus fruits, green leafy vegetables, and dairy products. While carbohydrates and fats can provide sources of energy, they do not supply the necessary nutrient to improve anemia.

A nurse teaches assistive personnel (AP) about how to care for a client with Parkinson disease. Which statement would the nurse include as part of this teaching? a. "Allow the client to be as independent as possible with activities." b. "Assist the client with frequent and meticulous oral care." c. "Assess the client's ability to eat and swallow before each meal." d. "Schedule appointments early in the morning to ensure rest in the afternoon."

a Clients with Parkinson disease do not move as quickly and can have functional problems. The client would be encouraged to be as independent as possible and provided time to perform activities without rushing. Although oral care is important for all clients, instructing the UAP to provide frequent and meticulous oral is not a priority for this client. This statement would be a priority if the client was immune-compromised or NPO. The nurse would assess the client's ability to eat and swallow; this would not be delegated. Appointments and activities would not be scheduled early in the morning because this may cause the client to be rushed and discourage the client from wanting to participate in activities of daily living.

A client hospitalized with sickle cell disease crisis frequently asks for opioid pain medications, often shortly after receiving a dose. The nurses on the unit believe that the client is drug seeking. When the client requests pain medication, what action by the nurse is best? a. Give the client pain medication if it is time for another dose. b. Instruct the client not to request pain medication too early. c. Request the primary health care provider leave a prescription for a placebo. d. Tell the client that it is too early to have more pain medication.

a Clients with sickle cell crisis often have severe pain that is managed with up to 48 hours of IV opioid analgesics. Even if the client is addicted and drug seeking, he or she is still in extreme pain. If the client can receive another dose of medication, the nurse would provide it. The other options are judgmental and do not address the client's pain. Giving a placebo is unethical.

The nurse is teaching a family caregiver about how best to communicate with the client who has been diagnosed with Alzheimer disease. Which statement by the caregiver indicates a need for further teaching? a. "I will avoid communicating with the client to prevent agitation." b. "I should use simple, short sentences and one-step instructions." c. "I can try to use gestures or pictures to communicate with the client." d. "I will limit the number of choices I provide for the client."

a Communication with the client is important to provide cognitive stimulation. Using short simple sentences, using gestures and pictures, and limiting choices provided for the client will help promote communication.

A nurse assesses a client who is recovering from a Whipple procedure. Which assessment finding alerts the nurse to immediately contact the primary health care provider? a. Drainage from a fistula b. Diminished bowel sounds c. Pain at the incision site d. Nasogastric (NG) tube drainage

a Complications of a Whipple procedure include secretions that drain from a fistula and peritonitis. Absent bowel sounds, pain at the incision site, and NG tube drainage are normal postoperative findings.

A client asks the nurse if eating only preservative- and dye-free foods will decrease cancer risk. What response by the nurse is best? a. "Maybe; preservatives, dyes, and preparation methods may be risk factors." b. "No; research studies have never shown those things to cause cancer." c. "There are other things you can do that will more effectively lower your risk." d. "Yes; preservatives and dyes are well known to be carcinogens."

a Dietary factors related to cancer development are poorly understood, although dietary practices are suspected to alter cancer risk. Suspected dietary risk factors include low-fiber intake and a high intake of red meat or animal fat. Preservatives, preparation methods, and additives (dyes, flavorings, sweeteners) may have cancer-promoting effects. It is correct to say that other things can lower risk more effectively, but this does not give the client concrete information about how to do so, and also does not answer the client's question.

A client who had therapeutic hypothermia after a traumatic brain injury is slowly rewarmed to a normal core temperature. For which assessment finding would the nurse monitor during the rewarming process? a. Cardiac dysrhythmias b. Loss of consciousness c. Nausea and vomiting d. Fever

a Due to fluid and electrolyte changes that typically occur during the rewarming process, the nurse monitors for cardiac dysrhythmias. The other findings are not common during this process.

The nurse is teaching a client a client about taking elbasvir for hepatitis C. What information in the client's history would the nurse need prior to drug administration? a. History of hepatitis B b. History of kidney disease c. History of cardiac disease d. History of rectal bleeding

a Elbasvir can cause liver toxicity and therefore the nurse would assess for a history of or current hepatitis B.

A nurse works on an oncology unit and delegates personal hygiene to assistive personnel (AP). What action by the AP requires intervention from the nurse? a. Allowing a very tired client to skip oral hygiene and sleep b. Assisting clients with washing the perianal area every 12 hours c. Helping the client use a soft-bristled toothbrush for oral care d. Reminding the client to rinse the mouth with water or saline

a Even though clients may be tired, they still need to participate in hygiene to help prevent infection. The nurse would intervene and explain this to AP. The other options are all appropriate.

The nurse is caring for a patient with leukemia who has severe fatigue. What action by the client best indicates that an important outcome to manage this problem has been met? a. Doing activities of daily living (ADLs) using rest periods b. Helping plan a daily activity schedule c. Requesting a sleeping pill at night d. Telling visitors to leave when fatigued

a Fatigue is a common problem for clients with leukemia. This client is managing his or her own ADLs using rest periods, which indicates an understanding of fatigue and how to control it. Helping to plan an activity schedule is a lesser indicator. Requesting a sleeping pill does not help control fatigue during the day. Asking visitors to leave when tired is another lesser indicator. Managing ADLs using rest periods demonstrates the most comprehensive management strategy.

A nurse reviews the electronic health record of a client who has Crohn disease and a draining fistula. Which documentation would alert the nurse to urgently contact the primary health care provider for additional prescriptions? a. Serum potassium of 2.6 mEq/L (2.6 mmol/L) b. Client ate 20% of breakfast meal c. White blood cell count of 8200/mm3 (8.2 × 109 /L) d. Client's weight decreased by 3 lb (1.4 kg)

a Fistulas place the patient with Crohn disease at risk for hypokalemia which can lead to serious dysrhythmias. This potassium level is low and would cause the nurse to intervene. The white blood cell count is normal. The other two findings are abnormal and also warrant intervention, but the potassium level takes priority.

A nurse is providing community screening for risk factors associated with stroke. Which person would the nurse identify as being at the highest risk for a stroke? a. A 27-year-old heavy-cocaine user. b. A 30-year-old who drinks a beer a day. c. A 40-year-old who uses seasonal antihistamines. d. A 65-year-old who is active and on no medications.

a Heavy drug use, particularly cocaine, is a risk factor for stroke. Heavy alcohol use is also a risk factor, but one beer a day is not considered heavy drinking. Antihistamines may contain phenylpropanolamine, which also increases the risk for stroke, but this person uses them seasonally and there is no information that they are abused or used heavily. The 65 year old has only age as a risk factor.

A client is receiving chemotherapy through a peripheral IV line. What action by the nurse is most important? a. Assessing the IV site and blood return every hour b. Educating the client on side effects c. Monitoring the client for nausea d. providing warm packs for comfort

a Intravenous chemotherapy can cause local tissue destruction if it extravasates into the surrounding tissues. Peripheral IV lines are more prone to this than centrally placed lines. The most important intervention is prevention, so the nurse would check hourly to ensure the IV site is patent, or frequently depending on facility policy. Education and monitoring for side effects such as nausea are important for all clients receiving chemotherapy. Warm packs may be helpful for some drugs, whereas for others ice is more comfortable. would monitor the site and check for blood return to prevent injury from infiltration or extravasation.

A nurse assesses a client who is prescribed alosetron. Which assessment question would the nurse ask this client before starting the drug? a. "Have you been experiencing any constipation?" b. "Are you eating a diet high in fiber and fluids?" c. "Do you have a history of high blood pressure?" d. "What vitamins and supplements are you taking?"

a Ischemic colitis is a life-threatening complication of alosetron. The nurse would assess the client for constipation because it places the client at risk for this complication. The other questions do not identify the risk for complications related to alosetron.

The nurse notes that the primary health care provider documented the presence of mucosal erythroplasia in a client. What does the nurse understand that this most likely means for this client? a. Early sign of oral cancer b. Fungal mouth infection c. Inflammation of the gums d. Obvious oral tumor

a Mucosal erythroplasia is the earliest sign of oral cancer. It is not a fungal infection, inflammation of the gums, or an obvious tumor.

The nurse is caring for a 60-year-old female client who sustained a thoracic spinal cord injury 10 years ago. For which potential complication will the nurse assess during this client's care? a. Fracture b. Malabsorption c. Delirium d. Anemia

a Older adults who have impaired mobility due to a health problem or injury are at risk for complications of immobility, such as osteoporosis (bone loss) which leads to fracture. Being an older woman increases that risk due to loss of estrogen to protect bone loss. The other choices are not problems of immobility. Delirium is possible but is more common in clients over 70 years of age.

A client is in the clinic for a follow-up visit after a moderate traumatic brain injury. The patient's spouse is very frustrated, stating that the patient's personality has changed and the situation is very difficult. What response by the nurse is most appropriate? a. Explain that personality changes are common following brain injuries. b. Ask the client why he or she is acting out and behaving differently. c. Refer the client and spouse to a head injury support group. d. Tell the spouse that this is expected and he or she will have to learn to cope.

a Personality and behavior often change permanently after head injury. The nurse will explain this to the spouse. Asking the client about his or her behavior isn't useful because the patient probably cannot help it. A referral might be a good idea, but the nurse needs to do something in addition to just referring the couple. Telling the spouse to learn to cope belittles his or her concerns and feelings.

A nurse assesses a client who is recovering from an open anterior cervical discectomy and fusion. Which complication would alert the nurse to urgently communicate with the primary health care provider? a. Auscultated stridor b. Weak pedal pulses c. Difficulty swallowing d. Inability to shrug shoulders

a Postoperative swelling can narrow the trachea, cause a partial airway obstruction, and manifest as stridor. The client may also have trouble swallowing, but maintaining an airway takes priority. Weak pedal pulses and an inability to shrug the shoulders are not complications of this surgery.

The nurse assesses a client who is recovering from a paracentesis 1 hour ago. Which assessment finding would require immediate action by the nurse? a. Urine output via indwelling urinary catheter is 20 mL/hr b. Blood pressure increases from 110/58 to 120/62 mm Hg c. Respiratory rate decreases from 22 to 16 breaths/min d. A decrease in the client's weight by 3 lb (1.4 kg)

a Rapid removal of ascitic fluid causes decreased abdominal pressure, which can contribute to hypovolemia. This can be manifested by a decrease in urine output to below 30 mL/hr. A slight increase in systolic blood pressure is insignificant. A decrease in respiratory rate indicates that breathing has been made easier by the procedure. The nurse would expect the client's weight to drop as fluid is removed. To prevent hypovolemic shock, no more than 2000 mL are usually removed from the abdomen at one time. The patient's weight typically only decreases by less than 2 kg or 4.4 lb.

The nurse assesses a client who has appendicitis. Which assessment finding would the nurse expect? a. Severe, steady right lower quadrant pain b. Abdominal pain associated with nausea and vomiting c. Marked peristalsis and hyperactive bowel sounds d. Abdominal pain that increases with knee flexion

a Right lower quadrant pain, specifically at McBurney's point, is characteristic of appendicitis. Usually if nausea and vomiting begin first, the client has gastroenteritis. Marked peristalsis and hyperactive bowel sounds are not indicative of appendicitis. Abdominal pain due to appendicitis decreases with knee flexion.

A client is admitted with a sudden decline in level of consciousness. What is the nursing action at this time? a. Assess the client for hypoglycemia and hypoxia. b. Place the client on his or her side. c. Prepare for administration of a fibrinolytic agent. d. Start a continuous IV heparin sodium infusion.

a The cause of a sudden decline in level of consciousness may or may not be related to a neurologic health problem. Therefore, the client must be evaluated for other common causes, especially hypoglycemia and hypoxia. Placing the client on his or her side may be helpful to prevent aspiration in case the client experiences vomiting, but the clinical situation does not indicate that the client has nausea or vomiting. Administering either an anticoagulant like heparin or a fibrinolytic agent assumes that the client has an acute ischemic stroke, which has not been confirmed through imaging tests.

A nurse cares for a client with end-stage pancreatic cancer. The client asks, "Why is this happening to me?" How would the nurse respond? a. "I don't know. I wish I had an answer for you, but I don't." b. "It's important to keep a positive attitude for your family right now." c. "Scientists have not determined why cancer develops in certain people." d. "I think that this is a trial so you can become a better person because of it"

a The client is not asking the nurse to actually explain why the cancer has occurred. The client may be expressing his or her feelings of confusion, frustration, distress, and grief related to this diagnosis. Reminding the client to keep a positive attitude for his or her family does not address the client's emotions or current concerns. The nurse would validate that there is no easy or straightforward answer as to why the client has cancer. Telling a client that cancer is a trial is untrue and may negatively impact the client-nurse relationship.

The nurse teaches a client who has viral gastroenteritis. Which dietary instruction would the nurse include in the health teaching? a. "Drink plenty of fluids to prevent dehydration." b. "You should only drink 1 L of fluids daily." c. "Increase your protein intake by drinking more milk." d. "Sips of cola or tea may help to relieve your nausea."

a The client should drink plenty of fluids to prevent dehydration. Milk products may not be tolerated. Caffeinated beverages increase intestinal motility and should be avoided.

A nurse is caring for four clients with leukemia. After hand-off report, which client would the nurse assess first? a. Client who had two bloody diarrhea stools this morning. b. Client who has been premedicated for nausea prior to chemotherapy. c. Client with a respiratory rate change from 18 to 22 breaths/min. d. Client with an unchanged lesion to the lower right lateral malleolus.

a The client who had two bloody diarrhea stools that morning may be hemorrhaging in the gastrointestinal (GI) tract and should be assessed first to monitor for or avoid the client from going into hypovolemic shock. The client with the slight change in respiratory rate may have an infection or worsening anemia and should be seen next. If the client's respiratory rate was greater than 28 to 30 breaths/min, the client may need the initial assessment. Marked tachypnea is an early sign of a deteriorating client condition. The other two clients are not a priority at this time.

A client is admitted with a diagnosis of cerebellar stroke. What intervention is most appropriate to include on the client's plan of care? a. Ambulate only with a gait belt. b. Encourage double swallowing. c. Monitor lung sounds after eating. d. Perform postvoid residuals.

a The client who has a cerebellar stroke would be expected to have ataxia, an abnormal gait. For the client's safety, he or she should have assistance and use a gait belt when ambulating. Ataxia is not related to swallowing, aspiration, or voiding.

After teaching a client with a high thoracic spinal cord injury, the nurse assesses the client's understanding. Which statement by the client indicates a correct understanding of how to prevent respiratory problems at home? a. "I'll use my incentive spirometer every 2 hours while I'm awake." b. "I'll drink thinned fluids to prevent choking." c. "I'll take cough medicine to prevent excessive coughing." d. "I'll position myself on my right side so I don't aspirate."

a The client with a cervical or high thoracic spinal cord injury typically has weak intercostal muscles and is at higher risk for developing atelectasis and stasis pneumonia. Using an incentive spirometer every 2 hours helps the client expand the lungs more fully and helps prevent atelectasis and other respiratory problems. Clients should drink fluids that they can tolerate; usually thick fluids are easier to tolerate. The client would be encouraged to cough and clear secretions, and placed in high-Fowler position to prevent aspiration.

A client has dumping syndrome after a partial gastrectomy. Which action by the nurse would be appropriate? a. Arrange a dietary consult. b. Increase fluid intake. c. Limit the client's foods. d. Make the client NPO.

a The client with dumping syndrome after a gastrectomy has multiple dietary needs. A referral to the registered dietitian nutritionist will be extremely helpful. Food and fluid intake is complicated and needs planning. The client should not be NPO.

A client hospitalized for chemotherapy has a hemoglobin of 6.1 mg/dL (61 mmol/L). The client is symptomatic but refuses blood transfusions. What medication does the nurse prepare to administer? a. Epoetin alfa b. Filgrastim c. Mesna d. Dexrazoxane

a The client's hemoglobin is very low, so the nurse prepares to administer epoetin alfa, a colony-stimulating factor that increases production of red blood cells. Filgrastim is for neutropenia. Mesna is used to decrease bladder toxicity from some chemotherapeutic agents. Dexrazoxane helps protect the heart from cardiotoxicity from other agents.

The nurse is caring for a client who has frequent gastric pain and dyspepsia. Which procedure would the nurse expect for the client to make an accurate diagnosis? a. Esophagogastroduodenoscopy (EGD) b. Abdominal arteriogram c. Nuclear medicine scan d. Magnetic resonance imaging (MRI)

a The gold standard for diagnosing disorders of the stomach is an EGD which allows direct visualization by the endoscopist into the esophagus, stomach, and duodenum.

The nurse assesses a patient who is recovering from an ileostomy placement. Which assessment finding would alert the nurse to immediately contact the primary health care provider? a. Pale and bluish stoma b. Liquid stool c. Ostomy pouch intact d. Blood-tinged output

a The nurse would assess the stoma for color and contact the primary health care provider if the stoma is pale, bluish, or dark because these changes indicate possible lack of perfusion. The nurse would expect the client to have an intact ostomy pouch with dark green liquid stool that may contain some blood.

A nurse cares for a client who states, "My husband is repulsed by my colostomy and refuses to be intimate with me." How would the nurse respond? a. "Let's talk to the ostomy nurse to help you and your husband work through this." b. "You could try to wear longer lingerie that will better hide the ostomy appliance." c. "You should empty the pouch first so it will be less noticeable for your husband." d. "If you are not careful, you can hurt the stoma if you engage in sexual activity."

a The nurse would collaborate with the ostomy nurse to help the client and her husband work through intimacy issues. The nurse would not minimize the client's concern about her husband with ways to hide the ostomy. The client will not hurt the stoma by engaging in sexual activity.

A nurse cares for a client who has a new colostomy. Which action would the nurse take? a. Empty the pouch frequently to remove excess gas collection. b. Change the ostomy pouch and barrier every morning. c. Allow the pouch to completely fill with stool prior to emptying it. d. Use surgical tape to secure the pouch and prevent leakage.

a The nurse would empty the new ostomy pouch frequently because of excess gas collection, and empty the pouch when it is one-third to one-half full of stool. The ostomy pouch does not need to be changed every morning. Ostomy barriers would be used to secure and seal the ostomy appliance; surgical tape would not be used.

The nurse is caring for a client experiencing upper gastrointestinal (GI) bleeding. What is the priority action for the client's care? a. Maintain airway, breathing, and circulation. b. Monitor vital signs, including orthostatic blood pressures. c. Draw blood for hemoglobin and hematocrit immediately. d. Insert a nasogastric (NG) tube and connect to intermittent suction.

a The priority action for any client experiencing deterioration or an emergent situation is monitor and maintain airway, breathing, and circulation (ABCs). Taking orthostatic blood pressures would not be appropriate, but the nurse would monitor vital signs carefully and draw blood for hemoglobin and hematocrit. An NG tube would also need to be inserted and connected to gastric suction to rest the GI tract. However, none of these actions take priority over maintaining ABCs.

The nurse is caring for a client who is prescribed lactulose. The client states, "I do not want to take this medication because it causes diarrhea." How would the nurse respond? a. "Diarrhea is expected; that's how your body gets rid of ammonia." b. "You may take antidiarrheal medication to prevent loose stools." c. "Do not take any more of the medication until your stools firm up." d. "We will need to send a stool specimen to the laboratory as soon as possible."

a The purpose of administering lactulose to this patient is to help ammonia leave the circulatory system through the colon. Lactulose draws water into the bowel with its high osmotic gradient, thereby producing a laxative effect and subsequently evacuating ammonia from the bowel. The patient must understand that this is an expected and therapeutic effect for him or her to remain compliant. The nurse would not suggest administering anything that would decrease the excretion of ammonia or holding the medication. There is no need to send a stool specimen to the laboratory because diarrhea is the therapeutic response to this medication.

A nurse cares for a client who had a colostomy placed in the ascending colon 2 weeks ago. The client states, "The stool in my pouch is still liquid." How would the nurse respond? a. "The stool will always be liquid with this type of colostomy." b. "Eating additional fiber will bulk up your stool and decrease diarrhea." c. "Your stool will become firmer over the next couple of weeks." d. "This is abnormal. I will contact your primary health care provider."

a The stool from an ascending colostomy can be expected to remain liquid because little large bowel is available to reabsorb the liquid from the stool. This finding is not abnormal. Liquid stool from an ascending colostomy will not become firmer with the addition of fiber to the client's diet or with the passage of time.

The nurse has educated a client on precautions to take with thrombocytopenia. What statement by the client indicates a need to review the information? a. "I will be careful if I need enemas for constipation." b. "I will use an electric shaver instead of a razor." c. "I should only eat soft food that is either cool or warm." d. "I won't be able to play sports with my grandkids."

a The thrombocytopenic client is at high risk for bleeding even from minor trauma. Due to the risk of injuring rectal and anal tissue, the client should not use enemas or rectal thermometers. This statement would indicate the client needs more information. The other statements are appropriate for the thrombocytopenic client.

A client has been admitted after sustaining a humerus fracture that occurred when picking up the family cat. What test result would the nurse correlate to this condition? a. Bence-Jones protein in urine b. Epstein-Barr virus: positive c. Hemoglobin: 18 mg/dL (180 mmol/L) d. Red blood cell count: 8.2 million/mcL (8.2 × 1012/L)

a This client has possible multiple myeloma. A positive Bence-Jones protein finding would correlate with this condition. The Epstein-Barr virus is a herpesvirus that causes infectious mononucleosis and some cancers. A hemoglobin of 18 mg/dL (180 mmol/L) is slightly high for a male and somewhat high for a female; this can be caused by several conditions, and further information would be needed to correlate this value with a specific medical condition. A red blood cell count of 8.2 million/mcL (8.2 × 1012/L) is also high, but again, more information would be needed to correlate this finding with a specific medical condition.

A client in the emergency department reports difficulty breathing. The nurse assesses the client's appearance as depicted below: *super veiny chest* What action by the nurse is most important? a. Assess blood pressure and pulse. b. Attach the client to a pulse oximeter. c. Have the client rate his or her pain. d. Facilitate urgent radiation therapy.

a This client has superior vena cava syndrome, in which venous return from the head, neck, and trunk is blocked. Decreased cardiac output can occur. The nurse would assess indicators of cardiac output, including blood pressure and pulse, as the priority. The other actions are also appropriate but are not as important. The ED nurse may or may not be able to facilitate radiation therapy.

A client with a history of prostate cancer is in the clinic and reports new onset of severe low back pain. What action by the nurse is most important? a. Assess the client's gait and balance. b. Ask the client about the ease of urine flow. c. Document the report completely. d. Inquire about the client's job risks.

a This client has symptoms of spinal cord compression, which can be seen with prostate cancer. This may affect both gait and balance and urinary function. For client safety, assessing gait and balance is most important. Documentation would be complete. The client may or may not have occupational risks for low back pain, but with his history of prostate cancer, this would not be where the nurse starts investigating.

A nurse is caring for a client admitted for Non-Hodgkin's lymphoma and chemotherapy. The client reports nausea, flank pain, and muscle cramps. What action by the nurse is most important? a. Request an order for serum electrolytes and uric acid. b. Increase the client's IV infusion rate. c. Instruct assistive personnel to strain all urine. d. Administer an IV antiemetic.

a This client's reports are consistent with tumor lysis syndrome, for which he or she is at risk due to the diagnosis. Early symptoms of TLS stem from electrolyte imbalances and can include lethargy, nausea, vomiting, anorexia, flank pain, muscle weakness, cramps, seizures, and altered mental status. The nurse would notify the primary health care provider and request an order for serum electrolytes. Hydration is important in both preventing and managing this syndrome, but the nurse would not just increase the IV rate. Assistive personnel may need to strain the client's urine and the client may need an antiemetic, but first the nurse would assess the situation further by obtaining pertinent lab tests.

The nurse observes a client with late-stage Alzheimer disease eat breakfast. Afterward the client states, "I am hungry and want breakfast." What is the nurse's best response? a. "I see you are still hungry. I will get you some toast." b. "You ate your breakfast 30 minutes ago." c. "It appears you are confused this morning." d. "Your family will be here soon. Let's get you dressed."

a Use of validation therapy with clients who have late-stage Alzheimer disease involves acknowledgment of the client's feelings and concerns. This technique has proved more effective in later stages of the disease because reality orientation only increases agitation. The other statements do not validate the client's concerns.

A client has a platelet count of 9000/mm3 (9 × 109 /L). The nurse finds the client confused and mumbling. What nursing action takes priority at this time? a. Call the Rapid Response Team. b. Take a set of vital signs. c. Institute bleeding precautions. d. Place the client on bedrest.

a With a platelet count this low, the client is at high risk of spontaneous bleeding. The most disastrous complication would be intracranial bleeding. The nurse needs to call the Rapid Response Team as this client has manifestations of a sudden neurologic change. Bleeding precautions will not address the immediate situation. Placing the client on bedrest is important, but the critical action is to call for immediate medical attention.

A client is receiving rituximab. What assessment by the nurse takes priority? a. Blood pressure b. Temperature c. Oral mucous membranes d. Pain

a pressure is the priority. Other complications of this drug include fever with chills/rigors, headache and abdominal pain, shortness of breath, bronchospasm, nausea and vomiting, and rash. Assessing the client's temperature and for pain are both pertinent assessments, but do not take priority over the blood pressure. Oral mucus membrane assessment is important for clients with cancer, but are not specific for this treatment.

The nurse assesses a client who has a mild traumatic brain injury (TBI) for signs and symptoms consistent with this injury. What signs and symptoms does the nurse expect? (Select all that apply.) a. Sensitivity to light and sound b. Reports "feeling foggy" c. Unconscious for an hour after injury d. Elevated temperature e. Widened pulse pressure

a b A mild TBI would possibly lead to sensitivity to light and sound and a feeling of mental fogginess. The patient would have been unconscious for less than 30 minutes. An elevated temperature is not related. A widened pulse pressure is indicative of increased intracranial pressure, not a mild TBI.

A nurse assesses a client who is recovering from an open traditional anterior cervical fusion. Which assessment findings would alert the nursing to a complication from this procedure? (Select all that apply.) a. Difficulty swallowing b. Hoarse voice c. Constipation d. Bradycardia e. Hypertension

a b Complications of the open traditional anterior cervical discectomy and fusion include dysphagia and hoarseness. Constipation, bradycardia, and hypertension are not complications of this procedure.

A nurse assesses a client with irritable bowel syndrome (IBS). Which questions would the nurse include in this client's assessment? (Select all that apply.) a. "Which food types cause an exacerbation of symptoms?" b. "Where is your pain or discomfort and what does it feel like?" c. "Have you lost a significant amount of weight lately?" d. "Are your stools soft, watery, and black?" e. "Do you often experience nausea and vomiting"

a b The nurse would ask the client about factors that may cause exacerbations of symptoms, including food, stress, and anxiety. The nurse would also assess the location, intensity, and quality of the patient's pain or discomfort. Clients who have IBS do not usually lose weight, have nausea and vomiting, or have stools that are black.

The nurse would recognize which signs and symptoms as consistent with brainstem tumors? (Select all that apply.) a. Hearing loss b. Facial pain c. Nystagmus d. Vomiting e. Hemiparesis

a b c Hearing loss (CN VIII), facial pain (CN V), and nystagmus (CN III, IV, and VI) all are indicative of a brainstem tumor because these cranial nerves originate in the brainstem. Vomiting and hemiparesis are more indicative of cerebral tumors.

The nurse assesses clients on a medical-surgical unit. Which clients would the nurse identify as at risk for secondary seizures? (Select all that apply.) a. A 26-year-old woman with a left temporal brain tumor b. A 38-year-old male client in an alcohol withdrawal program c. A 42-year-old football player with a traumatic brain injury d. A 66-year-old female client with multiple sclerosis e. A 72-year-old man with chronic obstructive pulmonary disease

a b c Clients at risk for secondary seizures include those with a brain lesion from a tumor or trauma, and those who are experiencing a metabolic disorder, acute alcohol withdrawal, electrolyte disturbances, and high fever. Clients with a history of stroke, heart disease, and substance abuse are also at risk. Clients with multiple sclerosis or chronic obstructive pulmonary disease are not at risk for secondary seizures.

A client's family members are concerned that telling the client about a new finding of cancer will cause extreme emotional distress. They approach the nurse and ask if this can be kept from the client. What actions by the nurse are most appropriate? (Select all that apply.) a. Ask the family to describe their concerns more fully. b. Consult with a social worker, chaplain, or ethics committee. c. Explain the client's right to know and ask for their assistance. d. Have the unit manager take over the care of this client and family. e. Tell the family that this secret will not be kept from the client.

a b c The client's right of autonomy means that the client must be fully informed as to his or her diagnosis and treatment options. The nurse cannot ethically keep this information from the client. The nurse can ask the family to explain their concerns more fully so everyone understands them. A social worker, chaplain, or ethics committee can become involved to assist the nurse, client, and family. The nurse would explain the client's right to know and ask the family how best to proceed. Enlisting their help might reduce their reluctance for the client to be informed. The nurse would not abdicate responsibility for this difficult situation by transferring care to another nurse. Simply telling the family that he or she will not keep this secret sets up an adversarial relationship. Explaining this fact along with the concept of autonomy would be acceptable, but this by itself is not.

The nurse working with oncology clients understands that interacting factors affect cancer development. Which factors does this include? (Select all that apply.) a. Exposure to carcinogens b. Genetic predisposition c. Immune function d. Normal doubling time e. State of euploidy

a b c The three interacting factors needed for cancer development are exposure to carcinogens, genetic predisposition, and immune function.

The nurse assesses a client who has meningitis. Which sign(s) and symptom(s) would the nurse anticipate? (Select all that apply.) a. Photophobia b. Decreased level of consciousness c. Severe headache d. Fever and chills e. Bradycardia

a b c d All of the choices except for bradycardia are key features of meningitis. Tachycardia is more likely than bradycardia due to the infectious process and fever.

The nurse is assessing a client with chronic leukemia. Which laboratory test result(s) is (are) expected for this client? (Select all that apply.) a. Decreased hematocrit b. Abnormal white blood cell count c. Low platelet count d. Decreased hemoglobin e. Increased albumin

a b c d Chronic leukemia affects all types of blood cells causing a decrease is red blood cells (RBCs) and platelets. When the number of RBCs decreases, the client's hemoglobin and hematocrit also decrease. White blood cell counts are also abnormal depending on disease progression and management.

The nurse is caring for a client being treated for Hodgkin lymphoma. For which side effect(s) of treatment will the nurse assess? (Select all that apply.) a. Severe nausea and vomiting b. Low platelet count c. Skin irritation at radiation site d. Low red blood cell count e. High white blood cell count

a b c d Drug and radiation therapy for Hodgkin lymphoma cause many side and adverse effects, including all of the choices except for a high white blood cell (WBC) count. Instead, most clients experience a low WBC count making them very susceptible to infections.

The nurse is teaching a client about risk factors for esophageal cancer. Which risk factors would the nurse include? (Select all that apply.) a. Alcohol intake b. Obesity c. Smoking d. Lack of fresh fruits and vegetables e. Untreated GERD f. Use of NSAIDs

a b c d e all of these factors increase the risk of esophageal cancer except for the use of NSAIDs. Untreated GERD causes damage to esophageal tissue which may develop into Barrett esophagus, or precancerous cells.

The nurse is taking a history on an older adult. Which factors would the nurse assess as potential risks for low back pain? (Select all that apply.) a. Scoliosis b. Spinal stenosis c. Hypocalcemia d. Osteoporosis e. Osteoarthritis

a b c d e All of these factors place the client at risk for low back pain due to changes in spinal alignment, loss of bone, or joint degeneration. Bone loss worsens if serum calcium levels are below normal.

The nurse is teaching a client about the risk of uncontrolled or untreated the client's gastroesophageal reflux disease (GERD). What complication(s) may occur if the GERD is not successfully managed? (Select all that apply.) a. Asthma b. Laryngitis c. Dental caries d. Cardiac disease e. Cancer

a b c d e Any of these complications may occur in clients who have uncontrolled or untreated GERD.

A nurse is caring for a group of stroke patients. Which clients would the nurse consider referring to a mental health provider? (Select all that apply.) a. Female client who exhibits extreme emotional lability b. Male client with an initial National Institutes of Health (NIH) Stroke Scale score of 38 c. Female client with mild forgetfulness and a history of depression d. Male client who has a past hospitalization for a suicide attempt e. Male client who is unable to walk or eat 3 weeks poststroke

a b c d e Patients most at risk for poststroke depression are those with a previous history of depression, severe stroke (NIH Stroke Scale score of 38 is severe), and poststroke physical or cognitive impairment.

The nurse is caring for a client who has possible acute pancreatitis. What serum laboratory findings would the nurse expect for this client? (Select all that apply.) a. Elevated amylase b. Elevated lipase c. Elevated glucose d. Decreased calcium e. Elevated bilirubin f. Elevated leukocyte count

a b c d e f All of these choices are correct. Amylase and lipase are pancreatic enzymes that are released during pancreatic inflammation and injury. Leukocytes also increased due to his inflammatory response. Pancreatic injury affects the ability of insulin to be released causing increased glucose levels. Bilirubin is also typically increased due to hepatobiliary obstruction. Calcium and magnesium levels decrease because fatty acids bind free calcium and magnesium causing a lowered serum level; these changes occur in the presence of fat necrosis.

The nurse is caring for a client who just had a minimally invasive inguinal hernia repair. Which nursing actions would the nurse implement? (Select all that apply.) a. Apply ice to the surgical area for the first 24 hours after surgery. b. Encourage ambulation with assistance within the first few hours after surgery. c. Encourage deep breathing after surgery but teach the client to avoid coughing. d. Assess vital signs frequently for the first few hours after surgery. e. Teach the client to rest for several days after surgery when at home. f. Teach the client not to lift more than 10 lb (4.5 kg) until allowed by the surgeon.

a b c d e f All of these nursing actions are appropriate for the client having MIS for inguinal hernia repair.

Which assessment finding(s) may indicate that a client may be experiencing a blood transfusion reaction? (Select all that apply.) a. Tachycardia b. Fever c. Bronchospasm d. Tachypnea e. Urticaria f. Hypotension

a b c d e f Several types of blood transfusion reactions can occur and cause all of the findings listed.

The nurse is assessing a client with hepatitis C. The client asks the nurse how it was possible to have this disease. What questions might the nurse ask to help the client determine how the disease was contracted? (Select all that apply.) a. "How old are you?" b. "Do you work in health care? c. "Are you receiving hemodialysis?" d. "Do you use IV drugs?" e. "Did you receive blood before 1992?" f. "Have you even been in prison or jail?"

a b c d e f The nurse would ask all of these questions because "baby boomers," people who use illicit drugs, people on hemodialysis, health workers, and prisoners are at a very high risk for hepatitis C. Additionally, individuals who received blood, blood products, or an organ transplant prior to 1992 before bloodborne disease screening of these products was mandated are at risk for hepatitis c

The nurse is caring for a client who has late-stage (advanced) cirrhosis. What assessment findings would the nurse expect? (Select all that apply.) a. Jaundice b. Clay-colored stools c. Icterus d. Ascites e. Petechiae f. Dark urine

a b c d e f All of these assessment findings are very common for a client who has late-stage cirrhosis due to biliary obstruction and poor liver function. The client has vascular lesions and excess fluid from portal hypertension.

The nurse is caring for a client diagnosed with probable gastroesophageal reflux disease (GERD). What assessment finding(s) would the nurse expect? (Select all that apply.) a. Dyspepsia b. Regurgitation c. Belching d. Coughing e. Chest discomfort f. Dysphagia

a b c d e f All of these signs and symptoms are commonly seen in clients who have GERD.

The nurse caring for clients who have cancer understands that the general consequences of cancer include which client problems? (Select all that apply.) a. Clotting abnormalities from thrombocythemia b. Increased risk of infection from white blood cell deficits c. Nutritional deficits such as early satiety and cachexia d. Potential for reduced gas exchange e. Various motor and sensory deficits f. Increased risk of bone fractures

a b c d e f The general consequences of cancer include reduced immunity and blood-producing functions, altered GI structure and function, decreased respiratory function, and motor and sensory deficits. Clotting problems often occur due to thrombocytopenia (not enough platelets), not thrombocythemia (too many platelets).

The nurse is assessing a client who has symptoms of stroke. What are the leading causes of a stroke for which the nurse would assess for this client? (Select all that apply.) a. Heavy alcohol intake b. Diabetes mellitus c. Elevated cholesterol d. Obesity e. Smoking f. Hypertension

a b c d e f The leading causes of stroke include all of these factors.

A nurse prepares to discharge a client who is newly diagnosed with a chronic inflammatory bowel disease. Which questions would the nurse ask in preparation for discharge? (Select all that apply.) a. Does your gym provide yoga classes? b. When should you contact your provider? c. What do you plan to eat for dinner? d. Do you have a scale for daily weights? e. How many bathrooms are in your home?

a b c e A home assessment for a client who has a chronic inflammatory bowel disease would include identifying adequacy and availability of bathroom facilities, opportunities for rest and relaxation, and the client's knowledge of dietary therapy, and when to contact the primary health care provider. The client does not need to perform daily weights.

Which of the following is (are) (a) risk factor(s) for gastric cancer? (Select all that apply.) a. Achlorhydria b. Chronic atrophic gastritis c. H. pylori infection d. Iron deficiency anemia e. Pernicious anemia

a b c e Achlorhydria, chronic atrophic gastritis, H. pylori infection, and pernicious anemia are all risk factors for developing gastric cancer. Iron deficiency anemia is not a risk factor.

The nurse assists the wound care/ostomy nurse assess a client prior to ostomy surgery. Which assessments would the nurse complete before marking the placement for the ostomy? (Select all that apply.) a. Contour of the abdomen when standing b. Location of the client's belt line c. Contour of the abdomen when lying d. Location of abdominal muscles e. Contour of the abdomen when sitting

a b c e Before marking the placement for the ostomy, the nurse would consider the contour of the abdomen in lying, sitting, and standing positions, the location of the belt line and possible location in the rectus muscle. The location of abdominal muscles is not considered.

The nurse recalls that the risk factors for acute gastritis include which of the following? (Select all that apply.) a. Alcohol b. Caffeine c. Corticosteroids d. Fruit juice e. Nonsteroidal anti-inflammatory drugs (NSAIDs)

a b c e Risk factors for acute gastritis include alcohol, caffeine, corticosteroids, and chronic NSAID use. Fruit juice is not a risk factor, although in some people it does cause distress.

The nurse is caring for a client with increasing intracranial pressure (ICP) following a stroke. Which evidence-based nursing actions are indicated for this client? (Select all that apply.) a. Hyperoxygenate the client before and after suctioning. b. Avoid sudden or extreme hip or neck flexion. c. Provide oxygen to maintain an SaO2 of 95% or greater. d. Maintain the client in a supine position at all times. e. Avoid clustering care nursing activities and procedures. f. Provide environmental stimulation to improve cognition.

a b c e These precautions help prevent further increases in ICP. Clustering nursing activities and procedures and providing stimulation can increase ICP and should be avoided.

The nurse is preparing a client who has chronic pancreatitis about how to prevent exacerbations of the disease. Which health teaching will the nurse include? (Select all that apply.) a. "Avoid alcohol ingestion." b. "Be sure and balance rest with activity." c. "Avoid caffeinated beverages." d. "Avoid green, leafy vegetables." e. "Eat small meals and high-calorie snacks."

a b c e Clients who have chronic pancreatitis need to avoid GI stimulants, including alcohol, caffeine, and nicotine. Food and snacks need to be high-calorie to prevent additional weight loss. Green vegetables can be consumed if tolerated by the client.

Based on the known risk factors for stroke, which health promotion practices would the nurse teach a client to promote heart health and prevent strokes? (Select all that apply.) a. Blood pressure control b. Aspirin use c. Smoking cessation d. Low carbohydrate diet e. Cholesterol management f. Increased red wine consumption

a b c e The evidence-based health promotion practices include blood pressure control, aspirin use, smoking cessation, and cholesterol management. There is no consensus on which diet is best to promote heart health and red wine does not protect the heart or prevent strokes.

A nurse cares for an older adult who is admitted to the hospital with complications of diverticulitis. Which actions would the nurse include in the client's plan of care? (Select all that apply.) a. Administer pain medications as prescribed. b. Palpate the abdomen for distention. c. Assess for sudden changes in mental status. d. Provide the client with a high-fiber diet. e. Evaluate stools for occult blood

a b c e When caring for an older adult who has diverticulitis, the nurse would administer analgesics as prescribed, palpate the abdomen for distention and tenderness, assess for confusion and sudden changes in mental status, and check stools for occult or frank bleeding. A low-fiber/residue diet would be provided when symptoms are present and a high-fiber diet when inflammation resolves.

The nurse is caring for a client who was recently diagnosed with pancreatic cancer. What factors present risks for developing this type of cancer? (Select all that apply.) a. Diabetes mellitus b. Cirrhosis c. Smoking d. Female gender e. Family history f. Older age

a b c e f All of these choices are risk factors except that pancreatic cancer occurs most frequently in men.

The nurse is preparing for discharge of a client who had a carotid artery angioplasty with stenting to prevent a stroke. For which signs and symptoms with the nurse teach the family to report to the primary health care provider immediately? (Select all that apply.) a. Muscle weakness b. Hoarseness c. Acute confusion d. Mild neck discomfort e. Severe headache f. Dysphagia

a b c e f Muscle weakness, acute confusion, severe headache, and dysphagia are all signs and symptoms that could indicate that a stroke occurred. Hoarseness and severe neck pain and swelling may occur as a result of the interventional radiologic procedure.

A nurse is providing community education on the seven warning signs of cancer. Which signs are included? (Select all that apply.) a. A sore that does not heal b. Changes in menstrual patterns c. Indigestion or trouble swallowing d. Near-daily abdominal pain e. Obvious change in a mole f. Frequent indigestion

a b c e f The seven warning signs for cancer can be remembered with the acronym CAUTION: changes in bowel or bladder habits, a sore that does not heal, unusual bleeding or discharge, thickening or lump in the breast or elsewhere, indigestion or difficulty swallowing, obvious change in a wart or mole, and nagging cough or hoarseness. Abdominal pain is not a warning sign.

A nurse is preparing to administer IV chemotherapy. What supplies does this nurse need? (Select all that apply.) a. "Chemo" gloves b. Face mask c. Impervious gown d. N95 respirator e. Shoe covers f. Eye protection

a b c f The Occupational Safety and Health Administration (OSHA) and the Oncology Nurses Society have developed safety guidelines for those preparing or administering IV chemotherapy. These include double gloves (or "chemo" gloves), eye protection, a face mask, and a gown. An N95 respirator and shoe covers are not required.

A client has thrombocytopenia. What actions does the nurse delegate to assistive personnel (AP)? (Select all that apply.) a. Apply the client's shoes before getting the client out of bed. b. Assist the client with ambulation. c. Shave the client with a safety razor only. d. Use a lift sheet to move the client up in bed. e. Use a water pressure device be set on low for oral care.

a b d Clients with thrombocytopenia are at risk of significant bleeding even with minor injuries. The nurse instructs the AP to put the client's shoes on before getting the client out of bed, assist with ambulation, shave the client with an electric razor, use a lift sheet when needed to reposition the client, and use a soft-bristled toothbrush for oral care. All of these measures help prevent client injury.

The nurse is preparing to administer a blood transfusion. Which action(s) by the nurse is (are) most appropriate? (Select all that apply.) a. Hang the blood product using normal saline and a filtered tubing set b. Take a full set of vital signs prior to starting the blood transfusion. c. Tell the client that someone will remain at the bedside for the first 5 minutes. d. Use gloves to start the client's IV if needed and to handle the blood product. e. Verify the client's identity, and checking blood compatibility and expiration time.

a b d Correct actions prior to beginning a blood transfusion include hanging the product with saline and the correct filtered blood tubing, taking a full set of vital signs prior to starting, and using gloves. Someone stays with the client for the first 15 to 20 minutes of the transfusion. Two registered nurses must verify the client's identity and blood compatibility.

The nurse assesses a client with ulcerative colitis. Which complications are paired correctly with their physiologic processes? (Select all that apply.) a. Lower gastrointestinal bleeding—erosion of the bowel wall b. Abscess formation—localized pockets of infection develop in the ulcerated bowel lining c. Toxic megacolon—transmural inflammation resulting in pyuria and fecaluria d. Nonmechanical bowel obstruction—paralysis of colon resulting from colorectal cancer e. Fistula—dilation and colonic ileus caused by paralysis of the colon

a b d Lower GI bleeding can lead to erosion of the bowel wall. Abscesses are localized pockets of infection that develop in the ulcerated bowel lining. Nonmechanical bowel obstruction is paralysis of the colon that results from colorectal cancer. When the inflammation is transmural, fistulas can occur between the bowel and bladder resulting in pyuria and fecaluria. Paralysis of the colon causing dilation and subsequent colonic ileus is known as a toxic megacolon.

After teaching a patient who has a permanent ileostomy, a nurse assesses the client's understanding. Which dietary items chosen for dinner indicate that the client needs further teaching? (Select all that apply.) a. Corn b. String beans c. Carrots d. Wheat rice e. Squash

a b d Clients with an ileostomy should be cautious of high-fiber and high-cellulose foods including corn, string beans, and rice. Carrots and squash are low-fiber items.

The nurse is caring for a client who is diagnosed with celiac disease and preparing to start natalizumab. Which health teaching would the nurse include in the teaching? (Select all that apply.) a. Need to have drug administered by a primary health care provider. b. Need to avoid crowds and individuals who have infection. c. Need to report injection reactions such as redness and swelling. d. Awareness of a rare but potentially fatal drug complication. e. Need to report any signs and symptoms of infection immediately.

a b d e All of these choices are correct except that the drug is given intravenously. Therefore, there is no need to teach the client to report injection reactions because the client does not self-administer the medication subcutaneously. Natalizumab can cause progressive multifocal leukoencephalopathy (PML), but it is a very rare disorder causing cognitive, sensory, and/or motor changes.

The nurse is caring for a client receiving a unit of whole blood. Which nursing action(s) is (are) appropriate regarding infusion administration. (Select all that apply.) a. Use a dedicated filtered blood administration set. b. Stay with the client for the first 15 to 20 minutes of the infusion. c. Infuse the blood over a 30-minute period of time. d. Monitor and document vital signs per agency policy. e. Use a 21-gauge or smaller catheter to administer the blood. f. Infuse the transfusion with intravenous normal saline.

a b d f Blood administration requires a dedicated and filtered intravenous set and a larger catheter or needle due to the viscosity of the infusion. Normal saline is the only IV fluid that is compatible with blood. Vital signs are frequently monitored and documented while the client is carefully assesses for signs and symptoms of a blood transfusion reaction, usually within the first 15 to 20 minutes. One unit of blood is administered in no less than 60 minutes.

The nurse is caring for a client with early encephalopathy due to cirrhosis of the liver. Which factors may contribute to increased encephalopathy for which the nurse would assess? (Select all that apply.) a. Infection b. GI bleeding c. Irritable bowel syndrome d. Constipation e. Anemia f. Hypovolemia

a b d f Anemia and irritable bowel syndrome are unrelated to developing or worsening encephalopathy, which is caused by increased protein which breaks down into ammonia. Infection can cause hypovolemia which would increase serum protein concentration. Constipation and GI bleeding causes a large protein load in the intestines.

A nurse is caring for an older adult receiving multiple packed red blood cell transfusions. Which assessment finding(s) indicate(s) possible transfusion circulatory overload? (Select all that apply.) a. Acute confusion b. Dyspnea c. Depression d. Hypertension e. Bradycardia f. Bounding pulse

a b d f Circulatory overload is the result of excessive body fluid which can cause signs and symptoms of heart failure including dyspnea, increased blood pressure, tachycardia (not bradycardia), and a bounding pulse. Dyspnea is caused by hypoxia which in older adults can cause acute confusion. Depression is not a common finding resulting from fluid overload.

A client has mucositis. What actions by the nurse will improve the client's nutrition? (Select all that apply.) a. Assist with rinsing the mouth with saline frequently. b. Encourage the client to eat room-temperature foods. c. Give the client hot liquids to hold in the mouth. d. Provide local anesthetic medications to swish and spit. e. Remind the client to brush teeth gently after each meal. f. Offer the client fluids to drink each hour.

a b d f Mucositis can interfere with nutrition. The nurse can help with rinsing the mouth frequently with water or saline; encouraging the client to eat cool, slightly warm, or room-temperature foods; providing swish-and-spit anesthetics; and reminding the client to keep the mouth clean by brushing gently after each meal. Drinking plenty of fluids (unless contraindicated for another condition) is another beneficial measure. Hot liquids would be painful for the client.

What action(s) by the nurse is (are) appropriate to promote nutrition in a client who had a partial gastrectomy? (Select all that apply.) a. Administer vitamin B12 injections. b. Ask the primary health care provider about folic acid replacement. c. Educate the client on enteral feedings. d. Obtain consent for total parenteral nutrition. e. Provide iron supplements for the client.

a b e After a partial or total gastrectomy, clients are at high risk for anemia due to vitamin B12 deficiency, folic acid deficiency, or iron deficiency. The nurse would provide supplements for all these nutrients. The client does not need enteral feeding or total parenteral nutrition.

A nurse is preparing to administer pantoprazole intravenously to prevent stress ulcers during surgery. What action(s) by the nurse is (are) most appropriate? (Select all that apply.) a. Administer the drug through a separate IV line. b. Infuse pantoprazole using an IV pump. c. Keep the drug in its original brown container. d. Take vital signs frequently during infusion. e. Use an in-line IV filter when infusing.

a b e When infusing pantoprazole, use a separate IV line, a pump, and an in-line filter. A brown wrapper and frequent vital signs are not needed.

The nurse assesses a client who is experiencing a common migraine without an aura. Which assessment finding(s) would the nurse expect? (Select all that apply.) a. Headache lasting up to 72 hours b. Unilateral and pulsating headache c. Abrupt loss of consciousness d. Acute confusion e. Pain worsens with physical activities f. Photophobia

a b e f A common migraine with an aura is usually accompanied by photophobia, phonophobia, unilateral and pulsating pain, and nausea and/or vomiting. These migraines usually last 4 to 72 hours and are aggravated by physical activity. Loss of consciousness and acute confusion are not associated with a common migrane without an aura

A nurse is preparing to administer a packed red blood cell transfusion to an older adult. Understanding age-related changes, what alteration(s) in the usual protocol is (are) necessary for the nurse to implement? (Select all that apply.) a. Assess vital signs at least every 15 minutes. b. Avoid giving other IV fluids. c. Premedicate to prevent transfusion reaction. d. Transfuse smaller bags of blood. e. Transfuse each unit over 8 hours. f. Assess the client for fluid overload.

a b f The older adult needs vital signs monitored as often as every 15 minutes for the duration of the transfusion because vital sign changes may be the only indication of a transfusion-related problem. To prevent fluid overload, the nurse obtains a prescription to hold other running IV fluids during the transfusion and assesses the client frequently for signs and symptoms of overload. The other options are not correct.

The nurse is caring for a client with sialadenitis. What comfort measures are appropriate for this client? (Select all that apply.) a. Applying warm compresses b. Applying ice to salivary glands c. Offering fluids every hour d. Providing lemon-glycerin swabs e. Reminding the patient to avoid speaking

a c Warm compresses and fluids can help promote comfort for this client. Application of ice or lemon-glycerin swabs would not be used. Speaking has no effect on this condition.

The nurse assesses a client who has Parkinson disease. Which signs and symptoms would the nurse recognize as a key feature of this disease? (Select all that apply.) a. Flexed trunk b. Long, extended steps c. Slow movements d. Uncontrolled drooling e. Tachycardia

a c d Key features of Parkinson disease include a flexed trunk, slow and hesitant steps, bradykinesia, and uncontrolled drooling. Tachycardia is not a key feature of this disease.

A nurse cares for older clients who have traumatic brain injury. What does the nurse understand about this population? (Select all that apply.) a. Admission can overwhelm the coping mechanisms for older clients. b. Alcohol is typically involved in most traumatic brain injuries for this age-group. c. These clients are more susceptible to systemic and wound infections. d. Other medical conditions can complicate treatment for these clients. e. Very few traumatic brain injuries occur in this age-group.

a c d Older adults often tolerate stress poorly, which includes being admitted to a hospital that is unfamiliar and noisy. Because of decreased protective mechanisms, they are more susceptible to both local and systemic infections. Other medical conditions can complicate their treatment and recovery. Alcohol is typically not related to traumatic brain injury in this population; such injury is most often from falls and motor vehicle crashes.

A nurse teaches the spouse of a client who has Alzheimer disease. Which statements should the nurse include in this teaching related to caregiver stress reduction? (Select all that apply.) a. "Establish advanced directives early." b. "Trust that family and friends will help." c. "Set aside time each day to be away from the client." d. "Use discipline to correct inappropriate behaviors." e. "Seek respite care periodically for longer periods of time."

a c d To reduce caregiver stress, the spouse should be encouraged to establish advanced directives early, set aside time each day for rest or recreation away from the client, seek respite care periodically for longer periods of time, use humor with the client, and explore alternative care settings and resources. Family and friends may not be available to help. A structured environment will assist the client with AD, but discipline will not correct inappropriate behaviors and not reduce caregiver stress.

The nurse is teaching assistive personnel (AP) about care of a client who has advanced cirrhosis. Which statements would the nurse include in the staff teaching? (Select all that apply.) a. "Apply lotion to the client's dry skin areas." b. "Use a basin with warm water to bathe the patient." c. "For the patient's oral care, use a soft toothbrush." d. "Provide clippers so the patient can trim the fingernails." e. "Bathe with antibacterial and water-based soaps."

a c d Clients with advanced cirrhosis often have pruritus. Lotion will help decrease itchiness from dry skin. A soft toothbrush would be used to prevent gum bleeding, and the client's nails would need to be trimmed short to prevent the patient from scratching himself or herself. These clients should use cool, not warm, water on their skin, and should not use excessive amounts of soap.

A nurse assesses a client who experienced a spinal cord injury at the T5 level 12 hours ago. Which assessment findings would the nurse correlate with neurogenic shock? (Select all that apply.) a. Heart rate of 34 beats/min b. Blood pressure of 185/65 mm Hg c. Urine output less than 30 mL/hr d. Decreased level of consciousness e. Increased oxygen saturation

a c d Neurogenic shock with acute spinal cord injury manifests with decreased oxygen saturation, symptomatic bradycardia, decreased level of consciousness, decreased urine output, and hypotension.

A client receiving chemotherapy has a white blood cell count of 1000/mm3 (1 × 109 /L). What actions by the nurse are most appropriate? (Select all that apply.) a. Assess all mucous membranes every 4 to 8 hours. b. Do not allow the client to eat meat or poultry. c. Listen to lung sounds and monitor for cough. d. Monitor the venous access device appearance hourly. e. Take and record vital signs every 4 to 8 hours. f. Encourage activity the client can tolerate.

a c d e Depending on facility protocol, the nurse would assess this client for infection every 4 to 8 hours by assessing all mucous membranes, listening to lung sounds, monitoring for cough, monitoring the appearance of the venous access device, and recording vital signs. Assisting the client with mobilization will also help prevent infection. Eating meat and poultry is allowed.

A nurse working with clients diagnosed with sickle cell disease (SCD) teaches about self-management to prevent exacerbations and sickle cell crises. What factor(s) should clients be taught to avoid? (Select all that apply.) a. Dehydration b. Exercise c. Extreme stress d. High altitudes e. Pregnancy

a c d e Several factors cause red blood cells to sickle in SCD, including dehydration, extreme stress, high altitudes, and pregnancy. Strenuous exercise can also cause sickling, but not unless it is very vigorous.

A nurse knows that job-related risks for developing oral cancer include which occupations? (Select all that apply.) a. Coal miner b. Electrician c. Metal worker d. Plumber e. Textile worker

a c d e The occupations of coal mining, metal working, plumbing, and textile work produce exposure to polycyclic aromatic hydrocarbons (PAHs), which are known carcinogens. Electricians do not have this risk.

The nurse is caring for a client with peritonitis. What assessment findings would the nurse expect? (Select all that apply.) a. Nausea and vomiting b. Distended rigid abdomen c. Abdominal pain d. Bradycardia e. Decreased urinary output f. Fever

a c d e f Peritonitis is an acute inflammatory disorder. Therefore, the client would likely have all of these signs and symptoms but would have tachycardia rather than bradycardia due to dehydration from fever.

The nurse assesses a client who has chronic pancreatitis. What assessment findings would the nurse expect for this client? (Select all that apply.) a. Ascites b. Weight gain c. Steatorrhea d. Jaundice e. Polydipsia f. Polyuria

a c d e f The client who has chronic pancreatitis has all of these signs and symptoms except he or she loses weight. Ascites and jaundice result from biliary obstruction; ascites is associated with portal hypertension. Steatorrhea is fatty stool that occurs because lipase is not available in the duodenum; because it is released by the disease pancreas into the bloodstream. Polydipsia, polyuria, and polyphagia result from diabetes mellitus, a common problem seen in clients whose pancreas is unable to release adequate amounts of insulin.

The nurse plans care for a patient who has hepatopulmonary syndrome. Which interventions would the nurse include in this client's plan of care? (Select all that apply.) a. Oxygen therapy b. Prone position c. Feet elevated on pillows d. Daily weights e. Physical therapy f. Respiratory therapy

a c d f Care for a client who has hepatopulmonary syndrome would include oxygen therapy, the head of bed elevated at least 30 degrees or as high as the client wants to improve breathing, elevated feet to decrease dependent edema, and daily weights. There is no need to place the patient in a prone position, on the patient's stomach. Although physical therapy may be helpful to a patient who has been hospitalized for several days, physical therapy is not an intervention specifically for hepatopulmonary syndrome. However, respiratory support from a specialized therapist may be needed.

A nurse assesses a client who recently experienced a traumatic spinal cord injury. Which assessment data would the nurse obtain to assess the client's coping strategies? (Select all that apply.) a. Spiritual beliefs b. Level of pain c. Family support d. Level of independence e. Annual income f. Previous coping strategies

a c d f Information about the client's preinjury psychosocial status, usual methods of coping with illness, difficult situations, and disappointments would be obtained. Determine the client's level of independence or dependence and his or her comfort level in discussing feelings and emotions with family members or close friends. Clients who are emotionally secure and have a positive self-image, a supportive family, and financial and job security often adapt to their injury. Information about the client's spiritual and religious beliefs or cultural background also assists the nurse in developing the plan of care. The other options do not supply as much information about coping.

A client receiving radiation therapy reports severe skin itching and irritation. What actions does the nurse delegate to assistive personnel (AP)? (Select all that apply.) a. Apply approved moisturizers to dry skin. b. Apply steroid creams to the skin. c. Bathe the client using mild soap. d. Help the client pat skin dry after a bath. e. Teach the client to avoid sunlight. f. Make sure no clothing is rubbing the site.

a c d f The nurse can delegate applying moisturizer approved by the radiation oncologist using mild soap for bathing, and helping the client pat wet skin dry after bathing. Any clothing worn over the site should be soft and not create friction. Steroid creams are not used for this condition. Hot water will worsen the irritation. Client teaching is a nursing function.

A nurse promotes the prevention of lower back pain by teaching clients at a community center. Which statement(s) would the nurse include in this education? (Select all that apply.) a. "Participate in an exercise program to strengthen back muscles." b. "Purchase a mattress that allows you to adjust the firmness." c. "Wear flat instead of high-heeled shoes to work each day." d. "Keep your weight within 20% of your ideal body weight." e. "Avoid prolonged standing or sitting, including driving."

a c e Exercise can strengthen back muscles, reducing the incidence of low back pain. Women should avoid wearing high-heeled shoes because they cause misalignment of the back. Prolonged standing and sitting should also be avoided. The other options will not prevent low back pain.

The nurse is caring for a client who has Alzheimer disease. The client's wife states, "I am having trouble managing his behaviors at home." Which questions would the nurse ask to assess potential causes of the client's behavior problems? (Select all that apply.) a. "Does your husband bathe and dress himself independently?" b. "Do you weigh your husband each morning around the same time?" c. "Does his behavior become worse around large crowds?" d. "Does your husband eat healthy foods including fruits and vegetables?" e. "Do you have a clock and calendar in the bedroom and kitchen?"

a c e To minimize behavior problems, the nurse would encourage the patient to be as independent as possible with ADLs, minimize excessive simulation, and assist the patient to remain orientated. The nurse would assess these activities by asking if the patient is independent with bathing and dressing, if behavior worsens around crowds, and if a clock and single-date calendar are readily available. Diet and weight are not related to the management of behavior problems for a patient who has Alzheimer disease.

Which risk factor(s) places a client at risk for leukemia? (Select all that apply.) a. Chemical exposure b. Genetically modified foods c. Ionizing radiation exposure d. Vaccinations e. Viral infections

a c e Chemical and ionizing radiation exposure and viral infections are known risk factors for developing leukemia. Eating genetically modified food and receiving vaccinations are not known risk factors.

The nurse plans care for a client who has acute pancreatitis and is prescribed nothing by mouth (NPO). With which health care team members would the nurse collaborate to provide appropriate nutrition to this client? (Select all that apply.) a. Registered dietitian nutritionist b. Nursing assistant c. Clinical pharmacist d. Certified herbalist e. Primary health care provider

a c e Clients who are prescribed NPO while experiencing an acute pancreatitis episode may need enteral or parenteral nutrition. The nurse would collaborate with the registered dietitian nutritionist, clinical pharmacist, and primary health care provider to plan and implement the more appropriate nutritional interventions. The nursing assistant and certified herbalist would not assist with this clinical decision.

A nurse is caring for a client who has been diagnosed with a small bowel obstruction. Which assessment findings would the nurse correlate with this diagnosis? (Select all that apply.) a. Serum potassium of 2.8 mEq/L (2.8 mmol/L) b. Loss of 15 lb (6.8 kg) without dieting c. Abdominal pain in upper quadrants d. Low-pitched bowel sounds e. Serum sodium of 121 mEq/L (121 mmol/L)

a c e Small bowel obstructions often lead to severe fluid and electrolyte imbalances. The client is hypokalemic (normal range is 3.5 to 5.0 mEq/L [3.5 to 5.0 mmol/L]) and hyponatremic (normal range is 136 to 145 mEq/L [136 to 145 mmol/L]). Abdominal pain across the upper quadrants is associated with small bowel obstruction. Dramatic weight loss without dieting followed by bowel obstruction leads to the probable development of colon cancer. High-pitched sounds may be noted with small bowel obstructions.

Which statement(s) about blood transfusion compatibilities is (are) correct? (Select all that apply.) a. Donor blood type A can donate to recipient blood type AB. b. Donor blood type B can donate to recipient blood type O. c. Donor blood type AB can donate to anyone. d. Donor blood type O can donate to anyone. e. Donor blood type A can donate to recipient blood type B.

a d Blood type A can be donated to people who have blood types A or AB. Blood type O can be given to anyone. Blood type B can be donated to people who have blood types B or AB. Blood type AB can only go to recipients with blood type AB.

A client has dumping syndrome. What menu selections indicate the client understands the correct diet to manage this condition? (Select all that apply.) a. Apricots b. Coffee cake c. Milk shake d. Potato soup e. Steamed broccoli

a d Canned apricots and potato soup are appropriate selections as they are part of a high-protein, high-fat, and low- to moderate-carbohydrate diet. Coffee cake and other sweets must be avoided. Milk products and sweet drinks such as shakes must be avoided. Gas-forming foods such as broccoli must also be avoided.

The nurse teaches a community group ways to prevent Escherichia coli infection. Which statements would the nurse include in this group's teaching? (Select all that apply.) a. "Wash your hands after any contact with animals." b. "It is not necessary to buy a meat thermometer." c. "Stay away from people who are ill with diarrhea." d. "Use separate cutting boards for meat and vegetables." e. "Avoid swimming in backyard pools and using hot tubs."

a d Washing hands after contact with animals and using separate cutting boards for meat and other foods will help prevent E. coli infection. The other statements are not related to preventing E. coli infection.

The nurse is caring for a client who has a nasogastric tube (NGT). Which actions would the nurse take for client care? (Select all that apply.) a. Assess for proper placement of the tube every 4 hours or per agency policy. b. Flush the tube with water every hour to ensure patency. c. Secure the NG tube to the client's chin. d. Disconnect suction when auscultating bowel peristalsis. e. Monitor the client's skin around the tube site for irritation.

a d e The nurse would frequently assess for NGT placement, patency, and output (drainage) every 4 hours or per agency policy. The nurse would also monitor the skin around the tube for irritation and secure the tube to the client's nose. When auscultating bowel sounds for peristalsis, the nurse would disconnect suction. NGT irrigation may or may not be prescribed. If it is prescribed, hourly irrigation is not appropriate.

The nurse learning about cancer development remembers characteristics of normal cells. Which characteristics does this include? (Select all that apply.) a. Differentiated function b. Large nucleus-to-cytoplasm ratio c. Loose adherence d. Nonmigratory e. Specific morphology f. Orderly and specific growth

a d e f Normal cells have the characteristics of differentiated function, nonmigratory, specific morphology, a smaller nucleus-to-cytoplasm ratio, tight adherence, and orderly and well-regulated growth.

The nurse is caring for a client in late-stage Alzheimer disease. Which assessment finding(s) will the nurse anticipate? (Select all that apply.) a. Immobile b. Has difficulty driving c. Wandering d. ADL dependent e. Incontinent f. Possible seizures

a d e f The client in late-stage Alzheimer disease is totally bedridden and immobile, and therefore, cannot ambulate to wander or drive. The client is incontinent and ADL dependent.

The nurse plans care for a client with epilepsy who is admitted to the hospital. Which interventions would the nurse include in this client's plan of care? (Select all that apply.) a. Have suction equipment with an airway at the bedside. b. Place a padded tongue blade at the bedside. c. Permit only clear oral fluids. d. Have oxygen administration set at the bedside. e. Maintain the client on strict bedrest. f. Ensure that the client has IV access.

a d f Oxygen and suctioning equipment with an airway must be readily available. If the client does not have an IV access, insert a saline lock, especially for those clients who are at significant risk for generalized tonic-clonic seizures. The saline lock provides ready access if IV drug therapy must be given to stop the seizure. Padded tongue blades may pose a danger to the client or nurse during a seizure and would not be used. Dietary restrictions and strict bedrest are not interventions associated with epilepsy.

After teaching a client who has chronic pancreatitis and will be discharged with enzyme replacement therapy, a nurse assesses the client's understanding. Which statement by the client indicates a need for further teaching? (Select all that apply.) a. "I will take the enzymes between meals." b. "The enteric-coated preparations cannot be crushed." c. "Swallowing the tables without chewing is best." d. "I will wipe my lips after taking the enzymes." e. "Enzymes should be taken with high-protein foods."

a e Client teaching related to self-management of enzyme replacement therapy would include taking the enzymes with meals and snacks but not mixing enzyme preparations with protein-containing foods. Clients would not crush enteric-coated preparations and should swallow tablets without chewing to minimize oral irritation and allow the drug to be released slowly. Wiping lips after taking enzymes also minimizes skin irritation.

A nurse assesses cerebrospinal fluid leaking onto a client's surgical dressing. What actions would the nurse take? (Select all that apply.) a. Place the client in a flat position. b. Monitor vital signs for hypotension. c. Utilize a bedside commode. d. Assess for abdominal distension. e. Report the leak to the surgeon.

a e If cerebrospinal fluid (CSF) is leaking from a surgical wound, the nurse would place the client in a flat position and contact the surgeon for repair of the leak. Hypotension and abdominal distension are not complications of CSF leakage.

A client with long-standing heart failure being treated for cancer has received a dose of ondansetron for nausea. What action by the nurse is most important? a. Assess the client for a headache or dizziness. b. Request a prescription for cardiac monitoring c. Instruct the client to change positions slowly. d. Weigh the client daily before eating.

b 5-HT3 antagonists, such as ondansetron, can prolong the QT interval within the cardiac conduction cycle. ECG monitoring is recommended in patients with electrolyte abnormalities (e.g., hypokalemia or hypomagnesemia), heart failure, bradyarrhythmias or patients taking other medications that can cause QT prolongation. The nurse would contact the primary health care provider and request cardiac monitoring. The nurse would assess the client for any other reported changes, but this is not a critical safety factor. Weight is not related directly to this drug.

A client is preparing to have a laparoscopic restorative proctocolectomy with ileo pouch-anal anastomosis (RCA-IPAA). Which preoperative health teaching would the nurse include? a. "You will have to wear an appliance for your permanent ileostomy." b. "You should be able to have better bowel continence after healing occurs." c. "You will have a large abdominal incision that will require irrigation." d. "This procedure can be performed under general or regional anesthesia."

b A RCA-IPAA can improve bowel continence although leakage may still occur for some clients. The procedure is a 2-step process performed under general anesthesia using a laparoscope which does not require an abdominal incision or permanent ileostomy.

A client has a platelet count of 9800/mm3 (9800 × 109 /L). What action by the nurse is most appropriate? a. Assess the client for calf pain, warmth, and redness. b. Instruct the client to call for help to get out of bed. c. Obtain cultures as per the facility's standing policy. d. Place the client on protective isolation precautions

b A client with a platelet count this low is at high risk for serious bleeding episodes. To prevent injury, the client would be instructed to call for help prior to getting out of bed. Calf pain, warmth, and redness might indicate a deep vein thrombosis, not associated with low platelets. Cultures and isolation relate to low white cell counts.

A client who had a complete spinal cord injury at level L5-S1 is admitted with a sacral pressure injury. What other assessment finding will the nurse anticipate for this client? a. Quadriplegia b. Flaccid bowel c. Spastic bladder d. Tetraparesis

b A low-level complete spinal cord injury (SCI) is a lower motor neuron injury because the reflect arc is damaged. Therefore, the client would be expected to have paraplegia and a flaccid bowel and bladder. Quadriplegia and tetraparesis are seen in clients with cervical or high thoracic SCIs.

The nurse is caring for a client with hepatic portal-systemic encephalopathy (PSE). The client is thin and cachectic, and the family expresses distress that the patient is receiving little dietary protein. How would the nurse respond? a. "A low-protein diet will help the liver rest and will restore liver function." b. "Less protein in the diet will help prevent confusion associated with liver failure." c. "Increasing dietary protein will help the patient gain weight and muscle mass." d. "Low dietary protein is needed to prevent fluid from leaking into the abdomen."

b A low-protein diet is prescribed when serum ammonia levels increase and/or the client shows signs of PSE. A low-protein diet helps reduce excessive breakdown of protein into ammonia by intestinal bacteria. Encephalopathy is caused by excess ammonia. A low-protein diet has no impact on restoring liver function. Increasing the patient's dietary protein will cause complications of liver failure and would not be suggested. Increased intravascular protein will help prevent ascites, but clients with liver failure are not able to effectively synthesize dietary protein.

The nurse is caring for a client who has cirrhosis of the liver. What nursing action is appropriate to help control ascites? a. Monitor intake and output. b. Provide a low-sodium diet. c. Increase oral fluid intake. d. Weigh the patient daily.

b A low-sodium diet is one means of controlling abdominal fluid collection. Monitoring intake and output does not control fluid accumulation, nor does weighing the client. These interventions merely assess or monitor the situation. Increasing fluid intake would not be helpful.

A client is receiving IV alteplase and reports a sudden severe headache. What is the nurse's first action? a. Perform a comprehensive pain assessment. b. Discontinue the infusion of the drug. c. Conduct a neurologic assessment. d. Administer an antihypertensive drug.

b A severe headache may indicate that the client's blood pressure has markedly increased and, therefore, the drug should be stopped immediately as the first action. The nurse would then perform the appropriate assessments and possibly administer an antihypertensive medication.

After teaching a client who is prescribed adalimumab for severe ulcerative colitis (UC), the nurse assesses the client's understanding. Which statement made by the client indicates a need for further teaching? a. "I will avoid large crowds and people who are sick." b. "I will take this medication with my breakfast each morning." c. "Nausea and vomiting are common side effects of this drug." d. "I should wash my hands after I play with my dog."

b Adalimumab is an immune modulator that is given via subcutaneous injection. It does not need to be given with food or milk. Nausea and vomiting are two common side effects. Adalimumab can cause immune suppression, so clients receiving the medication should avoid large crowds and people who are sick, and should practice good handwashing.

The nurse is caring for a client who is planning to have a laparoscopic colon resection for colorectal cancer tomorrow. Which statement by the client indicates a need for further teaching? a. "I should have less pain after this surgery compared to having a large incision." b. "I will probably be in the hospital for 3 to 4 days after surgery." c. "I will be able to walk around a little on the same day as the surgery." d. "I will be able to return to work in a week or two depending on how I do."

b All of these statements are correct about having minimally invasive laparoscopic surgery except that the hospital stay will likely be only 1 or 2 days.

The nurse is caring for a client who has a postoperative paralytic ileus following abdominal surgery. What drug is appropriate to manage this nonmechanical bowel obstruction? a. Alosetron b. Alvimopan c. Amitiptyline d. Amlodipine

b Alvimopan is the appropriate drug to promote peristalsis for clients who have a paralytic ileus. The other drugs do not affect intestinal activity.

A client has an external percutaneous transhepatic biliary catheter inserted for a biliary obstruction. What health teaching about catheter care would the nurse provide for the client? a. "Cap the catheter drain at night to prevent leakage and skin damage." b. "Position the drainage bag lower than the catheter insertion site." c. "Irrigate the catheter with an ounce of saline every night." d. "Pierce a hole in the top of the drainage bag to get rid of odors."

b An external temporary or permanent catheter drains bile by gravity into a bag that collects bile. Therefore, the drainage bag should be lower that the catheter insertion site. The catheter should not be capped or irrigated, and no holes should be made in the bag to prevent bile from having contact with the skin.

A client who has peptic ulcer disease is prescribed quadruple drug therapy for Helicobacter pylori infection. What health teaching related to bismuth would the nurse include? a. "Report stool changes to your primary health care provider immediately." b. "Do not take aspirin or aspirin products of any kind while on bismuth." c. "Take bismuth about 30 minutes before each meal and at bedtime." d. "Be aware that bismuth can cause frequent vomiting and diarrhea."

b Bismuth is a salicylate drug and causes stool discoloration but not vomiting and diarrhea. It does not have to be taken at a specific time relative to meals. Clients taking bismuth should not take other salicylates, such as aspirin or aspirin-containing products.

A nurse assesses the health history of a client who is prescribed ziconotide for chronic low back pain. Which assessment question would the nurse ask? a. "Are you taking a nonsteroidal anti-inflammatory drug?" b. "Have you been diagnosed with a mental health problem?" c. "Are you able to swallow oral medications?" d. "Do you smoke cigarettes or any illegal drugs?"

b Clients who have a severe mental health or behavioral health problem would not take ziconotide because the drug can cause psychotic symptoms such as hallucinations. The other questions do not identify a contraindication for this medication.

A nurse teaches a client who is recovering from an open traditional cervical spinal fusion. Which statement would the nurse include in this client's postoperative instructions? a. "Only lift items that are 10 lb (4.5 kg) or less." b. "Wear your neck brace whenever you are out of bed." c. "You must remain in bed for 3 weeks after surgery." d. "You will be prescribed medications to prevent graft rejection."

b Clients who undergo spinal fusion are fitted with a neck brace that they must wear throughout the healing process whenever they are out of bed. The client should not lift anything more than 10 lb (4.5 kg). The client does not need to remain in bed. Medications for rejection prevention are not necessary for this procedure.

A client has a nasogastric (NG) tube as a result of an upper gastrointestinal (GI) hemorrhage. What comfort measure would the nurse remind assistive personnel (AP) to provide? a. Lavaging the tube with ice water b. Performing frequent oral care c. Re-positioning the tube every 4 hours d. Taking and recording vital signs

b Clients with NG tubes need frequent oral care both for comfort and to prevent infection. Lavaging the tube is done by the nurse. Repositioning the tube, if needed, is also done by the nurse. The can take vital signs, but this is not a comfort measure.

A client who had a partial gastrectomy 3 days ago begins to experience vertigo, sweating, and tachycardia about 30 minutes after eating breakfast. What postoperative complication would the nurse suspect? a. Pyloric obstruction b. Dumping syndrome c. Delayed gastric emptying d. Pernicious anemia

b Dumping syndrome causes autonomic symptoms as food quickly leaves the stomach due to its decreased size after surgery.

1. The nurse learning about cellular regulation understands that which process occurs during the S phase of the cell cycle? a. Actual division (mitosis) b. Doubling of DNA c. Growing extra membrane d. No reproductive activity

b During the S phase, the cell must double its DNA content through DNA synthesis. Actual division, or mitosis, occurs during the M phase. Growing extra membrane occurs in the G1 phase. During the G0 phase, the cell is working but is not involved in any reproductive activity.

A client with cancer has anorexia and mucositis, and is losing weight. The client's family members continually bring favorite foods to the client and are distressed when the client won't eat them. What action by the nurse is best? a. Explain the pathophysiologic reasons behind the client not eating. b. Help the family show other ways to demonstrate love and caring. c. Suggest foods and liquids the client might be willing to try to eat. d. Tell the family the client isn't able to eat now no matter what they bring.

b Families often become distressed when their loved ones won't eat. Providing food is a universal sign of caring, and to some people the refusal to eat signifies worsening of the condition. The best option for the nurse is to help the family find other ways to demonstrate caring and love, because with treatment-related anorexia and mucositis, the client is not likely to eat anything right now. Explaining the rationale for the problem is a good idea but does not suggest to the family anything that they can do for the client. Simply telling the family the client is not able to eat does not give them useful information and is dismissive of their concerns.

A telehealth nurse speaks with a client who is recovering from a liver transplant 2 weeks ago. The client states, "I'm having right belly pain and have a temperature of 101° F (38.3° C)." How would the nurse respond? a. "The anti-rejection drugs you are taking make you susceptible to infection." b. "You should go to the hospital immediately to get checked out." c. "You should take an additional dose of cyclosporine today." d. "Take acetaminophen every 4 hours until you feel better soon."

b Fever, right abdominal quadrant pain, and jaundice are signs of possible liver transplant rejection; the client would be admitted to the hospital as soon as possible for intervention. Antirejection drugs do make a client more susceptible to infection, but this client has signs of rejection, not infection. The nurse would not advise the client to take an additional dose of cyclosporine or acetaminophen as these medications will not treat the acute rejection.

A nurse assesses a client with multiple sclerosis after administering prescribed fingolimod. For which common side effect would the nurse monitor? a. Peripheral edema b. Facial flushing c. Tachycardia d. Fever

b Fingolimod is an oral immunomodulator that has two common side effects—facial flushing and GI disturbance, such as diarrhea. Peripheral edema, tachycardia, and fever are not common side effects of this drug.

A nurse is preparing to administer a blood transfusion. What action is most important? a. Correctly identify client using two identifiers. b. Ensure that informed consent is obtained. c. Hang the blood product with Ringer's lactate. d. Stay with the client for the entire transfusion.

b If the facility requires informed consent for transfusions, this action is most important because it precedes the other actions taken during the transfusion. Correctly identifying the client and blood product is a National Patient Safety Goal, and is the most important action after obtaining informed consent. Ringer's lactate is not used to transfuse blood. The nurse does not need to stay with the client for the duration of the transfusion.

A nurse cares for a client who is experiencing status epilepticus. Which prescribed medication would the nurse anticipate to prepare for administration? a. Atenolol b. Lorazepam c. Phenytoin d. Lisinopril

b Initially, intravenous lorazepam or diazepam is administered to stop motor movements. This is followed by the administration of phenytoin. Atenolol, a beta blocker, and lisinopril, an angiotensin-converting enzyme inhibitor, are not administered for seizure activity. These drugs are typically administered for hypertension and heart failure.

The nurse is caring for a client who has been prescribed lubiprostone for irritable bowel syndrome (IBS-C). What health teaching will the nurse include about taking this drug? a. "This drug will make you very dry because it will decrease your diarrhea." b. "Be sure to take this drug with food and water to help manage constipation." c. "Avoid people who have infection as this drug will suppress your immune system." d. "Include high-fiber foods in your diet to help produce more solid stools."

b Lubiprostone is an oral laxative approved for women who have IBS with constipation (IBS-C). Water and food will also help to improve constipation. The drug is not used for clients who have diarrhea and does not affect the immune system. Although high-fiber foods are important for clients who have IBS, this client does not need fiber to help make stool more solid. Instead the fiber will help prevent constipation.

The family of a neutropenic client reports that the client "is not acting right." What action by the nurse is the priority? a. Ask the client about pain. b. Assess the client for infection. c. Take a set of vital signs. d. Review today's laboratory results.

b Neutropenic clients often do not have classic manifestations of infection, but infection is the most common cause of death in neutropenic clients. The nurse would definitely assess for infection. The nurse would assess for pain but this is not the priority.

1. After teaching a client with irritable bowel syndrome (IBS), a nurse assesses the client's understanding. Which menu selection indicates that the client correctly understands the dietary teaching? a. Ham sandwich on white bread, cup of applesauce, carbonated beverage b. Broiled chicken with brown rice, steamed broccoli, glass of apple juice c. Grilled cheese sandwich, small banana, cup of hot tea with lemon d. Baked tilapia, fresh green beans, cup of coffee with low-fat milk

b Patients with IBS are advised to eat a high-fiber diet. Chicken with brown rice, broccoli, and apple juice has the highest fiber content. Clients should avoid alcohol, caffeine, and other gastric irritants.

A client is scheduled for a percutaneous endoscopic lumbar discectomy. Which statement by the client indicates a need for further teaching? a. "I should have a lot less pain after surgery." b. "I'll be in the hospital for 2 to 3 days." c. "I should not have any major surgical complications." d. "I could possibly get an infection after surgery"

b Percutaneous endoscopic discectomy is a minimally invasive surgical procedure that requires a shorter hospital stay (23 hours or less) when compared to open traditional surgery. The risk for surgical complications is very low and clients experience less far pain from this procedure. However, due to interrupting skin integrity, infection may occur at the surgical site.

The nurse is caring for a client who is recovering from an open traditional Whipple surgical procedure. What action would the nurse take? a. Clamp the nasogastric tube. b. Place the patient in semi-Fowler position. c. Assess vital signs once every shift. d. Provide oral rehydration.

b Postoperative care for a patient recovering from an open Whipple procedure would include placing the client in a semi-Fowler position to reduce tension on the suture line and anastomosis sites and promote breathing, setting the nasogastric tube to low continuous suction to remove free air buildup and pressure, assessing vital signs frequently to assess fluid and electrolyte complications, and providing intravenous fluids.

A nurse is teaching a client who experiences migraine headaches and is prescribed propranolol. Which statement would the nurse include in this client's teaching? a. "Take this drug only when you have symptoms indicating the onset of a migraine headache." b. "Take this drug as prescribed, even when feeling well, to prevent vascular changes associated with migraine headaches." c. "This drug will relieve the pain during the aura phase soon after a headache has started." d. "This drug will have no effect on your heart rate or blood pressure because you are taking it for migraines."

b Propranolol is a beta-adrenergic blocker which is prescribed as prophylactic treatment to prevent the vascular changes that initiate migraine headaches. Heart rate and blood pressure will also be affected, and the client would monitor these side effects. The other responses do not discuss appropriate uses of this drug

The nurse plans care for a client with Crohn disease who has a heavily draining fistula. Which intervention would be the nurse's priority action? a. Low-fiber diet b. Skin protection c. Antibiotic administration d. Intravenous glucocorticoids

b Protecting the client's skin is the priority action for a patient who has a heavily draining fistula. Intestinal fluid enzymes are caustic and can cause skin breakdown or fungal infections if the skin is not protected. The plan of care for a client who has Crohn disease also includes adequate nutrition focused on high-calorie, high-protein, high-vitamin, and low-fiber meals, antibiotic administration, and glucocorticoids.

A client in the oncology clinic reports her family is frustrated at her ongoing fatigue 4 months after radiation therapy for breast cancer. What response by the nurse is most appropriate? a. "Are you getting adequate rest and sleep each day?" b. "It is normal to be fatigued even for months afterward." c. "This is not normal and I'll let the primary health care provider know." d. "Try adding more vitamins B and C to your diet"

b Radiation-induced fatigue can be debilitating and may last for months after treatment has ended. Rest and adequate nutrition can affect fatigue, but it is most important that the client (and family) understands this is normal.

The nurse cares for a client with middle-stage (moderate) Alzheimer disease. The client's caregiver states, "She is always wandering off. What can I do to manage this restless behavior?" What is the nurse's best response? a. "This is a sign of fatigue. The client would benefit from a daily nap." b. "Engage the client in scheduled activities throughout the day." c. "It sounds like this is difficult for you. I will consult the social worker." d. "The provider can prescribe a mild sedative for restlessness."

b Several strategies may be used to cope with restlessness and wandering. One strategy is to engage the client in structured activities. Another is to take the client for frequent walks. Daily naps and a mild sedative will not be as effective in the management of restless behavior. Consulting the social worker does not address the caregiver's concern.

A nurse participates in a community screening event for oral cancer. What client is the highest priority for referral to a primary health care provider? a. Client who has poor oral hygiene practices. b. Client who smokes and drinks daily. c. Client who tans for an upcoming vacation. d. Client who occasionally uses illicit drugs.

b Smoking and alcohol exposure create a high risk for this client. Poor oral hygiene is not related to the etiology of cancer but may cause a tumor to go unnoticed. Tanning is a risk factor, but short-term exposure does not have the same risk as daily exposure to tobacco and alcohol.

A nurse cares for a client with a new ileostomy. The client states, "I don't think my friends will accept me with this ostomy." How would the nurse respond? a. "Your friends will be happy that you are alive." b. "Tell me more about your concerns." c. "A therapist can help you resolve your concerns." d. "With time you will accept your new body."

b Social anxiety and apprehension are common in clients with a new ileostomy. The nurse would encourage the client to discuss concerns by restating them in an open-ended manner. The nurse would not minimize the client's concerns or provide false reassurance.

The nurse is caring for a client who is prescribed sulfasalazine. Which question would the nurse ask the client before starting this drug? a. "Are you taking Vitamin C or B? b. "Do you have any allergy to sulfa drugs?" c. "Can you swallow pills pretty easily?" d. "Do you have insurance to cover this drug?"

b Sulfasalazine is a sulfa drug given for clients who have ulcerative colitis. However, it should not be given to those who have an allergy to sulfa and sulfa drugs to prevent a hypersensitivity reaction.

The nurse obtains a health history on a client prior to administering prescribed sumatriptan succinate for migraine headaches. Which condition would alert the nurse to withhold the medication and contact the primary health care provider? a. Bronchial asthma b. Heart disease c. Diabetes mellitus d. Rheumatoid arthritis

b Sumatriptan succinate effectively reduces pain and other associated symptoms of migraine headache by binding to serotonin receptors and triggering cranial vasoconstriction. Vasoconstrictive effects are not confined to the cranium and can cause coronary vasospasm in clients with heart disease, hypertension, or Prinzmetal angina. The other conditions would not affect the client's treatment.

A client is preparing to have a fecal occult blood test (FOBT). What health teaching would the nurse include prior to the test? a. "This test will determine whether you have colorectal cancer." b. "You need to avoid red meat and NSAIDs for 48 hours before the test." c. "You don't need to have this test because you can have a virtual colonoscopy." d. "This test can determine your genetic risk for developing colorectal cancer."

b The FOBT is a screening test that is sometimes used to assess for microscopic lower GI bleeding. To help prevent false positive results, the client needs to avoid red meat, Vitamin C, and NSAIDs. The test is not diagnostic nor does it determine a client's genetic risk for colorectal cancer.

A client is admitted with acute pancreatitis. What priority problem would the nurse expect the client to report? a. Nausea and vomiting b. Severe boring abdominal pain c. Jaundice and itching d. Elevated temperature

b The client who has acute pancreatitis reports severe boring abdominal pain that is often rated by clients as a 10+ on a 0-10 pain scale. Nausea, vomiting, and fever may also occur, but that is not the client's priority for care.

A client with early-stage Alzheimer disease is admitted to the hospital with chest pain. Which nursing action is most appropriate to manage this client's dementia? a. Provide animal-assisted therapy as needed. b. Ensure a structured and consistent environment. c. Assist the client with activities of daily living (ADLs). d. Use validation therapy when communicating with the client.

b The client who has early Alzheimer disease (AD) does not require assistance with ADLs or validation therapy. While animal-assisted therapy may be helpful, some health care agencies do not allow this intervention. Therefore, the most appropriate action is to provide a structured and consistent environment while the client is hospitalized to prevent worsening of the client's symptoms.

The nurse is teaching assistive personnel (AP) about care for a male client diagnosed with acute ischemic stroke and left-sided weakness. Which statement by the AP indicates understanding of the nurse's teaching? a. "I will use "yes" and "no" questions when communicating with the client." b. "I will remind the client frequently to not get out of bed without help." c. "I will offer a urinal every hour to the client due to incontinence." d. "I will feed the client slowly using soft or pureed foods."

b The client who has left-sided weakness has likely had a right-sided stroke in the brain. Clients who have strokes on the right side of the brain tend to be very impulsive and exhibit poor judgment. Therefore, to keep the client safe, the staff will need to remind the client to stay in bed unless he has assistance to prevent falling. There is no evidence in the clinical situation that the client has aphasia (which is less common in those with right-sided strokes), difficulty swallowing, or urinary incontinence.

The nurse is caring for a client diagnosed with oral cancer. What is the nurse's priority for client care? a. Encourage fluids to liquefy the client's secretions. b. Place the client on Aspiration Precautions. c. Remind the client to use an incentive spirometer. d. Manage the client's pain and inflammation.

b The client who has oral cancer often has difficulty swallowing and is at risk for aspiration and possibly aspiration pneumonia. Therefore, the most important nursing action is to place the client on precautions to prevent aspiration. The nurse would implement the other actions but they are not as vital to promote client safety.

The nurse is assessing a client who has probable lymphoma. What is the most common early assessment finding for clients with this disorder? a. Weight gain b. Enlarged painless lymph node(s) c. Fever at night d. Nausea and vomiting

b The first change that is noted for clients with probable lymphoma is one or more enlarged lymph nodes. The other findings are either not common in clients with lymphoma or later findings.

A client has a recurrence of gastric cancer and is crying. What response by the nurse is most appropriate? a. "Do you have family or friends for support?" b. "Would you tell me what you are feeling now." c. "Well, we knew this would probably happen." d. "Would you like me to refer you to hospice?"

b The nurse assesses the client's emotional state with open-ended questions and statements and shows a willingness to listen to the client's concerns. Asking about support people is very limited in nature, and "yes-or-no" questions are not therapeutic. Stating that this was expected dismisses the client's concerns. The client may or may not be ready to hear about hospice, and this is another limited, yes-or-no question.

After teaching a client who has been diagnosed with hepatitis A, the nurse assesses the client's understanding. Which statement by the client indicates correct understanding of the teaching? a. "Some medications have been known to cause hepatitis A." b. "I may have been exposed when we ate shrimp last weekend." c. "I was infected with hepatitis A through a recent blood transfusion." d. "My infection with Epstein-Barr virus can co-infect me with hepatitis A."

b The route of transmission for hepatitis A infection is through close personal contact or ingestion of contaminated water or shellfish. Hepatitis A is not transmitted through medications, blood transfusions, or Epstein-Barr virus. Toxic and drug-induced hepatitis is caused from exposure to hepatotoxins, but this is not a form of hepatitis A. Hepatitis B can be spread through blood transfusions. Epstein-Barr virus causes a secondary infection that is not associated with hepatitis A.

A nurse learns that which of the following is the single biggest risk factor for developing cancer? a. Exposure to tobacco b. Advancing age c. Occupational chemicals d. Oncovirus infection

b The single biggest risk factor for developing cancer is advancing age. As one ages, immunity decreases and exposures increase. Tobacco use is the single most preventable cause of cancer. Exposure to chemicals and oncoviruses cause fewer cancers.

A client tells the oncology nurse about an upcoming vacation to the beach to celebrate completing radiation treatments for cancer. What response by the nurse is most appropriate? a. "Avoid getting salt water on the radiation site." b. "Do not expose the radiation area to direct sunlight." c. "Have a wonderful time and enjoy your vacation!" d. "Remember you should not drink alcohol for a year."

b The skin overlying the radiation site is extremely sensitive to sunlight after radiation therapy has been completed. The nurse would inform the client to avoid sun exposure to this area. This advice continues for 1 year after treatment has been completed. The other statements are not appropriate.

After a craniotomy, the nurse assesses the client and finds dry, sticky mucous membranes, acute confusion, and restlessness. The client has IV fluids running at 75 mL/hr. What action by the nurse would the nurse take first? a. Assess the client's urinary output. b. Assess the client's serum sodium level. c. Increase the rate of the IV infusion. d. Provide oral care every hour.

b This client has signs and symptoms of hypernatremia, which is a possible complication after craniotomy. The nurse would assess the client's serum sodium level first and then possibly increase the rate of the IV infusion. Providing oral care is also a good option but does not take priority over assessing laboratory results.

A client receiving a blood transfusion develops anxiety and low back pain. After stopping the transfusion, what action by the nurse is most important? a. Document the events in the client's medical record. b. Double-check the client and blood product identification. c. Place the client on strict bedrest until the pain subsides. d. Review the client's medical record for known allergies.

b This client most likely had a hemolytic transfusion reaction, most commonly caused by blood type or Rh incompatibility. The nurse should double-check all identifying information for both the client and blood type. Documentation occurs after the client is stable. Bedrest may or may not be needed. Allergies to medications or environmental items are not related.

A client is in the emergency department reporting a brief episode during which he was dizzy, unable to speak, and felt numbness in his left leg. Currently the client's neurologic examination is normal. About what drug would the nurse plan to teach the patient? a. Alteplase b. Clopidogrel c. Heparin sodium d. Mannitol

b This client's signs and symptoms are consistent with a transient ischemic attack, and the client would likely be prescribed aspirin or clopidogrel to prevent platelet aggregation on discharge. Alteplase is used for ischemic stroke. Heparin and mannitol are not used for this condition.

A client is having a catheter placed to deliver chemotherapy beads into a liver tumor via the femoral artery. What action by the nurse is most important? a. Assessing the client's abdomen beforehand b. Ensuring that informed consent is on the chart c. Marking the client's bilateral pedal pulses d. Reviewing client teaching done previously

b This is an invasive procedure requiring informed consent. The nurse would ensure that consent is on the chart. The other actions are also appropriate but not as important as ensuring the client has given consent.

A nurse in a hematology clinic is working with four clients who have polycythemia vera. Which client would the nurse assess first? a. Client with a blood pressure of 180/98 mm Hg b. Client who reports shortness of breath c. Client who reports calf tenderness and swelling d. Client with a swollen and painful left great toe

b Clients with polycythemia vera often have clotting abnormalities due to the hyperviscous blood with sluggish flow. The client reporting shortness of breath may have a pulmonary embolism and should be seen first. The client with a swollen calf may have a deep vein thrombosis and should be seen next. High blood pressure and gout symptoms are common findings with this disorder.

The nursing is teaching a client diagnosed with gastroesophageal reflux disease (GERD) who is planning to have an endoscopic radiofrequency (Stretta) procedure. What preprocedure health teaching would the nurse include? (Select all that apply.) a. "You will need to be on a liquid diet for the first week after the procedure." b. "Avoid taking any NSAIDs like ibuprofen for 10 days before the procedure." c. "Contact the primary health care provider after the procedure if you have increased pain." d. "You will need a nasogastric tube for a few days after the procedure." e. "You will have a small incision in your stomach area that will have a wound closure.

b c The client having this procedure does not have an incision and will not require a nasogastric tube (NGT). The client should avoid an NGT placement for at least a month after the procedure. A liquid diet is required for only 24 hours after the procedure and then the client should progress to include soft floods like custard and applesauce.

The nurse is caring for a client who had an open traditional esophagectomy. Which assessment findings would the nurse report immediately to the primary health care provider? (Select all that apply.) a. Nausea b. Wound dehiscence c. Fever d. Tachycardia e. Moderate pain f. Fatigue

b c d Wound dehiscence is a serious, potentially life-threatening problem that needs immediate attention of the primary health care provider, typically the surgeon. Fever and tachycardia may indicate that the client has a postoperative infection, another serious, potentially life-threatening complication. Indications of both of these problems need to be documented and reported by the nurse. Nausea, fatigue, and moderate pain are expected postoperative assessment findings.

The nurse is assessing a client who has hepatitis C. What extrahepatic complications would the nurse anticipate? (Select all that apply.) a. Pancreatitis b. Polyarthritis c. Heart disease d. Myalgia e. Peptic ulcer disease f. Ulcerative colitis

b c d The client who has hepatitis C has complications that do not relate to the liver, including polyarthritis, myalgia, heart disease and vasculitis, renal disease, and cognitive impairment.

The nurse is teaching a client who had a descending colostomy 2 days ago about the ostomy stoma. Which changes in the stoma would the nurse teach the client to report to the primary health care provider? (Select all that apply.) a. Stool consistency is similar to paste. b. Stoma becomes dark and dull. c. Skin around the stoma becomes excoriated. d. Skin around stoma becomes protruded. e. Stoma becomes retracted into the abdomen.

b c d e A colostomy placed in the descending colon would be expect to have a paste-like stool consistency. However, if the stoma becomes retracted or discolored, the client should report those changes to the primary health care provider. Skin around the stoma that becomes protruded would suggest the formation of a peristomal hernia, and skin excoriation needs appropriate management. Therefore, both of those skin changes would need to be reported to the primary health care provider.

A nurse assesses a client who is recovering from an open traditional Whipple surgical procedure. Which assessment finding(s) alert(s) the nurse to a complication from this surgery? (Select all that apply.) a. Clay-colored stools b. Substernal chest pain c. Shortness of breath d. Lack of bowel sounds or flatus e. Urine output of 20 mL/6 hr

b c d e Myocardial infarction (chest pain), pulmonary embolism (shortness of breath), adynamic ileus (lack of bowel sounds or flatus), and acute kidney injury (urine output of 20 mL/6 hr) are common complications for which the nurse must assess the client after the Whipple procedure. Clay-colored stools are associated with cholecystitis and are not a complication of a Whipple procedure.

A client is admitted with a confirmed left middle cerebral artery occlusion. Which assessment findings will the nurse expect? (Select all that apply.) a. Ataxia b. Dysphagia c. Aphasia d. Apraxia e. Hemiparesis/hemiplegia f. Ptosis

b c d e f All of these assessment findings are common in clients who have a stroke caused by an occlusion of the left middle cerebral artery with the exception of ataxia (most often present in clients who have cerebellar strokes). This artery supplies the majority of the left side of the brain where motor, sensory, speech, and language centers are located.

The nurse is caring for a client with probable colorectal cancer (CRC). What assessment findings would the nurse expect? (Select all that apply.) a. Weight gain b. Rectal bleeding c. Anemia d. Change in stool shape e. Electrolyte imbalances f. Abdominal discomfort

b c d f The client who has CRC usually experiences unintentional weight loss and rectal bleeding, either gross or occult. As a result of bleeding, the client has anemia and fatigue. Electrolyte imbalances are not common, but the client may note that the shape or consistency of stool has changed.

a nurse assesses a patient who has celiac disease. Which signs and symptoms would the nurse expect? (Select all that apply.) a. Weight gain b. Anorexia c. Constipation d. Anal fistula e. Abdominal pain

b c e Signs and symptoms of celiac disease include weight loss, anorexia, constipation, and abdominal pain. Anal fistulas are not associated with celiac disease.

A nurse is participating in primary prevention efforts directed against cancer. In which activities is this nurse most likely to engage? (Select all that apply.) a. Demonstrating breast self-examination methods to women b. Instructing people on the use of chemoprevention c. Providing vaccinations against certain cancers d. Screening teenage girls for cervical cancer e. Teaching teens the dangers of tanning booths f. Educating adults about healthy eating habits

b c e f Primary prevention aims to prevent the occurrence of a disease or disorder, in this case cancer. Secondary prevention includes screening and early diagnosis. Primary prevention activities include teaching people about chemoprevention, providing approved vaccinations to prevent cancer, teaching teens the dangers of tanning beds, and educating adults on eating habits to reduce the risk of getting cancer. Breast examinations and screening for cervical cancer are secondary prevention methods.

A nurse is discharging a client from the emergency department who has a mild traumatic brain injury. What information obtained from the client represents a possible barrier to self-management? (Select all that apply.) a. Does not want to purchase a thermometer. b. Is allergic to acetaminophen. c. Laughing, says "Strenuous? What's that?" d. Lives alone and is new in town with no friends. e. Plans to have a beer and go to bed once home.

b d e Clients who have mild traumatic brain injuries should take acetaminophen for headache. An allergy to this drug may mean that the patient takes aspirin or ibuprofen, which should be avoided. The patient needs neurologic checks every 1 to 2 hours, and this client does not seem to have anyone available who can do that. Alcohol needs to be avoided for at least 24 hours. A thermometer is not needed. The patient laughing at strenuous activity probably does not engage in any kind of strenuous activity, but the nurse should confirm this.

A nurse cares for a patient who has a chronic inflammatory bowel disease. Which actions would the nurse take to prevent skin excoriation? (Select all that apply.) a. Cleanse the perineum with an antibacterial soap. b. Use medicated wipes instead of toilet paper. c. Identify foods that decrease constipation. d. Apply a thin coat of aloe cream to the perineum. e. Gently pat the perineum dry after cleansing.

b d e To prevent skin excoriation from frequent bowel movements associated with inflammatory bowel disease, the nurse would encourage good skin care with a mild soap and water and gently patting the area dry after each bowel movement. Using medicated wipes instead of toilet paper and applying a thin coat of aloe cream are appropriate. The client should identify and avoid foods that increase diarrhea. Antibacterial soaps are harsh and should not be used.

The nurse is caring for a client with a long history of peptic ulcer disease. What assessment findings would the nurse anticipate if the client experiences upper gastrointestinal (GI) bleeding? (Select all that apply.) a. Decreased heart rate b. Decreased blood pressure c. Bounding radial pulse d. Dizziness e. Hematemesis f. Decreased urinary output

b d e f The client who has upper GI bleeding would likely have vomiting that contains blood (hematemesis), and would have signs and symptoms of dehydration such as a decreased blood pressure, dizziness, and/or decreased urinary output. The heart rate increases rather than decreases and the pulse is weak rather than bounding in clients who are dehydrated.

A nurse plans care for a client with a halo fixator. Which interventions would the nurse include in this client's plan of care? (Select all that apply.) a. Remove the vest for client bathing. b. Assess the pin sites for signs of infection. c. Loosen the pins when sleeping. d. Decrease the patient's oral fluid intake. e. Assess the chest and back for skin breakdown.

b e The nurse would assess the pin sites for signs of infection or loose pins. The nurse would also assess the client's chest and back for skin breakdown from the halo vest. The vest is not removed for bathing and the pins are not intentionally loosened.

A nurse assesses a client who has cirrhosis of the liver. Which laboratory findings would the nurse expect in clients with this disorder? (Select all that apply.) a. Elevated aspartate transaminase b. Elevated international normalized ratio (INR) c. Decreased serum globulin levels d. Decreased serum alkaline phosphatase e. Elevated serum ammonia f. Elevated prothrombin time (PT)

b e f Elevated INR and PT are indications of clotting disturbances and alert the nurse to the increased possibility of hemorrhage. Elevated ammonia levels increase the client's confusion. The other values are abnormal and associated with liver disease but do not necessarily place the client at increased risk for complications.

During an interview, the client tells the nurse that the client has a duodenal ulcer. Which assessment finding would the nurse expect? a. Hematemesis b. Pain when eating c. Melena d. Weight loss

c All of the other assessment findings are more commonly seen in clients who have gastric ulcers rather than duodenal ulcers.

A nurse is caring for a young male client with lymphoma who is to begin treatment. What teaching topic is a priority? a. Genetic testing b. Infection prevention c. Sperm banking d. Treatment options

c All teaching topics are important to the client with lymphoma, but for a young male, sperm banking is of particular concern if the client is going to have radiation to the lower abdomen or pelvis.

A client is admitted with a traumatic brain injury. What is the nurse's priority assessment? a. Complete neurologic assessment b. Comprehensive pain assessment c. Airway and breathing assessment d. Functional assessment

c Although the client has a brain injury, the most important assessment is to assess the client's ABCs, which includes airway, breathing, and circulation. The other assessments are performed later after the client is stabilized.

A nurse assesses a client who has ulcerative colitis and severe diarrhea. Which assessment would the nurse complete first? a. Inspection of oral mucosa b. Recent dietary intake c. Heart rate and rhythm d. Percussion of abdomen

c Although the client with severe diarrhea may experience skin irritation and hypovolemia, the client is most at risk for cardiac dysrhythmias secondary to potassium and magnesium loss from severe diarrhea. The client would have her or his electrolyte levels monitored, and electrolyte replacement may be necessary. Oral mucosa inspection, recent dietary intake, and abdominal percussion are important parts of physical assessment but are lower priority for this patient than heart rate and rhythm.

The nurse reviews the laboratory results for a client who has possible appendicitis. Which laboratory test finding would the nurse expect? a. Decreased potassium level b. Increased sodium level c. Elevated leukocyte count d. Decreased thrombocyte count

c Appendicitis is an acute inflammatory disorder that frequently results in elevation of leukocytes (white blood cells). Serum electrolytes are not affected because the client does not usually have diarrhea. Thrombocyte (platelet) count is unrelated to this GI disorder.

After teaching a client who has alcohol-induced cirrhosis, a nurse assesses the client's understanding. Which statement made by the client indicates a need for further teaching? a. "I cannot drink any alcohol at all anymore." b. "I should not take over-the-counter medications." c. "I need to avoid protein in my diet." d. "I should eat small, frequent, balanced meals."

c Based on the degree of liver involvement and decreased function, protein intake may have to be decreased. However, some protein is necessary for the synthesis of albumin and normal healing. The other statements indicate accurate understanding of self-care measures for this client.

A nurse is learning the difference between normal cells and benign tumor cells. What information does this include? a. Benign tumors grow through invasion of other tissue. b. Benign tumors have lost their cellular regulation from contact inhibition. c. Growing in the wrong place or time is typical of benign tumors. d. The loss of characteristics of the parent cells is called anaplasia.

c Benign tumors are basically normal cells growing in the wrong place or at the wrong time. Benign cells grow through hyperplasia, not invasion. Benign tumor cells retain contact inhibition. Anaplasia is a characteristic of cancer cells.

The nurse is teaching a group of college students about the importance of preventing meningitis. Which health promotion activity is the most appropriate for preventing this disease? a. Eating a well-balanced diet that is high in protein b. Having an annual physical examination c. Obtaining the recommended meningitis vaccination and boosters d. Identifying signs and symptoms for early treatment

c CDC-recommended vaccinations and boosters are available for prevention of a number of diseases including meningococcal meningitis. While the other activities are appropriate for general health promotion, they are not specific to meningitis prevention.

The nurse is caring for a client who has cirrhosis from substance abuse. The client states, "All of my family hates me." How would the nurse respond? a. "You should make peace with your family." b. "This is not unusual. My family hates me too." c. "I will help you identify a support system." d. "You must attend Alcoholics Anonymous."

c Clients who have cirrhosis due to addiction may have alienated relatives over the years because of substance abuse. The nurse would assist the client to identify a friend, neighbor, clergy/spiritual leader, or group for support. The nurse would not minimize the patient's concerns. Attending AA may be appropriate, but this response doesn't address the client's concern. "Making peace" with the client's family may not be possible. This statement is not client-centered.

The nurse teaches assistive personnel (AP) about how to care for a client with early-stage Alzheimer disease. Which statement would the nurse include? a. "If she is confused, play along and pretend that everything is okay." b. "Remove the clock from her room so that she doesn't get confused." c. "Reorient the client to the day, time, and environment with each contact." d. "Use validation therapy to recognize and acknowledge the client's concerns."

c Clients who have early-stage Alzheimer disease would be reoriented frequently to person, place, and time. The AP would reorient the client and not encourage the client's delusions. The room would have a clock and white board with the current date written on it. Validation therapy is used with late-stage Alzheimer disease.

A client with multiple sclerosis is being discharged from rehabilitation. Which statement would the nurse include in the client's discharge teaching? a. "Be sure that you use a wheelchair when you go out in public." b. "Wear an undergarment brief at all times in case of incontinence." c. "Avoid overexertion, stress, and extreme temperature if possible." d. "Avoid having sexual intercourse to conserve energy."

c Clients who have multiple sclerosis have chronic fatigue and are prone to disease exacerbation (flare-up) is they overexert, are stressed, or are exposed to extreme temperature and humidity. They should not wear briefs unless they have actual problems with continence and should not use a wheelchair if they are able to ambulate with a cane or walker. Maintaining independence and self-esteem is important, so participating in sexual activities is encouraged.

The nurse assesses a client who has a history of migraines. Which symptom would the nurse identify as an early sign of a migraine with aura? a. Vertigo b. Lethargy c. Visual disturbances d. Numbness of the tongue

c Early warning of impending migraine with aura usually consists of visual changes, flashing lights, or diplopia. The other symptoms are not associated with an impending migraine with aura.

The nurse is preparing to teach a client recently diagnosed with multiple sclerosis about taking glatiramer acetate. Which statement by the client indicates a need for further teaching? a. "I will rotate injection sites to prevent skin irritation." b. "I need to avoid large crowds and people with infection." c. "I should report any flulike symptoms to my primary health care provider." d. "I will report any signs of infection to my primary health care provider."

c Glatiramer is given by subcutaneous injection. The first dose is administered under medical supervision, but the nurse teaches the client how to self-administer the medication after the initial dose, reminding the client about the need to rotate injection sites. Like other immunomodulators, this drug can make the client susceptible to infection. However, flulike symptoms occur more commonly with interferons rather than glatiramer.

A client experiences impaired swallowing after a stroke and has worked with speech-language pathology on eating. What nursing assessment best indicates that the expected outcome for this problem has been met? a. Chooses preferred items from the menu. b. Eats 75 to 100% of all meals and snacks. c. Has clear lung sounds on auscultation. d. Gains 2 lb (1 kg) after 1 week.

c Impaired swallowing can lead to aspiration and then aspiration pneumonia, so the expected outcome for this problem is to experience no aspiration. Clear lung sounds is the best indicator that aspiration has not occurred. Choosing menu items is not related to this problem. Eating meals does not indicate that the client is not still aspirating. A weight gain indicates improved nutrition but still does not show a lack of aspiration.

A client who is experiencing a traumatic brain injury has increasing intracranial pressure (ICP). What drug will the nurse anticipate to be prescribed for this client? a. Phenytoin b. Lorazepam c. Mannitol d. Morphine

c Increased intracranial pressure is often the result of cerebral edema as a result of traumatic brain injury. Therefore, as osmotic diuretic such as mannitol or a loop diuretic like furosemide is administered. The other drugs are not appropriate to manage increasing ICP.

A nurse assesses a client who has cholecystitis. Which sign or symptom indicates that this condition is chronic rather than acute? a. Temperature of 100.1° F (37.8° C) b. Positive Murphy sign c. Clay-colored stools d. Upper abdominal pain after eating

c Jaundice, clay-colored stools, and dark urine are more commonly seen with chronic cholecystitis. The other symptoms are seen in clients with either chronic or acute cholecystitis.

After teaching a client who has diverticulitis, a nurse assesses the client's understanding. Which statement made by the client indicates a need for further teaching? a. "I'll ride my bike or take a long walk at least three times a week." b. "I must try to include at least 25 g of fiber in my diet every day." c. "I will take a laxative nightly at bedtime to avoid becoming constipated." d. "I should use my legs rather than my back muscles when I lift heavy objects."

c Laxatives are not recommended for patients with diverticulitis because they can increase pressure in the bowel, causing additional outpouching of the lumen. Exercise and a high-fiber diet are recommended for clients with diverticulitis because they promote regular bowel function. Using the leg muscles rather than the back for lifting prevents abdominal straining.

A nurse cares for a client with colorectal cancer who has a new colostomy. The client states, "I think it would be helpful to talk with someone who has had a similar experience." How would the nurse respond? a. "I have a good friend with a colostomy who would be willing to talk with you." b. "The ostomy nurse will be able to answer all of your questions." c. "I will make a referral to the United Ostomy Associations of America." d. "You'll find that most people with colostomies don't want to talk about them."

c Nurses need to become familiar with community-based resources to better assist clients. The local chapter of the United Ostomy Associations of America has resources for clients and their families, including ostomates (specially trained visitors who also have ostomies). The nurse would not suggest that the client speak with a personal contact of the nurse. Although the ostomy nurse is an expert in ostomy care, talking with him or her is not the same as talking with someone who actually has had a colostomy. The nurse would not brush aside the client's request by saying that most people with colostomies do not want to talk about them. Many people are willing to share their ostomy experience in the hope of helping others.

A nurse is caring for a client with paraplegia who is scheduled to participate in a rehabilitation program. The client states, "I don't understand the need for rehabilitation; the paralysis will not go away and it will not get better." How would the nurse respond? a. "If you don't want to participate in the rehabilitation program, I'll let your primary health care provider know." b. "Rehabilitation programs have helped many patients with your injury. You should give it a chance." c. "The rehabilitation program will teach you how to maintain the functional ability you have and prevent further disability." d. "When new discoveries are made regarding paraplegia, people in rehabilitation programs will benefit first."

c Participation in rehabilitation programs has many purposes, including prevention of disability, maintenance of functional ability, and restoration of function. The other responses do not meet this client's needs.

The nurse is caring for a client who has been diagnosed with peptic ulcer disease. For which complication would the nurse monitor? a. Large bowel obstruction b. Dyspepsia c. Upper gastrointestinal (GI) bleeding d. Gastric cancer

c Peptic ulcer disease (PUD) can cause gastric mucosal damage or perforation, which causes upper GI bleeding. Dyspepsia is a symptom of PUD, gastritis, and gastric cancer. PUD affects the stomach and/or duodenum, not the colon.

A nurse is caring for a client who is about to receive a bone marrow transplant. To best help the client cope with the long recovery period, what action by the nurse is best? a. Arrange a visitation schedule among friends and family. b. Explain that this process is difficult but must be endured. c. Help the client find things to hope for each day of recovery. d. Provide plenty of diversionary activities for this time.

c Providing hope is an essential nursing function during treatment for any disease process, but especially during the recovery period after bone marrow transplantation, which can take up to 3 weeks. The nurse can help the client look ahead to the recovery period and identify things to hope for during this time. Visitors are important to clients, but may pose an infection risk. Telling the client that the recovery period must be endured does not acknowledge his or her feelings. Diversionary activities are important, but not as important as instilling hope.

A client is receiving rituximab and asks how it works. What response by the nurse is best? a. "It causes rapid lysis of the cancer cell membranes." b. "It destroys the enzymes needed to create cancer cells." c. "It prevents the start of cell division in the cancer cells." d. "It sensitizes certain cancer cells to chemo"

c Rituximab prevents the initiation of cancer cell division. The other statements are not accurate.

A client is admitted with superior vena cava syndrome. What action by the nurse is most appropriate? a. Administer a dose of allopurinol. b. Assess the client's serum potassium level. c. Gently inquire about advance directives. d. Prepare the client for emergency surgery.

c Superior vena cava syndrome is often a late-stage manifestation. After the client is stabilized and comfortable, the nurse would initiate a conversation about advance directives. Allopurinol is used for tumor lysis syndrome. Potassium levels are important in tumor lysis syndrome, in which cell destruction leads to large quantities of potassium being released into the bloodstream. Surgery is rarely done for superior vena cava syndrome.

Four clients are receiving tyrosine kinase inhibitors (TKIs). Which of these four clients would the nurse assess first? a. Dry, itchy, peeling skin b. Serum calcium of 9.2 mg/dL (2.3 mmol/L) c. Serum potassium of 2.8 mEq/L (2.8 mmol/L) d. Weight gain of 0.5 lb (1.1 kg) in 1 day

c TKIs can cause electrolyte imbalances. This potassium level is very low, so the nurse would assess this client first. Dry, itchy, peeling skin can be a problem in clients receiving cancer treatments, and the nurse would assess that client next because of the potential for discomfort and infection. This calcium level is normal. TKIs can also cause weight gain, but the client with the low potassium level is more critical.

The nurse is caring for a client who has a risk gene for developing cirrhosis. Which racial/ethnic group has this gene most often? a. Blacks b. Asian/Pacific Islanders c. Latinos d. French

c The Patatin-like phospholipase domain containing 3 gene (PNPLA3) has been identified as a risk gene for cirrhosis, which occurs most often in Latinos when compared to other populations.

The nurse documents the vital signs of a client diagnosed with acute pancreatitis: Apical pulse = 116 beats/min Respirations = 28 breaths/min Blood pressure = 92/50 What complication of acute pancreatitis would the nurse suspect that the client might have? a. Electrolyte imbalance b. Pleural effusion c. Internal bleeding d. Pancreatic pseudocyte

c The client is exhibiting signs of hypovolemia most likely due to internal bleeding or hemorrhage. Due to decreased blood volume, the blood pressure is low and the heart rate increases to compensate for hypovolemia to ensure organ perfusion. Respirations often increase to increase oxygen in the blood.

A nurse assesses a client with a spinal cord injury at level T5. The client's blood pressure is 184/95 mm Hg, and the client presents with a flushed face and blurred vision. After raising the head of the bed, what action would the nurse take next? a. Initiate oxygen via a nasal cannula. b. Recheck the client's blood pressure. c. Palpate the bladder for distention. d. Administer a prescribed beta blocker.

c The client is manifesting symptoms of autonomic dysreflexia. Common causes include bladder distention, tight clothing, increased room temperature, and fecal impaction. If persistent, the client could experience neurologic injury such as s stroke. The other actions are not appropriate for this complication.

A client has thrombocytopenia. What statement indicates that the client understands self-management of this condition? a. "I brush and use dental floss every day." b. "I chew hard candy for my dry mouth." c. "I usually put ice on bumps or bruises." d. "Nonslip socks are best when i walk"

c The client should be taught to apply ice to areas of minor trauma. Flossing is not recommended. Hard foods should be avoided. The client should wear well-fitting shoes when ambulating.

The nurse is preparing a client for discharge from the emergency department after experiencing a transient ischemic attack (TIA). Before discharge, which factor would the nurse identify as placing the client at high risk for a stroke? a. Age greater than or equal to 75 b. Blood pressure greater than or equal to 160/95 c. Unilateral weakness during a TIA d. TIA symptoms lasting less than a minute

c The client who has a TIA is at risk for a stroke is he or she has one-sided (unilateral) weakness during a TIA. Risk factors also include an age greater than or equal to 60, blood pressure greater than or equal to 140/90 (either or both systolic and diastolic), and/or a long duration of TIA symptoms. One minute is not a very long time for symptoms to occur.

The nurse is caring for a client who is diagnosed with a complete small bowel obstruction. For what priority problem is this client most likely at risk? a. Abdominal distention b. Nausea c. Electrolyte imbalance d. Obstipation

c The client who has a small bowel obstruction is at the highest risk for fluid and electrolyte imbalances, especially dehydration and hypokalemia due to profuse vomiting. Nausea, abdominal distention, and obstipation are also usually present, but these problems are not as life threatening as the imbalances in electrolytes.

A client is admitted with a diagnosis of possible strangulated inguinal hernia. For which complication would the nurse monitor? a. Paralytic ileus b. Bowel volvulus c. Sepsis d. Colitis

c The client who has a strangulated inguinal hernia would likely develop bowel necrosis which can lead to sepsis. The nurse would observe for early signs and symptoms of sepsis such as fever, tachypnea, and tachycardia. If the client's condition is not promptly managed, bowel perforation, septic shock, and death can result.

The nurse is teaching a client diagnosed with stomatitis about special mouth care. Which statement by the client indicates a need for further teaching? a. "I need to take out my dentures until my mouth heals." b. "I'll try to eat soft foods that aren't spicy and acidic." c. "I will use a more firm toothbrush to keep my mouth clean." d. "I'll be sure to rinse my mouth often with warm salt water."

c The client who has stomatitis has oral inflammation which causes discomfort. Therefore, all of these actions help to avoid irritation except for needing to use a soft toothbrush or gauze rather than a firm one.

A client has an open traditional hiatal hernia repair this morning. What is the nurse's priority for client care at this time? a. Managing surgical pain b. Ambulating the client early c. Preventing respiratory complications d. Managing the nasogastric tube

c The client who has traditional surgery (rather than minimally invasive surgery) is at risk for respiratory complications such as atelectasis and pneumonia because he or she has an incision that may prevent the client from taking deep breaths or using an incentive spirometer. Therefore, the nurse's priority is to prevent these potentially life-threatening respiratory problems.

A nurse teaches a client who is at risk for colorectal cancer. Which dietary recommendation would the nurse teach the client? a. "Eat low-fiber and low-residual foods." b. "White rice and bread are easier to digest." c. "Add vegetables such as broccoli and cauliflower to your diet." d. "Foods high in animal fat help to protect the intestinal mucosa."

c The client would be taught to modify his or her diet to decrease animal fat and refined carbohydrates. The client should also increase high-fiber foods and Brassica vegetables, including broccoli and cauliflower, which help to protect the intestinal mucosa from colon cancer.

After teaching a client who is prescribed pancreatic enzyme replacement therapy, the nurse assesses the client's understanding. Which statement by the client indicates a need for further teaching? a. "The capsules can be opened and the powder sprinkled on applesauce if needed." b. "I will wipe my lips carefully after I drink the enzyme preparation." c. "The best time to take the enzymes is immediately after I have a meal or a snack." d. "I will not mix the enzyme powder with food or liquids that contain protein."

c The enzymes must be taken immediately before eating meals or snacks. If the client cannot swallow the capsules whole, they can be opened up and the powder sprinkled on applesauce, mashed fruit, or rice cereal. The client should wipe his or her lips carefully after drinking the enzyme preparation because the liquid could damage the skin. Protein items will be dissolved by the enzymes if they are mixed together

The nurse is teaching a client how to avoid the formation of hemorrhoids. What lifestyle change would the nurse include? a. Avoiding alcohol b. Quitting smoking c. Decreasing fluid intake d. Increasing dietary fiber

c The major cause of hemorrhoid formation is constipation. Therefore, the nurse teaches the client ways to prevent constipation, which include increasing dietary fiber, increasing exercise and fluid intake, and avoiding straining when have a stool.

The nurse assesses a client who is hospitalized with an exacerbation of Crohn disease. Which assessment finding would the nurse expect? a. Positive Murphy sign with rebound tenderness to palpitation b. Dull, hypoactive bowel sounds in the lower abdominal quadrants c. High-pitched, rushing bowel sounds in the right lower quadrant d. Reports of abdominal cramping that is worse at night

c The nurse expects high-pitched, rushing bowel sounds due to narrowing of the bowel lumen in Crohn disease. A positive Murphy sign is indicative of gallbladder disease, and rebound tenderness often indicates peritonitis. Dullness in the lower abdominal quadrants and hypoactive bowel sounds is not commonly found with Crohn disease. Nightly worsening of abdominal cramping is not consistent with Crohn disease.

The nurse witnesses a client begin to experience a tonic-clonic seizure and loss of consciousness. What action would the nurse take first? a. Start fluids via a large-bore catheter. b. Administer IV push diazepam. c. Turn the client's head to the side. d. Prepare to intubate the client.

c The nurse would turn the client's head to the side to prevent aspiration and allow drainage of secretions. Anticonvulsants are administered on a routine basis if a seizure is sustained. If the seizure is sustained (status epilepticus), the client must be intubated and would be administered oxygen, 0.9% sodium chloride, and IV push lorazepam or diazepam.

A nurse cares for a client with a spinal cord injury. With which interprofessional health team member would the nurse collaborate to assist the client with activities of daily living? a. Social worker b. Physical therapist c. Occupational therapist d. Case manager

c The occupational therapist instructs the patient in the correct use of all adaptive equipment. In collaboration with the therapist, the nurse instructs family members or the caregiver about transfer skills, feeding, bathing, dressing, positioning, and skin care. The other team members are consulted to assist the client with other issues.

A nurse cares for a young client with a new ileostomy. The client states, "I cannot go to prom with an ostomy." How would the nurse respond? a. "Sure you can. Purchase a prom dress one size larger to hide the ostomy appliance." b. "The pouch won't be as noticeable if you avoid broccoli and carbonated drinks prior to the prom." c. "Let's talk to the ostomy nurse about options for ostomy supplies and dress styles." d. "You can remove the pouch from your ostomy appliance when you are at the prom so that it is less noticeable."

c The ostomy nurse is a valuable resource for patients, providing suggestions for supplies and methods to manage the ostomy. A larger dress size will not necessarily help hide the ostomy appliance. Avoiding broccoli and carbonated drinks does not offer reassurance for the client. Ileostomies have an almost constant liquid effluent, so pouch removal during the prom is not feasible.

A nurse assessing a client with colorectal cancer auscultates high-pitched bowel sounds and notes the presence of visible peristaltic waves. Which action would the nurse take? a. Ask if the client is experiencing pain in the right shoulder. b. Perform a rectal examination and assess for polyps. c. Recommend that the client have computed tomography. d. Administer a laxative to increase bowel movement activity.

c The presence of visible peristaltic waves, accompanied by high-pitched or high-pitched bowel sounds, is indicative of bowel obstruction caused by the tumor. The nurse would contact the primary health care provider with these results and recommend a computed tomography scan for further diagnostic testing. This assessment finding is not associated with right shoulder pain; peritonitis and cholecystitis are associated with referred pain to the right shoulder. The nurse generalist is not qualified to complete a rectal examination for polyps, and laxatives would not help this client.

The nurse is assessing a client in sickle cell disease (SCD) crisis. What priority client problem will the nurse expect? a. Infection b. Pallor c. Pain d. Fatigue

c The priority expected client problem for clients experiencing sickle cell disease crisis is pain, often concentrated in the legs, arms, and joints. Clients may also be fatigued and pale but these symptoms are not a priority for care. Infection is not expected but can occur in clients who have SCD crisis.

A client with peptic ulcer disease is in the emergency department and reports gastric pain that has gotten much worse over the last 24 hours. The client's blood pressure when lying down is 112/68 mm Hg and when standing is 98/52 mm Hg. What action by the nurse is most appropriate? a. Administer a proton pump inhibitor (PPI). b. Call the Rapid Response Team. c. Start a large-bore IV with normal saline. d. Tell the patient to remain lying down.

c This client has orthostatic changes to the blood pressure, indicating fluid volume loss. The nurse would start a large-bore IV with isotonic solution. PPIs are not a treatment for an ulcer. The Rapid Response Team is not needed at this point. The client should be put on safety precautions, which includes staying in bed, but this is not the most appropriate action at this time.

A nurse has taught a client about dietary changes that can reduce the chances of developing cancer. What statement by the client indicates the nurse needs to provide additional teaching? a. "Foods high in vitamin A and vitamin C are important." b. "I'll have to cut down on the amount of bacon I eat." c. "I'm so glad I don't have to give up my juicy steaks." d. "Vegetables, fruit, and high-fiber grains are important."

c To decrease the risk of developing cancer, one should cut down on the consumption of red meats and animal fat. The other statements are correct.

The nurse is teaching the daughter of a client who has middle-stage Alzheimer disease. The daughter asks, "Will the sertraline my mother is taking improve her dementia?" How would the nurse respond about the purpose of the drug? a. "It will allow your mother to live independently for several more years." b. "It is used to halt the advancement of Alzheimer disease but will not cure it." c. "It will not improve her dementia but can help control emotional responses." d. "It is used to improve short-term memory but will not improve problem solving."

c Drug therapy is not effective for treating dementia or halting the advancement of Alzheimer disease. However, certain psychoactive drugs may help suppress emotional disturbances and manage depression, psychoses, or anxiety. Drug therapy will not allow the client with middle-stage dementia to safely live independently.

A nurse assesses a client who is recovering from the implantation of a vagal nerve-stimulation device. For which signs and symptoms would the nurse assess as common complications of this procedure? (Select all that apply.) a. Bleeding b. Infection c. Hoarseness d. Dysphagia e. Seizures

c d Complications of surgery to implant a vagal nerve-stimulation device include hoarseness (most common), dyspnea, neck pain, and dysphagia. The device is tunneled under the skin with an electrode connected to the vagus nerve to control simple or complex partial seizures. Bleeding is not a common complication of this procedure, and infection would not occur during the recovery period.

The nurse assesses a client who has possible gastritis. Which assessment finding(s) indicate(s) that the client has chronic gastritis? (Select all that apply.) a. Anorexia b. Dyspepsia c. Intolerance of fatty foods d. Pernicious anemia e. Nausea and vomiting

c d Intolerance of fatty or spicy foods and pernicious anemia are signs of chronic gastritis. Anorexia and nausea/vomiting can be seen in both conditions. Dyspepsia is seen in acute gastritis.

A nurse assesses a client who is recovering from an open traditional lumbar laminectomy with fusion. Which complications would the nurse report to the primary health care provider? (Select all that apply.) a. Surgical discomfort b. Redness and itching at the incision site c. Incisional bulging d. Clear drainage on the dressing e. Sudden and severe headache

c d e Bulging at the incision site or clear fluid on the dressing after open back surgery strongly suggests a cerebrospinal fluid leak, which constitutes an emergency. Loss of cerebrospinal fluid may cause a sudden and severe headache. Pain, redness, and itching at the site are normal.

After teaching a client who is recovering from a colon resection to treat early-stage colorectal cancer (CRC), the nurse assesses the client's understanding. Which statements by the client indicate understanding of the teaching? (Select all that apply.) a. "I must change the ostomy appliance daily and as needed." b. "I will use warm water and a soft washcloth to clean around the stoma." c. "I might start bicycling and swimming again once my incision has healed." d. "I will make sure that I make lifestyle changes to prevent constipation." e. "I will be sure to have the recommended colonoscopies."

c d e The client has had a colon resection for early CRC and there is no indication that the client also had a colostomy. Follow up with recommended colonoscopies are essential to monitor for CRC recurrence. Avoiding constipation will help improve intestinal motility which helps to decrease the risk for CRC recurrence. Exercise and other activities do not need to be restricted after the client has healed.

After teaching a male client with a spinal cord injury at the T4 level, the nurse assesses the his understanding. Which client statements indicate a correct understanding of the teaching related to sexual effects of his injury? (Select all that apply.) a. "I will explore other ways besides intercourse to please my partner." b. "I will not be able to have an erection because of my injury." c. "Ejaculation may not be as predictable as before." d. "I may urinate with ejaculation but this will not cause infection." e. "I should be able to have an erection with stimulation."

c d e Men with injuries above T6 often are able to have erections by stimulating reflex activity. For example, stroking the penis will cause an erection. Ejaculation is less predictable and may be mixed with urine. However, urine is sterile, so the client's partner will not get an infection.

A client has received a bone marrow transplant and is waiting for engraftment. What action(s) by the nurse are most appropriate? (Select all that apply.) a. Not allowing any visitors until engraftment b. Limiting the protein in the client's diet c. Placing the client in protective precautions d. Teaching visitors appropriate hand hygiene e. Telling visitors not to bring live flowers or plants

c d e The client waiting for engraftment after bone marrow transplant has no white cells to protect him or her against infection. The client is on protective precautions and visitors are taught hand hygiene. No fresh flowers or plants are allowed due to the standing water in the vase or container that may harbor organisms; clients are also told not to work with houseplants in the home. Limiting protein is not a healthy option and will not promote engraftment.

A nurse assesses a client with paraplegia from a spinal cord injury and notes reddened areas over the client's hips and sacrum. What actions would the nurse take? (Select all that apply.) a. Apply a barrier cream to protect the skin from excoriation. b. Perform range-of-motion (ROM) exercises for the hip joint. c. Reposition the client off of the reddened areas. d. Get the client out of bed and into a chair several times a day. e. Apply a pressure-reducing mattress.

c e Appropriate interventions to relieve pressure on the reddened areas include frequent repositioning, using a pressure-reducing mattress, and having the client sit in a chair to remove pressure from the hips and sacrum. Correct sitting position would allow the pressure to be on both ischial tuberosities. ROM exercises are used to prevent contractures.

The nurse is collaborating with the occupational therapist to assist a client with a complete cervical spinal cord injury to transfer from the bed to the wheelchair. What ambulatory aid would be most appropriate for the client to meet this outcome? a. Rolling walker b. Quad cane c. Adjustable crutches d. Sliding board

d A client who has a complete cervical spinal cord injury is unable to use any extremity except for parts of the hands and possibly the lower arms. Therefore, the client would be unable to use any of these ambulatory aids except for a sliding board, also known as a slider, which provides a "bridge" between the bed and a chair. The client uses his or her arms in a locked position to support the body while moving slowly across the board.

The nurse is caring for four clients with traumatic brain injuries. Which client would the nurse assess first? a. Client with amnesia for the incident b. Client who has a Glasgow Coma Scale score of 12 c. Client with a PaCO2 of 36 mm Hg and on a ventilator d. Client who has a temperature of 102° F (38.9° C)

d A fever is a poor prognostic indicator in patients with brain injuries. The nurse should see this client first. A Glasgow Coma Scale score of 12, a PaCO2 of 36, and amnesia for the incident are all either expected or positive findings.

A nurse is assessing a client with glioblastoma. What assessment is most important? a. Abdominal palpation b. Abdominal percussion c. Lung auscultation d. Neurologic examination

d A glioblastoma arises in the brain. The most important assessment for this client is the neurologic examination.

Which of these client assessment findings is typically associated with oral cancer? a. Dry sticky oral membranes b. Increased appetite c. Itchy rash in oral cavity d. Painless red or raised lesion

d A painless red or raised lesion often indicates a diagnosis of oral cancer. The client usually has a decreased appetite and thick secretions. Itchiness is not a common finding associated with oral cancer.

The nurse is caring for a client scheduled to have a transjugular intrahepatic portal-systemic shunt (TIPS) procedure. What client assessment would the nurse perform prior to this procedure? a. Musculoskeletal assessment b. Neurologic assessment c. Mental health assessment d. Cardiovascular assessment

d A postprocedure complication of a TIPS procedure is right-sided heart failure. Therefore, the nurse would perform a cardiovascular assessment before the procedure to determine if the client has signs and symptoms of heart failure.

A nurse works with clients who have alopecia from chemotherapy. What action by the nurse takes priority? a. Helping clients adjust to their appearance b. Reassuring clients that this change is temporary c. Referring clients to a reputable wig shop d. Teaching measures to prevent scalp injury

d All of the actions are appropriate for clients with alopecia. However, the priority is client safety, so the nurse would first teach ways to prevent scalp injury.

The nurse teaches a client about how to prevent transmission of gastroenteritis. Which statement by the nurse indicates a need for further teaching? a. "I won't let anyone use my dishes or glasses." b. "I'll wash my hands with antibacterial soap." c. "I'll keep my bathroom extra clean." d. "I'll cook all the meals for my family."

d All of these statements are correct except for that the client should not prepare meals for others to help prevent transmission of gastroenteritis.

A client with multiple myeloma demonstrates worsening bone density on diagnostic scans. About what drug does the nurse plan to teach this client? a. Bortezomib b. Dexamethasone c. Thalidomide d. Zoledronic acid

d All the options are drugs used to treat multiple myeloma, but the drug used specifically for bone manifestations is zoledronic acid, which is a bisphosphonate. This drug class inhibits bone resorption and is used to treat osteoporosis as well.

The nurse is preparing to administer IV alteplase for a client diagnosed with an acute ischemic stroke. Which statement is correct about the administration of this drug? a. The recommended time for drug administration is within 90 minutes after admission to the emergency department. b. The drug is given in a bolus over the first 3 minutes followed by a continuous infusion. c. The maximum dosage of the drug, including the bolus, is 120 mg intravenously. d. The drug is not given to clients who are already on anticoagulant or antiplatelet therapy.

d Alteplase is a thrombolytic which dissolves clots and can cause bleeding as an adverse effect. Clients who are already taking an anticoagulant or antiplatelet agent are at risk for bleeding and therefore they are not candidates for alteplase therapy.

A client diagnosed with Parkinson disease will be starting ropinirole for symptom control. Which statement by the client indicates a need for further teaching? a. "This drug should help decrease my tremors and help me move better." b. "I need to change positions slowly to prevent dizziness or falls." c. "I should take the drug at the same time each day for the best effect." d. "I know the drug will probably make help me prevent constipation."

d Although ropinirole is a dopamine agonist and mimics dopamine to promote movement, it does not work to prevent constipation. This class of drugs can cause orthostatic hypotension and should be taken at the same time every day.

A nurse asks the staff development nurse what "apoptosis" means. What response best? a. Growth by cells enlarging b. Having the normal number of chromosomes c. Inhibition of cell growth d. Programmed cell death

d Apoptosis is programmed cell death. With this characteristic, organs and tissues function with cells that are at their peak of performance. Growth by cells enlarging is hyperplasia. Having the normal number of chromosomes is euploidy. Inhibition of cell growth is contact inhibition.

After teaching the wife of a client who has Parkinson disease, the nurse assesses the wife's understanding. Which statement by the client's wife indicates that she correctly understands changes associated with this disease? a. "His masklike face makes it difficult to communicate, so I will use a white board." b. "He should not socialize outside of the house due to uncontrollable drooling." c. "This disease is associated with anxiety causing increased perspiration." d. "He may have trouble chewing, so I will offer bite-sized portions."

d Because chewing and swallowing can be problematic, small frequent meals and a supplement are better for meeting the client's nutritional needs. A masklike face and drooling are common in clients with Parkinson disease. The client would be encouraged to continue to socialize and communicate as normally as possible. The wife should understand that the client's masklike face can be misinterpreted and additional time may be needed for the client to communicate with her or others. Excessive perspiration is also common in clients with Parkinson disease and is associated with the autonomic nervous system's response.

A client is scheduled for a hepatobiliary iminodiacetic acid (HIDA) scan. What would the nurse include in client teaching about this diagnostic test? a. "You'll have to drink a contrast medium right before the test." b. "You'll need to do a bowel prep the nursing before the test." c. "You'll be able to drink liquids up until the test begins." d. "You'll have a large camera close to you during the test."

d Clients having a HIDA scan are NPO and receive an injectable nuclear medicine contrast. No bowel preparation is required. A large camera is close to the client for most of the test which can be a problem for clients who are claustrophobic.

A nurse is assessing a female client who is taking hormone therapy for breast cancer. What assessment finding requires the nurse to notify the primary health care provider immediately? a. Irregular menses b. Edema in the lower extremities c. Ongoing breast tenderness d. Red, warm, swollen calf

d Clients receiving hormone therapy are at risk for thromboembolism. A red, warm, swollen calf is indicative of deep vein thrombosis and would be reported to the provider. Irregular menses, edema in the lower extremities, and breast tenderness are not as urgent as the possible thromboembolism.

The nurse has taught a client with cancer ways to prevent infection. What statement by the client indicates that more teaching is needed? a. "I should take my temperature daily and when I don't feel well." b. "I will discard perishable liquids after sitting out for over an hour." c. "I won't let anyone share any of my personal toiletries." d. "It's alright for me to keep my pets and change the litter box."

d Clients should wash their hands after touching their pets and would not empty or scoop the cat litter box. The other statements are appropriate for self-management.

After teaching a patient with diverticular disease, a nurse assesses the client's understanding. Which menu selection indicates the client correctly understood the teaching? a. Roasted chicken with rice pilaf and a cup of coffee with cream b. Spaghetti with meat sauce, a fresh fruit cup, and hot tea c. Garden salad with a cup of bean soup and a glass of low-fat milk d. Baked fish with steamed carrots and a glass of apple juice

d Clients who have diverticular disease are prescribed a low-residue diet. Whole grains (rice pilaf), uncooked fruits and vegetables (salad, fresh fruit cup), and high-fiber foods (cup [240 mL] of bean soup) would be avoided with a low-residue diet. Canned or cooked vegetables are appropriate. Apple juice does not contain fiber and is acceptable for a low-residue diet

After teaching a client who has a history of cholelithiasis, the nurse assesses the client's understanding. Which menu selection indicates that the client understands the dietary teaching? a. Lasagna, tossed salad with Italian dressing, and low-fat milk b. Grilled cheese sandwich, tomato soup, and coffee with cream c. Cream of potato soup, Caesar salad with chicken, and a diet cola d. Roasted chicken breast, baked potato with chives, and orange juice

d Clients with cholelithiasis should avoid foods high in fat and cholesterol, such as whole milk, butter, and fried foods. Lasagna, low-fat milk, grilled cheese, cream, and cream of potato soup all have high levels of fat. The meal with the least amount of fat is the chicken breast dinner.

The nurse prepares to discharge a client with early to moderate Alzheimer disease. Which statement to maintain client safety would the nurse include in the discharge teaching for the caregiver? a. "Provide periods of exercise and rest for the client." b. "Place a padded throw rug at the bedside." c. "Provide a highly stimulating environment." d. "Install safety locks on all outside doors."

d Clients with early to moderate Alzheimer disease have a tendency to wander, especially at night. If possible, alarms would be installed on all outside doors to alert family members if the client leaves. At a minimum, all outside doors should have safety locks installed to prevent the client from going outdoors unsupervised. The client would be allowed to exercise within his or her limits, but this action does not ensure his or her safety. Throw rugs are a slip and fall hazard and would be removed. A highly stimulating environment would likely increase the client's confusion.

A nurse assesses clients at a community health center. Which client is at highest risk for developing colorectal cancer? a. A 37-year-old who drinks eight cups of coffee daily. b. A 44-year-old with irritable bowel syndrome (IBS). c. A 60-year-old lawyer who works 65 hours per week. d. A 72-year-old who eats fast food frequently.

d Colon cancer is rare before the age of 40, but its incidence increases rapidly with advancing age. Fast food tends to be high in fat and low in fiber, increasing the risk for colon cancer. Coffee intake, IBS, and a heavy workload do not increase the risk for colon cancer.

After teaching a client who is diagnosed with new-onset epilepsy and prescribed phenytoin, the nurse assesses the client's understanding. Which statement by the client indicates a correct understanding of the teaching? a. "To prevent complications, I will drink at least 2 L of water daily." b. "This medication will stop me from getting an aura before a seizure." c. "I will not drive a motor vehicle while taking this medication." d. "Even when my seizures stop, I will continue to take this drug."

d Discontinuing antiepileptic drugs can lead to the recurrence of seizures or status epilepticus. The client does not need to drink more water and can drive while taking this medication. The drug will not stop an aura before a seizures

The primary health care provider prescribes donepezil for a client diagnosed with early-stage Alzheimer disease. What teaching about this drug will the nurse provide for the client's family caregiver? a. "Monitor the client's temperature because the drug can cause a low grade fever." b. "Observe the client for nausea and vomiting to determine drug tolerance." c. "Donepezil will prevent the client's dementia from progressing as usual." d. "Report any client dizziness or falls because the drug can cause bradycardia."

d Donepezil is a cholinesterase inhibitor that may temporarily slow cognitive decline for some clients but does not alter the course of the disease. The family caregiver would want to monitor the client's heart rate and report any incidence of dizziness or falls because the drug can cause bradycardia. It does not typically cause fever or nausea/vomiting.

A client had an open traditional Whipple procedure this morning. For what priority complication would the nurse assess? a. Urinary tract infection b. Chronic kidney disease c. Heart failure d. Fluid and electrolyte imbalances

d Due to the length and complexity of this type of surgery, the client is at risk for fluid and electrolyte imbalances. The nurse would assess for signs and symptoms of these imbalances so they can be managed early to prevent potentially life-threatening complications.

A client with cancer is admitted to a short-term rehabilitation facility. The nurse prepares to administer the client's oral chemotherapy medications. What action by the nurse is most appropriate? a. Crush the medications if the client cannot swallow them. b. Give one medication at a time with a full glass of water. c. No special precautions are needed for these medications. d. Wear personal protective equipment when handling the medications.

d During the administration of oral chemotherapy agents, nurses must take the same precautions that are used when administering IV chemotherapy. This includes using personal protective equipment. These medications cannot be crushed, split, or chewed. Giving one at a time is not needed.

The nurse plans care for a client with Parkinson disease. Which intervention would the nurse include in this client's plan of care? a. Restrain the client to prevent falling. b. Ensure that the client uses incentive spirometry. c. Teach the client pursed-lip breathing techniques. d. Keep the head of the bed at 30 degrees or greater.

d Elevation of the head of the bed will help prevent aspiration. The other options will not prevent aspiration, which is the greatest respiratory complication of Parkinson disease, nor do these interventions address any of the complications of Parkinson disease. Pursed-lip breathing increases exhalation of carbon dioxide; incentive spirometry expands the lungs. The client should not be restrained to prevent falls. Other less restrictive interventions should be used to maintain client safety.

The nurse is caring for a client who is scheduled for a paracentesis. Which action is appropriate for the nurse to take? a. Have the client sign the informed consent form. b. Get the patient into a chair before the procedure. c. Help the client lie flat in bed on the right side. d. Assist the client to void before the procedure.

d For safety, the patient would void just before a paracentesis to prevent bladder damage to the procedure. The primary health care provider would have the client sign the consent form. The proper position for a paracentesis is sitting upright in bed or, alternatively, sitting on the side of the bed and leaning over the bedside table.

The nurse is caring for a client experiencing sickle cell disease crisis. Which priority action would help prevent infection? a. Administering prophylactic antibiotics b. Monitoring the client's temperature c. Checking the client's white blood cell count d. Performing frequent handwashing

d Frequent and thorough handwashing is the most important intervention that helps prevent infection. Antibiotics are not usually used to prevent infection. Monitoring the client's temperature or white blood cell count helps to detect the presence of infection, but prevent it

A client asks about the process of graft-versus-host disease. What explanation by the nurse is correct? a. "Because of immunosuppression, the donor cells take over." b. "It's like a transfusion reaction because no perfect matches exist." c. "The patient's cells are fighting donor cells for dominance." d. "The donor's cells are actually attacking the patient's cells."

d Graft-versus-host disease is an autoimmune-type process in which the donor cells recognize the client's cells as foreign and begin attacking them. The other answers are not accurate.

The nurse is caring for a client with a large bowel obstruction due to fecal impaction. What position would be appropriate for the client while in bed? a. Prone b. Supine c. Recumbent d. Semi-Fowler

d Having the client in a semi-sitting position helps to decrease the pressure caused by abdominal distention and promotes thoracic expansion to facilitate breathing.

The nurse is caring for a client who has cirrhosis of the liver. Which risk factor is the leading cause of cirrhosis? a. Metabolic syndrome b. Liver cancer c. Nonalcoholic fatty liver disease d. Hepatitis C

d Hepatitis C is the leading cause of cirrhosis and an also cause liver cancer. Clients with nonalcoholic fatty liver disease often have metabolic syndrome and can also develop cirrhosis.

A nurse receives a hand-off report on a female client who had a left-sided stroke with homonymous hemianopsia. What action by the nurse is most appropriate for this client? a. Assess for bladder and bowel retention and/or incontinence. b. Listen to the client's lungs after eating or drinking for diminished breath sounds. c. Support the client's left side when sitting in a chair or in bed. d. Remind the client to move her head from side to side to increase her visual field.

d Homonymous hemianopsia is blindness on the same side of both eyes. The client must turn his or her head to see the entire visual field. This condition is not related to bladder function, difficulty swallowing, or lack

An assistive personnel is caring for a client with leukemia and asks why the client is still at risk for infection when the white blood cell count (WBC) is high. What response by the nurse is correct? a. "If the WBCs are high, there already is an infection present." b. "The client is in a blast crisis and has too many WBCs." c. "There must be a mistake; the WBCs should be very low." d. "Those WBCs are abnormal and don't provide protection."

d In leukemia, the WBCs are abnormal and do not provide protection to the client against infection. The other statements are not accurate.

The nurse is preparing to teach a client with chronic hepatitis B about lamivudine therapy. What health teaching would the nurse include? a. "Follow up on all appointments to monitor your lab values." b. "Do not take amiodorone at any time while on this drug." c. "Monitor for jaundice, rash, and itchy skin while on this drug." d. "Report any changes in urinary elimination while on this drug."

d Lamivudine can cause renal impairment and the nurse would remind the client of changes that may indicate kidney damage.

The nurse caring for clients with gastrointestinal disorders would recall that omeprazole is a drug in which classification? a. Gastric acid inhibitor b. Histamine receptor blocker c. Mucosal barrier fortifier d. Proton pump inhibitor

d Omeprazole is a proton pump inhibitor.

A nurse assesses clients at a community center. Which client is at greatest risk for low back pain? a. A 24-year-old female who is 25 weeks pregnant. b. A 36-year-old male who uses ergonomic techniques. c. A 53-year-old female who uses a walker. d. A 65-year-old female with osteoarthritis.

d Osteoarthritis causes changes to support structures, increasing the client's risk for low back pain. The other clients are not at high risk

The nurse assesses a client with gastroenteritis. What risk factor would the nurse consider as the most likely cause of this disorder? a. Consuming too much fruit b. Consuming fried or pickled foods c. Consuming dairy products d. Consuming raw seafood

d Raw seafood is often contaminated and unless cooked can would most likely cause gastroenteritis. Any of the other food can also become contaminated if not stored properly or contaminated by workers/cooks who contaminate these foods.

The nurse assesses a client with a history of epilepsy who experiences stiffening of the muscles of the arms and legs, followed by an immediate loss of consciousness and jerking of all extremities. How would the nurse document this type of seizure? a. Atonic b. Myoclonic c. Absence d. Tonic-clonic

d Seizure activity that begins with stiffening of the arms and legs, followed by loss of consciousness and jerking of all extremities, is characteristic of a tonic-clonic seizure. An atonic seizure presents as a sudden loss of muscle tone followed by postictal confusion. A myoclonic seizure presents with a brief jerking or stiffening of extremities that may occur singly or in groups. Absence seizures present with automatisms, and the client is unaware of his or her environment.

A client is in the oncology clinic for a first visit since being diagnosed with cancer. The nurse reads in the client's chart that the cancer classification is TISN0M0. What does the nurse conclude about this client's cancer? a. The primary site of the cancer cannot be determined. b. Regional lymph nodes could not be assessed. c. There are multiple lymph nodes involved already. d. There are no distant metastases noted in the report.

d TIS stands for carcinoma in situ; N0 stands for no regional lymph node metastasis; and M0 stands for no distant metastasis.

A client with a severe traumatic brain injury has an organ donor card in his wallet. Which nursing action is appropriate? a. Request a directive form the client's primary health care provider. b. Ask the family if they agree to organ donation for the client. c. Wait until brain death is determined before acting on organ donation. d. Contact the local organ procurement organization as soon as possible.

d The appropriate nursing action is to respect the client's desire to be an organ donor and contact the local organ procurement organization even if family members do not agree. In most agencies, the primary health care provider does not have to write an order or directive to approve the organ donation. Family consent is not required.

The nurse is caring for a client who had a hemorrhagic stroke. Which assessment finding is the earliest sign of increasing intracranial pressure (ICP) for this client? a. Projectile vomiting b. Dilated and nonreactive pupils c. Severe hypertension d. Decreased level of consciousness

d The earliest sign of increasing ICP is decreased level of consciousness. The other signs occur later.

The nurse initiates care for a client with a cervical spinal cord injury who arrives via emergency medical services. What action would the nurse take first? a. Assess level of consciousness. b. Obtain vital signs. c. Administer oxygen therapy.

d The first priority for a client with a spinal cord injury is assessment of respiratory status and airway patency. Clients with cervical spine injuries are particularly prone to respiratory compromise due to interference with diaphragmatic innervation. The other actions would be performed after airway and breathing are assessed.

The nurse is taking a history from a daughter about her father's onset of stroke signs and symptoms. Which statement by the daughter indicates that the client likely had an embolic stroke? a. Client's symptoms occurred slowly over several hours. b. Client because increasingly lethargic and drowsy. c. Client reported severe headache before other symptoms. d. Client has a long history of atrial fibrillation.

d The major cause of embolic strokes is a history of heart disease, especially atrial fibrillation. Most clients who have an embolic stroke have acute sudden neurologic symptoms but stay alert rather than lethargic. Decreasing level of consciousness and severe headache are more common in clients who have hemorrhagic strokes.

After teaching a client newly diagnosed with epilepsy, the nurse assesses the client's understanding. Which statement by the client indicates a need for additional teaching? a. "I will wear my medical alert bracelet at all times." b. "While taking my medications, I will not drink any alcoholic beverages." c. "I will tell my doctor about my prescription and over-the-counter medications." d. "If I am nauseated, I will not take my epilepsy medication."

d The nurse must emphasize that antiepileptic drugs must be taken even if the client is nauseated. Discontinuing the medication can predispose the client to seizure activity and status epilepticus. The client should not drink alcohol while taking seizure medications. The client should wear a medical alert bracelet and should make the primary health care provider aware of all drugs he or she is taking to prevent complications of polypharmacy.

A nurse cares for a client who has a family history of colorectal cancer. The client states, "My father and my brother had colon cancer. What is the chance that I will get cancer?" How would the nurse respond? a. "If you eat a low-fat and low-fiber diet, your chances decrease significantly." b. "You are safe. This is an autosomal dominant disorder that skips generations." c. "Preemptive surgery and chemotherapy will remove cancer cells and prevent cancer." d. "You should have a colonoscopy more frequently to identify abnormal polyps early."

d The nurse would encourage the patient to have frequent colonoscopies to identify abnormal polyps and cancerous cells early. The abnormal gene associated with colon cancer is an autosomal dominant gene mutation that does not skip a generation and places the client at high risk for cancer. Changing the client's diet to more high-fiber (not low-fiber) and preemptive chemotherapy may decrease the client's risk of colon cancer but will not prevent it.

A client with a stroke is being evaluated for fibrinolytic therapy. What information from the client or family is most important for the nurse to obtain? a. Loss of bladder control b. Other medical conditions c. Progression of symptoms d. Time of symptom onset

d The time limit for initiating fibrinolytic therapy for a stroke is 3 to 4.5 hours, so the exact time of symptom onset is the most important information for this client. The other information is not as critical.

A new nurse has been assigned a client who is in the hospital to receive iodine-131 treatment. Which action by the nurse is best? a. Ensure the client is placed in protective isolation. b. Have pregnant visitors stay 6 feet from the client c. No special action is necessary to care for this client. d. Read the policy on handling radioactive excreta.

d This type of radioisotope is excreted in body fluids and excreta (urine and feces) and would not be handled directly. The nurse would read the facility's policy for handling and disposing of this type of waste. The other actions are not warranted.

A nurse is preparing to administer a blood transfusion. Which action is most important? a. Document the transfusion. b. Place the client on NPO status. c. Place the client in isolation. d. Put on a pair of gloves.

d To prevent bloodborne illness, the nurse should don a pair of gloves prior to hanging the blood. Documentation is important but not the priority at this point. NPO status and isolation are not needed.

Which statement about carcinogenesis is accurate? a. An initiated cell will always become clinical cancer. b. Cancer becomes a health problem once it is 1 cm in size. c. Normal hormones and proteins do not promote cancer growth. d. Tumor cells need to develop their own blood supply.

d Tumors need to develop their own blood supply through a process called angiogenesis. An initiated cell needs a promoter to continue its malignant path. Normal hormones and proteins in the body can act as promoters. A 1-cm tumor is a detectable size, but other events have to occur for it to become a health problem.

The nurse is teaching a client about the use of viscous lidocaine for oral pain. What health teaching would the nurse include? a. "Use the drug before every meal to prevent aspiration." b. "Increase your intake of citrus foods to help with healing." c. "Use the drug only at bedtime because you won't be eating." d. "Be sure to check food temperatures before eating."

d Viscous lidocaine has an anesthetic effect in the oral cavity. Therefore, to promote client safety, the nurse would want to teach the client to check food temperature before eating.

A client continues to have persistent low back pain even after using a number of nonpharmacologic pain management strategies. Which prescribed drug would the nurse anticipate that the client might need to manage the pain? a. Oxycontin b. Gabapentin c. Lorazepam d. Tramadol

d When nonpharmacologic strategies, including physical therapy, are not effective in managing pain, current standards recommend a mild opioid such as tramadol or serotonin-norepinephrine reuptake inhibitor. Strong opioids such as oxycontin and benzodiazepines such as lorazepam are not considered best practice.

Which statement by a client with leukemia indicates a need for further teaching by the nurse? a. "I will use a soft-bristled toothbrush and avoid flossing." b. "I will not take aspirin or any aspirin product." c. "I will use an electric shaver instead of my manual one." d. "I will take a daily laxative to prevent constipation."

d The client experiencing leukemia needs to prevent injury to prevent bleeding, including avoiding hard-bristled toothbrushes, floss, aspirin, and straight or manual safety razors. However, although constipation can cause hemorrhoids or rectal bleeding, laxatives can cause fluid and electrolyte imbalances and abdominal cramping. Stool softeners would be a better option to allow the passage of soft stool.

A nurse prepares to provide perineal care to a client with meningococcal meningitis. Which personal protective equipment would the nurse wear? (Select all that apply.) a. Particulate respirator b. Isolation gown c. Shoe covers d. Surgical mask e. Gloves

d e Meningococcal meningitis is spread via saliva and droplets, and Droplet Precautions are necessary. Caregivers would wear a surgical mask when within 6 feet (1.8 m) of the client and would continue to use Standard Precautions, including gloves. A particulate respirator, an isolation gown, and shoe covers are not necessary for Droplet Precautions.


Conjuntos de estudio relacionados

Children's Exam 3 Practice Questions

View Set

Ex_04_01_ Fill in the blanks with "must", "should" or "ought (to)". Use the correct form of the infinitive

View Set

Social Psych Mid-Term #3 Chapter 9

View Set

Chapter 1- basic principles of insurance

View Set

BAS 282: New Product Development: SmartBook

View Set